You are on page 1of 362
CLC ae cue Pierre eek eased GATE & ESE 2017 Prelims Thoroughly Revised and Updated Reasoning & Aptitude Also useful for UPSC (CSAT), MBA Entrance, Wipro, SSC, Bank (PO), TCS, Railways, Infosys, various Public Sector Units and other Competitive Exams conducted by UPSC i Reasoning & Aptitude For Various Competitive Examinations + Infosys + UPC | + wipro + ssc © Railways | * TCs | + MBA Entrance © OL * Bank (PO), ete by Nem Singh (IRS) MADE EASY Publications A_unit of MADE EASY Group Corporate Office: 44-A/1, Kalu Sarai, New Delhi-16 | Regd. Office: 25-A, Ber Sarai, New Delhi-16 Website: winwmadeeasypublications.org | Phone: 011-45124612, 011-32059862 le = Mnoe ES (MADE EASY Publications Corporate Office: 44-A/1, Kalu Sarai, New Delhi-16; Regd. fice: 25-A, Ber Sarai, New Delhi-1 10016 ‘Webs: waww.madeeasypublications.org; Phone: 01 1-26560862, 011-32059862 E-mail: info@madeeasypublications org; iesmadeeasy@yahoo.co.in Reasoning & Aptitude Copyright © 2010, by MADE EASY Publications. Al rights are reserved. No part of his publication may be reproduced: stored in or introduced into 2 totieval system, or transmitted in any form or by any means (electronic, mechanical, photo- copying, recording or otherwise), without the prior written permission of the above mentioned publisher ofthis book. 1 utEdition: 2008 2nd Editon: 2008 ard Edition:2010 i “Typeset a: MADE EASY Publiations, New Dehi-1 10016 on oN CONTENTS Section A: Arithmetic Number System. Average Ratio & Proportion... Percentage Profit and Loss. ‘Simple Interest & Compound Interest. Time & Work, Time, Speed & Distance Section B: Algebra & Geometry Surds, Indices & Logarithm, Progression Permutation & Combination Probabilty So Geometry, Section C: Reasoning & Data Interpretation Blood Relation Coding & Decoding Cubes and Dices. Diréction Sense Test Graph. Table Bar Diagram .. Pie-chart.. Miscellaneous Puzzles, Logical Venn Diagram Analytical Reasoning 9-26 27-40 4-54 55-76 77-94 95-111 113-135 197-161 165-175 _ 177-192 193-208 209-226 227-270 273-275 277-281 283-290 291-300 301-306 307-311 313-325 327-338 339-350 361-357 359-364 ta The chapter of number system is amongst most important chapter in the whole of mathematios syllabus for the competitive Examination, ‘We can consider this chapter as the backbone of mathematics. The students are advised to go through this chapter with utmost care, under- standing each and every aspect of this topic. In this chapter basic definition of different types ‘of numbers are given. Then on the basis of this, various questions with solved examples are given for easy understanding, Number System Natural Numbers: Counting numbers 1, 2, 3, 4, 5 are known as natural numbers. The set of natural numbers can be represented bYN #11, 2.3,4, 5 orcad ‘Whole Numbers It we include 0 among the natural numibers, then the numbers 0, 1, 2, 3, 4, 6 ele are are called whole numbers. Whole number W=(0, 1, 2, 3,4, 8 nc} Clearly, every natural number is awhole number but 0 is a whole number which is not @ natural number. Number System Integers All counting numbers and their negatives including zero are known as integers. The set of integers. Ue (ous =B, 2,1, 0,1, 2,3, bad Positive integers The set (1, 2, 3, 4, 5...) is a set of all positive integers. Positive Integers and natural numbers are synonyms. Negative Integers The set [-1, -2 J Is @ set of all negative integers. 0s neither positive nor negative. Rational Numbers ‘The numbers of the form pia, where p and q are integers and q # 0are known as rational numbers. 25 4 eo: 3.7. Zete Irrational Numbers ‘Those numbers which when expressed in decimal form are neither términating nor repeating decimals, are known as irrational numbers €9.: V2, V8, V6, neto, MADE EASY * Reasoning & Aptitude | 9 MADE EASY Note that the exact value of isnot 22, 22g rational number while = is an irrational number. ‘Some times x is also considered equivalent to 385 Fig” This value is also approximate value of x and not the exact value Similarly 3.14 is not an exact value of it Real Numbers The rational and irrational numbers combined. together are called real numbers. og. Bv,4+V.6415 ete are all real numbers, The set of all real numbers are denoted by R. Coinplex Numbers Comhplex numbers can be represented in form of a +ib, where @ and b are real numbers and i= VA, Thus 3+4i, 6+2, i, 21 etc are Complex rumbers Even Numbers All those numbers which are exactly divisible by 2 ate called even numbers. eg. 2, 4, 6, 8, 10, etc. (Odd Numbers Allthose numbers which are not exactly divisible by 2 are called odd number. Eg: 1,3,5, 7, 9 etc. Prime Numbers Number divisible by 1 and itself and not divisible by any other number are called prime number. &g.:2,3,5,7, 11 ete. Note: 2s the only Prime number which is even. Rest all are odd Prime numbers. ‘Composite Numbers Natural numbers greater than 1 which are not prime are composite numbers. 29: 4,6,9, 15, etc. Co prime Numbers ‘Two numbers which have only 1 as the common factors are called Coprimes or relatively prime to each others, eg.: (3, 7) 8, 9) (86, 28) et. Test of Divisibility 1. Divisibility by 2 Anumber's divisible by 2if tre unt digits, 2010.0 dhisible by 2 eg.: 22, 42, 84, 3872 etc. 2. Divisibilty by 3 A number is divisible by 3 if the sum of digit in the number is divisible by 3. eg: 2553, Here 245 +54+3= 15, whichis divisible by hence 2588 is divisible by 3. 3. Divisibilty by 4 Annumberis divisible by 4if its last two digit isdvisible by 4, €9.: 2652, here 62 is divisible by 4 s0.2652 is divisible by 4 9.9772, 584, 904 ete 4. Divisibilty by 5 ‘A number is divisible by 5 i the units digit in number is 0 or 5. eg.: 50, 505, 405 etc. 5. Divisibility by 6 A number is divisible by 6 if the number is even and sum of digits is divisible by 3 g.: 4536 is an even number also sum of digit 44543 +6 = 18is divisible by 3. eg: 72, 8448, 3972 etc, 6. Divisibility by 8 ‘Arnumbet is divisible by 8 if last three dig of itis divisible by 8. eg.: 47472 here 472 is divisible by 8 hence this number 47472 is divisible by 8, MADE EASY » = — © Reasoning & Aptitude | 10 MADE EASY + Number System | 11 7. Divisibilty by 9 ‘A number is divisible by 9 if the sum of its ight is dvisibie by 9. €g.: 108936 here 1+0+8+9+3+6 is 27 which is divisible by 9 & hence 108936 is divisible bya 8. Divisibility by 10 ‘A number is divisible by 10 fits unit digit is 0. €9.: 90, 800, 740, 34920 etc. 9. Divisibility by 14 ‘Anumberis divisible by 11 ifthe difference ‘of sum of digit at odd places and sum of digit at even places is either 0 or divisible by 11 eg. 1931, the sum of digit at odd place is 143 and sum of digit at even places is +1 and their difference is 4-4 = 0. so 1931 is divisible by 11 HCF and'LCM of Numbers HOF (Highest Common Factor) of two or more number is the greatest number that divides each one of them exactly. For example 8 is the highest common factor of 16 and 40, HCF is also called greatest common divisior (G. C.D.)orG.C. M. i.e. Greatest Common measure. LOM. (Least Common Multiple) of two or more number is the least or a lowest number which is exactly visible by each of them. For example LCMof 8 and 12is 24, because itis. the first number which is multiple of both 8 and 12, LCM and HCF of Fractions Numerator Fractions are written in form of ———————- ect Deriominators Where denominator is not equal fo zero, (H.C.F. of Numerators) LC. Fof Fraction = {HOF of Numerators) H.C-Fol Fraction = (Cry of Dencminators) ___(LOMot Numerators) 1 Fraction = (LCM of Numerat LOM of Fraction = (CE cf Denominators) 12,43 Example: find HOF & LOM of 5.5 and cr = HOF of (12.3) _ LOM(23,7) 42 LOM of (123) _6 tom HCF of (2, 3, 7) 4 Important Algebraic Formulae 1. (a+b)? =a? +2ab+b? 2. (a-b)’ =a? -2ab+b? 3. (a-b)(a+b) =a? -b* 4, (a+bf +(a-b) =2(a? +b?) 5. (a+b)* (2b)? = dab 6. (a+b) =a9+3a%b+3ab? +b* =a +b? +3ab(a+b) 7. (a-b)° =a? ~3a°b+ Sab? -b? ° = —Sab(a-b) 8. 0°46 =(a4b)(e? -ab+b?) b? =(a—b)(a? +ab+b*) a® +b? +08 ~3abo pS tate classe 10 a? +b? +c? -ab-be~ca ( ) MADE EASY * Reasoning & Aptitude | 11 11, at ot = (22) -(02)° = (a? +b*)(a+b)(a-b) Condition of Divisibilty for Alzebraic Function 1, a+ bis exactly divisible by a+b only when nisodd Ex: 2? +9 = (a+b)(a®-ab+b*) isdivisble by atb, also a® + b® is divisible by arb 2. af + bis never divisible by a-b (whether n is odd or even) Ex.: a8 + b% = (a + b) (a? ~ ab + b?) is not ivisible by (a-b) a! +b is also not divisible by (a -b) 3. a—b"is exactly divisible by (a—_b) (whether nis odd or even) Ex.: 2° bis exactly divisible by (a—b) also a2? also exactly divisible by (a~b) Factors of Composite Number Composite numbers are the numbers which can Be Tactorised into prime factors, or simply we Gan say that composite number are those numbers which are not prime. For eg.: 8s a composite number since it can be factorised into B= 2x2x2 Similarly 9is also a composite number, ie 9=3x3 ‘Composite number = P* xP2* xP3>....P* here, P,, Pp, Py.n.P, are prime numbers and Ay, Az» 2, are their respective powers. Factors of composite number = (A, + 1) y+ tly + 1) For eg.: 18 = 2x3 x3 = 2" x3? Factors of 18 = (1+ 1)x(2+1)=2x3=68 Clearly it contains six actors 1, 2,8, 6, 9 and 18 Factors of other Composite numbers 6 = 2! x3" Factors = (1+ 1) x(1+ 1)=4= 1,2,3.and6 72 = 2x2%2x3x3 = 2 xo Factors =(3+ 1) x(2+ 1) = 12 Ex.1 Find the factors of composite number 360 Sol. 360 =2 x2 x2 x3 x3K5 3 x3 x5! Factors = (3 +1) (2+ 1)(1+ 1) = 24 Ex.2. Find all the factors of 120. Sol: 120 =2x2x2x3x5 = Bx3t x6! Factors = (8 + 1)(1+1)(141)= 16 coon Counting Number of Zeros Sometimes we come across problems in which We have to count number of zeros at the end of factorial of any number, For example Number of zero at the end of 10! 10! = 10 x9 x8 x7 x6 x5 x4 x3 x2 x1 Here basically we have to count number of ives, because multiplication a five by any even number willresuitin Oat the end of final product. In 10! we have 2 fives thus total number of zeros are 2 Short Cut - Counting number of zeros at the end of nl ala na an. Value willbe 2+ er ‘The integral value of this sum will be the total number of zeros. Ex. 1 Numer of zeros at the end of 10! 4 peter integral value is 2 Ex.2 Number of zeros at the end of 100! 100 , 100 , 100 So: “5 ete MADE EASY ‘© Reasoning & Aptitude | 12 MADE EASY integral valve will be 2044 = 24 zeros Ex:3 Number of zeros at the end of 126! Sol. integral value will be 25+5+1 = 21 zeros, Ex4 Number of zeros at the end of 500! ‘500 ,, 500 , 500, 500 5 8S integral value will be 100+ 20+ 4 = 124 zeros. Ex5 Number of zeros at the end of 1000! 1900 , 1000 , 1000 , 1000 , 1000 Se ee 200+ 40 +8 +1 = 249 zeros Sol: Ex6 Number of zeros at the end of 50! 50,50, 50 5 eS Integral value will be 10+2= 12zeros Ex.7 Number of zeros at the end of 90! 90, 90 , 90 24349. 6 8S Integral value will be 18 +3 =21 zeros Cyclicity Cyolicity of a number is used maifily for the calculation of unit digits 1. Cyclioity of 1. In 19, unit digit will always be 1 Number System | 13 2. Cyelicity of 2 222 2 After every four intervals it repeats so cycle of 2 2.4.8.6 Ex.1 Find unit digit of number 2% Sol: Here unit digit will repeat as 2, 4, 8, 6 alter every foyr interval til $2 next 63 wil bbe 2 and 54 willbe 4. So unit digit wil bed. Ex2 Unit digit of 2% Sol: Here, 4,8, 6 willrepeat after every four interval til 320 next digit will be 2, 4, So unit digit of 23 will be & Ex.3_ Find unit digit of 12"? x22% Sol: Unit digit of 12% will be 6 and 222 will be 4, So unit digit of 12° x22%will be 6x4 = 2 [@]; 4 Ans. 3. Cyolicity of 3. 6561 After every four intervals 3,9, 7 and 1 are repeated. So cycle of Sis 3,9, 7, 1 Ex.1 Find unit digit of 9% Sol: Cycleof 3is3, 9, 7, 1 which repeats after ‘every four interval til 3%, So next unit digit will be 3. MADE EASY + Reasoning & Aptitude | 13 14 | * Number System Ex2 Find unit digit of 183° Sol: Cycle of 3is3, 9,7, 1 which repeats atjse ‘every four intervals til 1332, So next unit digit wll be 3. Ex3 Find unit digit of 963% x73” Sol: —_Unitdigit of 963 willbe 7 and unit digit of 7378 will be 3. So unit digit of 9636 x73" will be 7 x3=_21 1e. 1 Ans. Ex.4 Find unit digit of 122% x 13349 Sol: Unit digit of 12222 will be 4 because cycle of 2 is 2, 4, 8, 6 and unit digit of 133° will be 3. So unit digit of 1221% x 133'9 willbe 4 x3 = 12i.e.2 Ans, 4. Cyolicity of 4 aed, 2216 ®=64 4 = 258 Cycle is 4, 6, ve Unit digit of 4" depends on value of n. Itnis odd unit digit is 4 and itn is even digit is6. Ext Find unit digit of 4825 Sol.: Since 425 is odd number unit digit will bed. Ex2 Find unit digit of 412, Sol: Since 1024s even number unit digit wll bes. Ex. Find unit digit of 133° x4, Sol: Unit digit of 133% is 7 and unit digit of 4°95 4:50 unit digit of 133% x4 will be 7x4= 2812.8, 5. Cyclicity of 5. 5125 = 25 582125 54 = 625, Uni igt will always be 5. MADE EASY 6. Cyclicity of 6. Unit digit will always be 6. Ex.1 Find unit digit of 4® x6 Sol: Unitdigitot 4% 4 andunit digit ofstis | 6 so unit digit of 4® x65 will be 4 x6 = 24 ies Ex.2 Find unit digit of 14445 x 1262, Sol: Unit digit of 1445 is 4 and unit digit of 126s 6, So unit digit of 144° x 126 7. Cyclicity of 7. TMs7 7249 P2343 T= 2401 75= 16807 = 117649 7? = 823543 78= 5764801 Cycle of Zs 7,9, 3, 1 Ex.1_ Find unit digit of 1717 Sol: Cycle of 7 repeats 7,9, 3,1 til 17*next digit will be 7. So ans. is 7. Ex2 Find unitdigit of 1717x277 Sol.: Unit digit of 1717 is 7 and unit digit of 27°" ig 3. So unit digit of 17°” 272” wll be7x3=2t1e.1 x3 Find unit digit of 177 x272” x37? Sol.: Unit digit of 17°7 is 7, unit digit of 2727 is 3 and unit digit of 37° is 7. So unt digit of 17°7 x 2727 x37 will be 7 x3 x. 147 ie. 7. 8. Cyclicity of 8. a=8 = 64 MADE EASY. + Reasoning & Aptitude | 14 ‘MADE EASY. + Number System | 15 Ex. Ex. 8 = 4096 8° = 92768 So cycle of Bis 8, 4, 2, 6 1. Find unit digit of 18°, Cycle of 8 repeats 8, 4, 26 after every four intervals til 18°° next digits will be 8 and 4. So unit digit of 18 will be 4 2 Find unit digit of 181° x26? x 28822, Unit digit of 1818s 4, unit digit of 262 is 6, unit digit of 28828 is 6. So unit digit of 18°° x28% x2682% willbe 4 x6 x6 = 144 led, Cyclicity of 3 g=9 # 9 = 6561 Cycle of 9 is 9, 1 : {In 9° unit digit will be 9 if n is odd and unit digit will be 1 ifn is even. 1 Find unit digit of 1114 1224 1399+ 14144 1518 Unit digit of 11" is 1 Unit digit of 12%is 6 Unit digit of 13% js 3 Unit digi of 147 is 6 Unit digit of 15% is 5 So unit digit of given sum will be 14643+645= 2110.1 2 Find unit digit of 2171 222? x23 x 24% x 255 25% will give 5 in unit place, when rmutipied by any even number ie. 0,2, 4, 6,8 it will give zero at unit place. So, zero willbe the unit digit of given question. Cyclicity table ia 22, 4,6, 9, 7,1 6 2 3 4 5 25 Remainder of expression Remainder Theorem axbxe nm [le.axb xe when divided by n] is equal to the remainder of expression S2*PaCn SaxBoXOn ig, a, xb, xc, when a divided by n], where Ex. Sol x. a, 8 remainder when a is divided by n, by; is remainder when b is divided by n, and «6, i8 remainder when c is divided by n. 1. Find the remainder of 15 x 17 x19 when divided by 7. 17 Rona stoxpesson 2171 yy 1x3x5 be equal to *F BL 7 jet On dividing 15 by 7 we get 1 as remainder On dividing 17 by 7 we get 3 as remainder On dividing 19 by 7 we get § as remainder and combined remainder will be equal to remainder of 2 jet 2. Findremainder of expression 12*20*2t will be equal to remainder of expression 12x 9 which is equal to 6. MADE EASY ‘* Reasoning & Aptitude | 15 16 | | * Number System MADE EASY {)Polynomial Theorem Ex4 hiss very powerful theorem to find the remainder. According to polynomial theorem. (e+ ay =a" + nC, x4 4 nC, a? 9? Sol +nO,x"8 a8... ,xta™! +a (1) _ era) x" nC; ACs x9 +. nf (2) Ex remainder of expression (2) will be equal 0 Sol: remainder of because rest of the term contains x are completely divisible by x Ex. Find remainder of Ex.1 Find the remainder of 5 rax7+ay _ (4) sa; &, C+" : oe AT - 8 500 0 According to polynomial theorem 2 og AB x2_ 4 9 x2 remainder will be equal to remainder of 7 7 7 7 = Remainder is 2 8 ExT Findremainder of &. 3° Ex2_ Find remainder of — 0 Sra ops 0 7 sa: SF we oe TOOT 7 7 (7a) po st: So Pie, 22 _ (74x? 7 7 7 = Cx? ie 7 7 tatont? = Remainder is 2 Ex.3 Find remainder of == inder of 2 x8. Findremainder of =, so; UxSxI7 _ Sew ty 6 ¢ 76 so, OD, wears According to remainder theorem} ST PF MADE EASY * Reasoning & Aptitude | 16 MADE EASY Case-1 On Givi (2x2? xd One? og x4 7 7 = Remainder is 4. More On Remainders iding a number by a, b & c it we get ak, 'o-k and o-k as remainder respectively then that umber will be n x LCM of fa, b, cK. . For ex (1): On dividing a number by 4, 5 & 6 we get 3, 4, & 5 as remainder. Find the number. Sol.: 4 5 6 Remainder a 4 8 which is equalto (4— 1), (5~ 1), (6-1), 80 that number will be: nx LCM of (4,5, 6)~ 1, ie, 60-1= 58 Note: n such numbers are possible. Here we have taken n as 1. Othér numbers are 119, 179, 239, etc. Where value of nis 2, 3, & 4 respectively. Ex2 (On dividing a number by 5,6 and 7 we got 3, 4 and 5 a5 remainder, Find the number. * Number System | 17 We get 18, 38, 88,78, 98, 118 when value of nis 1, 2, 3, 4, 8 and 6 respectively Highest double digit such number wil be 98 Ans. Ex4 On dividing a number by 4, 5 and 6 we get 2,3 and 4 as remainder find highest possible three digit such number. Sol. 4 8&8 6 Remainder 2 3 4 which is equal t0 (4-2), (6 =2). (6 - 2), that number will be n xLCM of [4, 5, 8-2 = x60-2 When n = 1 we get 88. Highest possible three digit such number willbe 958. Ex5 On dividing a number by 5, 6 and 7 we get 8, 4 and 5 as remainder. Find highest possible three digit such number. Sol. - . 5 6&8 7 Remainder 3 4 s which is equal to(5-2), (6-2), (7 ~ 2) thatnumber will be: n#LOM(G, 6, 7}:2=n x210-2 Highest possible three digit number wil Sol: 5 6 7 be 838. Remainder 3 4 8 a +H whichis equalto (5-2), (6-2), 7-2) Caserl thatrumber vil be On dividing a number 2, b and c if we get k as neLOMol(s,6.7)-2= 210-2 208, "mainder always, then that number wil be ee + LOM of + Note: Here we have taken value of nas 1 neLeMol(a,b. otk Ex.1_ On dividing a number by §, 6 and 7 if we £3 Ondividing a number by 4 and § we are gelting 2 & 3 as remainders, Find the et2 as remainder aivay, Ing thatrnumber highest double digit such a number. Sol: That number will be Sol nxLOM of (5,6, 7}+ 2 4 8 2210422212 Boman to 39 Ex2__ On dividing a number by 6 and 7 we get wich seaualio 2) C2) 4 as remainder always, find the highest petoMer ta 5) 2 possible three digit such number. MADE EASY * Reasoning & Aptitude | 17 nLOM[6,7]+4 =n x42 + 4; three digit highest possible such number will be 966 + 4 = 970 Ans. Case-Ill itanumber atter adding kis exactly divisible by .@, bande then that number willbe. nxLCM (a, b, chk Ex.1 Find a number which after adding § is Givisible by 6 and 7. Sol: That number will be ni xLOM of (6, 7)-5 ifn = 1 then. 42-5 = 97. . x2. Find a number which after adding 7 is divisible by 10, 11 and 12. Sol: That number will be nxLCM of [10, 11, 12]-7 itn= 1 then 660 - 7 = 653 Ans. nooo Squares of Numbers Squares of numbers are frequently used for calculations on various types of problems. It is advisable to remember square of at least first thirty numbers, From following table we come to know that square of anumber always ends with 0, 1, 4, 5,689 as unit digit Square of a number can neverhave2,3,7 & 8 in its unit place. On observing squares of numbers between 21 to 2 we get following pattern. ar =4lai] 29" = oat 22'= 484) 28" = 8/84 23°=5|29| 27" = 7)29) 24? = 5|76) 26" = 6176) 25% = 6125] Last two digits are common, Shortcuts for calculation of squares of numbers between 41 to 50. Ex.1 Find square of 48? eo bawh Bie ‘Square of -2 is equal to 04 so wé ave. written 04 in last two places. Square of 5 is 28, from which we added =2 and get 28, So square of 48 is 2804. Ex2 442= Sol: 44/s~6 distance away from 50 te + 2 F225 25 % 1986 Ex3 422-7 Sol: 42is-8 distance away from 50 50 -8 + + =25 os e 1764 x4 2 Sol. ~9 distance away from 50 MADE EASY * Reasoning & Aptitude | 18 * Number System | 19 MADE EASY 50 2 Ex.2 (19? i 7 18 oem BS si ‘ea 1 x(1 + 2)25 = 205% 1681 Ans Sol: Similarly, we can find squares of other square ‘of number between 50 & 60. x3 45°=? Ext 63=2 Sol: 53's +3 distance away from 50 Sol: [4x(4+ 1)]28 = 2025 0 “3 Exa 65-7 Sol: [6x(6+ 1)]25 = 4225 ves $s 6x@+ 0 a x5 e5%=? Sol: [8 x(B + 1)]25=> 7205 x2 562=? ; Sol: 56is +6 distance away from 50 Ex6 95° =? # 5 Sol: [9 x(9 + 1)]25 > 9025 ve th Some Multiplication Techniques 36 Muliplication usualy tekes longer time than Ex3 58° Sol: 58s +8 distance away from 50 50 o seas § & ea Ex4 59% =? Sol: 891s +9 distance away from 50 0 8 + + 8=25 25 8 387 Similarly, we can find square of other numbers between 50 and 60 speedily. Observation. ‘Square of two digit number having 5 in unit places can be calculated very easily nS heren may 1 t0 9. (n5)?= [n+ (n+ 1)}25 addition and subtraction. While solving problems related to ratio and proportion and percentage we have to multiply numbers in and around 100 frequently. Here, we have some tricks fo make it avery enjoyable way to calculate the number. Case-1 When both numbers are greater than 100. Ext 106 x 103 =? Sol: 106 is + 6 distance away from 100 and 109 is + 3 distance away from 100. 1 8 137 Ns i |B $0 106 x 103 willbe equal to 10918. Here either we are adding 106 & 3 or we are adding 103 & 6, in both case we will get 108 Ex2 104 x 105=7 Sol: tos +4 ers 10820 = 10920 Ans. MADE EASY © Reasoning & Aptitude | 19 zu | © wumper system Ex3 108 x11 Sol: 103 m 19 = 11988 Ans, | Exa et aan 105 7% 5, 2647 08 = 12705 Note: Multiplication of 24 and 5 will be 108, we will write 8 infirst two place and 1 will become carry which will be added to (105 + 21) and finally we get 12708 as answer. x6 118x118=7 tev. 8, 138) = 19924 Ans, Ex.6 117x177=? eae eae = 13689 Ans. ber 118x106? 15 415: 1037 ™ +08 124120 = 12420 Ans. Ex.8 125x103 Sol: 128. 425 eae = 12875 Ans. Ex9 111x107 =? meat 1077 47 1877 = 11877 Ans, Ex10 1131122? Sol: 119. +19 127 a2 125182 = 12652 Ans. Case-t When both numbers are less than 100. Ext 92x93=2 Sol.: 92is-8 distance away from 100, and 93 is-7 distance away from 100, thus 24 wt ae = 8556 Ans, Ex.2 88x95 =7 Sol: @8is—12 distance away from 100, and 95 is*5 distanoe away from 100, thus ‘ 2 95 33 = 6360 Ans, Note: Here we are adding 95 and -12 or 88 and -5. xa" 99x87 «7 Sol: 93 a 9 oo 001 An MADEEASY ‘* Reasoning & Aptitude | 20 MADE EASY x4 89x88 =7 Sol: 99 88 78 Note: Here we get 132'by multiplying -11 and -12, where 1 is carry. Wehave to add 1 with 88 and 11 of 89 and ~12 and we get desired result as 7892. ExS 97 x94=? Sol: 97 a wes oF 18 = 9118 Ans. x6 85x98 =? Sol: 35 « ® 88 . =.8330 Ans, ExT 94x98 =7 Sol: 94 6 a5 % 2 = 9212 Ans, ExB 82 x82=7 Sol: 8 18 82 = 87 alee = 6734 Ans, [Noté: Here we have added 3 while adding 82 & -18 & get 6734) Case-IIl When one number is gfeater than 100 and another number is less than 100. Ex.1 105 x93 =? Sol: 105 is +5 distance away from and 93 is 7 distance away trom 100. MADE EASY Ex2 ‘+ Number System | 21 Here we have to substract 35 trom 9800 and tinal result will be, 9765. wt att x 8 9900 132 9900 132 9768 Here we have to substract 192 from9900 and final result will be 9768. oooo | Solved Examples |. The last digit of the number obtained by multiplying the numbers 81 x82 x83 x84 x 85 x86 x87 x88 x89 will be f@) 0 (b) 9 7 @)2 Ansa) Last digit will be mutiptication of 1x23 x4 x5 x6 x7 x8 x9 Since 5 and 2 are given here, their ‘multiplication will result in zero as last digit, The sum of the digits of a two-digit number Is 10, while when the digits are reversed, the number decreases by 54. Find the the ‘changed number. (a) 2B (b) 19 (©). 97 (0) 46 Ans: (a) Going through options we get 82-28 = 54 * Reasoning & Aptitude | 21 22 | * Number System MADEEASY 3. The sum of two numbers is 15 and their geometric mean is 20% lower than thelr arithmetic mean. Find the numbers. @ 114 () 12,3 ©) 13,2 @ 10,5 ‘Ans: (b) Going through options only 12 and 3 satisfices the conditon 1243 AM= Se 78 GM=Ji2x3=6 which is 20% less than 75. 4, 11 A381 is divisible by 11, find the value of the smallest natural number A? @s (0) 6 7 @e Ans. (c) ‘A381 is divisible by 11 if and only if (A+ 8) = (3 + 1) is divisible by 11. So, A=7 Satisties the condition 5. If 381A is divisible by 9, find the value of ‘smallest natural number A? (5 () 6 7 © 9 Ans: (b) 381 Ais divisible by 9 So 3+8+1+A=12+A is divisible by 9 SoAz6 6. Find the LOM of 5/2, 8/9, 11/14, (@) 280 (b) 960 (©) 420 (2) None of these Ans: (d) LoM of fraction = LOM of numerators HC.Fof Denominators Here, 5/2, 8/9, 11/14, so LCMof (5, 8, 11) _ 440 HOF of (2, 9, 14) Lom: 7. A number 158 is divisible by 6. Which of these willbe true about the positive integer 8? (@) B will be even (©) B will be odd (6) Bwill be divisible by 6 (@) Both aandc ‘Ans: (d) 15 Bis divisible by 6 0B should be even number also 14548 should be divisible by 8 only 6 is the value for 8 8. Find the number of zeros at the end of 100! (a) 24 (b) 28 (©) 26 (2) None of these ‘Ans: (a) 1001 100 , 100 , 100 ae No. of Zeros = +4 (Whole number of) = 20 + 4 = 24 9. If 146! is divisible by 5°, then ‘find the maximum value of n (a) 94 (b) 35 (©) 36 * (0) 37 Ans: (b) 146 “pr then highest value of 5 will be 146 146 146 eg gr 48+ 10. Find the number of divisors of 1420. (a) 14 (b) 15 (c) 13 (a) 12 Ans: (d) 1420 = 142 x10 = 71 x2 x2 x5 = 2? x5! x71" No. of divisor = (241) (1+1) (141) = 12 11. Find the HCF and LCM of the polynomials (x? -5x+6) and (x? -7x+10), MADE EASY ‘* Reasoning & Aptitude | 22 MADE EASY = Number System | 23 (8) (1-2, e-2)64-3)(x-8) (©) (X-2).x-2)(x-3) (©) (-9h(K-2)(x-9)(x-9) (0) None of these Ans: (a) x2 5x46 = (x-9)(x-2) 22-7410 = (x-8)(x-2) So, HCF = (x-2) LOM = (x- 2) (x3) (x-5) Directions for questions 12-14: different prime numbers P and Q. Find the number of divisors of the following: 12.P.0. @2 (©) 4 6 (a8 ‘Ans:(b) PQ=P!xa! 35 Number of divisor = (1 + 1) (14 1)=4 13. P2Q ° @2 4 os 8 * Ans: (¢) PQ Number of divisor = (2+1) (141) = 4 14, Page @2 () 4 6 (@) 12 Ans: (d) sg? Number of divisors = (3+1) (2+1) = 12 45. A mikman has three different qualities of milk, 408 gallons of tst quality, 465 gallons ‘of 2nd quality and 496 gallons of 3rd quality, Find the least possible numer of bottles of equal size in which different milk of diferent Qualities can be filed without mixing? @) 4 (o) 46 (©) % @ 44 Ans: (d) itis given that gallons of ‘st quality : 403, lind quality : 465 lira quality : 496 least number of gallons willbe in size of 403, 465 and 496 403 = 18 x31 465 = 15 x31 496 HOF = 31 so we required 19415+16 gallons 16. What is the greatest number of 4 digits that when divided by any of the numbers 6, 9, 12, 17 leaves a remainder of 1? (@) 9997 (b) 9793, (c) 9895 (d) 9487, Ans: (b) LOM of 6, 9, 12, 17 = 612 greatest number of 4 digit divisible by 612 is 9792, to get remainder 1 number should beg7a2e1 17. Which of the following is not a perfect square? (@) 100858 (b) 3, 25, 137 (©) 945728 @ Allof these Ans: (d) ‘Square of number never ends up with 2, 3, 7.8 18, Which of the following can never be in the ending of a perfect square? @6é (o) 00 (©) 000 5 Ans: (c) 19, The LCMof 5, 8, 12, 20wilinot be a multiple of @3 9 ) 8 @s ‘Ans: (b) LCMot 5, 8, 12, 20 will not be a multiple of 9 MADE EASY © Reasoning & Aptitude | 23 24 |» Number System MADE EASY 20. The LOM ot (16—»2) and (x2+x—-6) is (2) (xe 3Ylx+3\'4—H)(D) (4 —F)ox49) (©) eNx-3)— (@) None of these Ans: (4) 16-x? =(4-x)(4+x) (x@+x-8)= (x+3)(x-2) LoM wit (16-x*){x? +x-6) 21. GCD of x24 and 2x-6is (a) x42 (0) x-2 (©) #-2 (0) 242 Ans; (b) Pha (x-2)(K42) (x +x~6)=(x+3)(x-2) GOD =(x~2) 22. Decompose the number 20 into two terms: such that thelr product is the greatest. @) x (b) =8, x015 (d) none of these here, x,+%220 Xeno 10 28. Which of the following can be a number divisible by 247 (@) 4,98, 15, 604 (©) 13,62, 480 Ans: (¢) ae Anumber divisible by 24 will be divisible by Bands Only 1362480 satisfies the divsibilty criteria (&) 25, 61, 284 (d) None of these 24, For a number to be divisible by 88, it should be (@) Divisible by 22 and 8 (0) Divisible by 11 and 8 (©) Divisible by 11 and thrice by 2 (A) Both band ‘Ans: (b) A number to be divisible by 88 It should be divisible by 8 and 11 because 8 and 11 are co prime numbers whose multiplication gives 88. 285. Find the GCD of the polynomials (x+3)? (x-2) («+ 1)? and (x+1)8 (43) (X44). () (x43) (x41)? (x2) 44) (©) (443) (¥-2) (x41) (ce) (©) (x43) (x44 (@) None of these ‘Ans: (0) GCD of (x+3)(x=2)(x+ i and (x+ HP (x+3)(x+4) will be (x43) (x +1)? 26, Find the LCM of (x+3) (6x2+5x-4) and (2x2+ Tx +3) (x +3) (@) (2x + 1) (x +3) (3x +4) (b) (42-1) (x + 3)? Gx + 4) (©) (A 1) +3) Ox+4) (B) (2x 1) (x + 3) (Bx + 4) Ans: (b) (#3)(6x2 + 9x-4) =(c+3)(2x=1)(Bx +4) (2x? +7x-+3)(x+3) = (2x+1)(x+3)(x+3) LOM= (2x+1)(2x—1)(x+3)° (ax+4) = (4x2 = 1) +9)" (3x44) 27. The product of three consecutive natural ‘numbers, the frst of which isan even number, is always divisible by (@) 12 (b) 24 @s (A) Allof these ‘Ans: () Three consecutive number will be n(n+1) (042) f nis even number then (n+2) wil also bbe an even number and one of them wil be divisible by 3. Hence number is always divisible by 12. for example ifn=2then 2.3.4 = 24ifn = 4 then 4. 5.6 = 120 MADE EASY * Reasoning & Aptitude | 24 MADE EASY 28. Find the pairs of natural numbers whose least ‘common multiple is 78 and the greatest common divisor is 13, (@) 58and 13 or 16 and 29, (©) 98 and 23 or 36 and 49 (©) 18and 73 or 56 and 93 (d) 78 and 13 of 26 and 39 Ans: (d) LCM = 78 and GCD = 13 Clearly 13, 78 and 26, 39 are the two numbers 29. Find two natural numbers whose sum is 85 and the least common multiple is 102. (@) 30.and 55 (0) 17 and 68 (©) 35.and 65 (@) Stand 34 Ans: (d) Sum x+y=85 LOM = 102 only numbers are 51 and 34 30. What digits should be put in place of ¢ in 38¢ to make it divisible by ‘© Number System | 25 Cy) Solved Examples Fill in the blank indicated by a star in the number 4* 56 so as to make it divisible by 33, @3 (o) 4 © 5 (@) None of these Ans. (a) 4°56 is divisible by 93 if and only if it is divisible by 3 and 11 4°56 willbe divisible by 3 if * wil be equal 100,3,6,9 4°56 is divisible by 11 if (4.4 5)~(* +6) will be divisible by 11 50 * should be 3 . Find the least number which being divided by 9, 12, 16 and 90 leaves in each case a remainder 3? (@) 623 (0) 723 (©) 728 (0) None of these Ans. (b) LCM of 9, 12, 16 and 30 is 720 so required number is LOM+3 = 723 . Find the number which being increased by 1 willbe exactly divisible by 17, 22, 33 and 102 (@) 1131 (o) 1121 (©) 1001 {(d) None of these Ans. (b) LCM of 17, 22, 93 and 102is 1122 So the given number is 1122 ~ 1=1121 Find the greatest number less than 10000 which is divisible by 48, 60 and 64 (@) 9600 (®) 8500 (©) 7600 (@) none of these Ans. (a) The required number will be the largest four digit number in form of n*(LCM) of 48, 60 and 64 LCM of 48, 60 and 64 is 960 ‘So the largest four digit number willbe 9600 3. Find the least multiple of 11 which when divided by 8, 9, 12, 14 leaves 4as remainder in each case. me @3 @4 (5 6 @s @ 10 ‘Ans: 38¢ (1) + to make it dvisibie by 2, ¢ should be 0,2,4,6,8 (2) + tomake it divisible by 3, ¢ should be 147 () + tomake it divisible by 4, ¢ should be 0.48 (4) + to make it divisible by 5, ¢ should be 0.5 (5) + to make it divisible by 6, ¢ should be 4 (6) >to make it divisible by 9, ¢ should be 7 (7) = to make it civisible by 10, ¢ should beo oooo MADE EASY * Reasoning & Aptitude | 25 26 | * Number System MADEEASY =| h (@) 1012 (b) 1037 after which I have Rs. 648 left over. What is | (©) 1080 (2) none of these my income? Ans. (a) (@) Rs. 1200 (0) Rs. 1400 The number is divisible by 11 and can be (©) Rs. 1700 (6) none of these written in form n(LOM) +4, LOM of 8, 9, 12, 14is 504 So the number may be 508 & 1012 but 508 is not divisible by 11 so itis 1012 6. Find the greatest number of four digit which when divided by 15, 20, 28 leaves in each case a remainder 2? (@) 9662 (v) 9077 (©) 1090 (@) none of these Ans. (a) The required number will be (n*LCM) +2 LOM of 15, 20, 28's 420 So greatest number divisible by 420is 9660 (Four digit number) So required number will be 9662 7. Find the two numbers of 3 digits each of whose GCF is 310 and LCMis 1860 (a) 620,930 (b) 240,540 (©) 720, 1080 (6) none of these Ans. (a) Going by options we can determine tht only option (a) satisties the given condition, 8. TheLCMoftwonumberis 12times their HCF. The sum of HCF and LCM is 403. If one umber is 93 find the other. (@) 134 (b) 124 (© 128 (d) none of these Ans. (b) Itis given that LOM=12 times HOF also LCM +HCF = 403, So, 13 HCF = 403, => HCF = 31 LOM = 372 also we know that HOF HCF xLCM= Number({) x Number(2) 81x372=93xN2 2. N2=124 9. Ihave to spend 1/10 of my income on house rent, 1/10 of remainder on conveyance 1/3 of further remainder on children's education 10. Ans. (a) One alternate method Let | have x: rapes AMter spending =f ofitonhouserentIhave—) sot itie., Now out ot 2 FE spent io" 9 705% O" conveyance so remainder willbe 99 8% 40" 700" ~ 100 1, BK | tx ‘ Fi 3 of ie urther | spent = of 55 he. Tq into ss chides edvaton non nee Sf ‘A man had two sons, To the elder he gave 77 of his property, to the younger Pot remainder, the rest to the widow. Find the Share of the sons if the widow gets Rs. 9600. _ (@) Rs. 1200, 1000 (b) Rs. 6000, 2000 () Rs. 7500, 1000 (d) none of these Ans. (d) Younger son gets 1), 5 1 Elder son gets 3600 x ( sa oooo MADE EASY ‘* Reasoning & Aptitude | 26 | | | | | I | | t cr ‘The average of a number is a measure of central tendency of a set of Numbers. In other words, it is an estimate of where the center point of a set ‘of number lies. Average = SUT! Sot oN number ‘ N This also means ‘Average xN = sim of set of numbers Iiystration: Let suppose the seore of sachin Tendular in last § matches are 90, 150, 10, 70, 45, then average score willbe 90+150+10+70+45 _ 965 5 5 Av Concept of Weighted Average When we have two or more groups whose individual averages are known, then to find the combined average ofthe all elements of al the groups we use weighted average. Thus, we have K groups with Averages A,, A... A and having yy My nn Ry elements then the weighted average 08 given by formula: fay DALE ey tO te es +0 Illustration: Let Rahuls average score in 5 test of maths is 75, 4 test of physics is 80 and 3 test of Average chemistry is 90 the find the averall average marks obtained by him in these subjects. Here we have to calculate weighted average which can be calculated by 75x5+80x4+90x3 _ 965 54443 12 = 80.4 approx Sometimes weighted average is used to find merit ‘order of students in competition examinations Like for examples if ina Pre Engineering Test (PET) sum of scores obtained by two students are equal then their weighted average is calculated toting order of ranking ' Illustration: Rahul & Revi Scored following marks in PET exam, Subject [Rahul [Revi [Maths [90 [60 Physics [70 _ [60 Chemistry [60_ [60 In this case total scores are equal for Rahul and Ravi, To find ranking weights can be given in following manner. Maths 3, Physics 2, and Chemistry 1 in this case weighted average can be calculated as A (Rahul 2 MADE EASY * Reasoning & Aptitude | 27 28 | * Average MADE EASY 7. The average of first ten odd prime number = Durex is soxgueox2+80%1 345474114194174 19423420431 A,(Ravi) = eae 10 440 - 12-7333 6 Here weighted average of Rahul is more than weighted average of Ravi, so Rahul will be goon preferred in ranking Some Important Results Cy Solved Example 4. Average of first ton natural Number is 142484 rn 85 1. The average age of 24 students and the 10 10 principalis 15 years. When the principalsage 2. Average of first ten whole Number is. is excluded, the average age decreases by 1 year. What is the age of principal? OFF 2434 45 gg @ 38 (b) 40 0 0 (©) 99 (d) Data inadequate 3. The average of first ten even number is. Ans. (c) 2446s M04, Average age of 24 students - 10 10 _ sum ofages of 24 student’ 4. The average of first 10 odd number is, 26 14345+ 19 _ 100 _49 14 = Sumot ages of 24 students 10 10 ates 24 5. Average of first ten prime number is ‘2 Sum of ages of 24 students = 24% 14 2434547411413 417419423 +29 = 336 years 10 Let the Age of principal be x year 129 Then, average age of 24 students and the -Ra129 principal 6. The average of first ten composite number sue is: A464+84+9410412+14+ 154 16+18 1g = 364 10 2 12 yp 39x = 25x15-336 = 39 years -a MADE EASY ‘* Reasoning & Aptitude | 28 te MADE EASY + Average | 29 2, The average weight of 3 men A, 8 and C is £84kg, Another man D joins the group and the average now becomes 80 kg. If another man E, whose weight is 8 kg more than that of D, replaces A then the average weight of 8,C,D ‘and E becomes 78 kg. The weight of Ais (@) 70kg. (©) 72g, (6) 79kg (@) 78kg Ans. (c) ‘Sum of weight of A.B and C = 3x84 = 252kg sum of weights of A.B, C and D = 4°80 =320 kg weight of D = 320-252 = 68 kg, Weight of E = 68 +3 = 71 kg Now, sum of weight of A,B, C, D and 20 + 71 = 391 kg sum of weight of B,C, D and E = 4x78=312kg * ~, weight of A = 991 - 312 = 79 kg 3. Three years ago, the average age of A, B and C was 27 years and that of B and C, 5 years ago was 20 years. As present age is, (@) S0years (b) 35 years ©) 40years (0) 48 years Ans. (0) The sum of ages of A,B and C, there years ago = 27 x3 = 81 years Present sum of ages of A,B, and C =81+3.x3 = 90 years ‘sum of ages of B and C, 5 years ago = 20x2 = 40 yoars Present sum of ages of B,and C =40+2x5 = 50 years As Present age = 90-50 = 40 years 4, Vijay Tendulkar has certain average for 9 innings. In the tenth inning, he scores 100 runs thereby increasing his average by 8 runs. His new average is ‘Ans. (c) Let the old average and new average by x and y respectively Then, y=x+8 (i) (ii) From (and (i) we get 9x + 100 The average of the first five multiples of 7is (@) 2 () 21 (©) 2 (@) 9 Ans. (b) First five multiples of seven are 7, 14,21, 28, 35 average of first five multiples of 7 = HIE2#9¢499). . The average age of a family of 6 members is 22 years. If the age of the youngest member be 7 years, what was the average age of the family atthe birth of the youngest member? (@) 1 (0) 18 © 21 (@ 12 Ans. (b) Present sum of ages of family member = 6 x22 = 192 years sum of ages of family member, 7 years ago = 192-6 x7 = 90 years average age of the family, at the birth of youngest member 90 = Bose 3 7 18 years The average age of 8 persons in a committee is increased by 2 years when twomen aged 35 years and 45 years are substituted by two women. Find the average age othe two women. @ 2 (bo) 26 @) @ (©) 45 ©) 2 (a) 92 @ 5 (O) 42 MADE EASY '* Reasoning & Aptitude | 29 30 |» Average MALE EAST Ans. (a) Let the average age of 8 persons in the ‘committee be x years LLetthe sum of ages of two women be y years. ‘Then sum of ages of 6 persons excluding two men aged 35 and 45 years = x= (35 + 45) = Bx— 80 8x-80+y A gain, x +2 3 = 8K+16=8x-B0+y ay=96 <, average age of two women 96 = Be ag 5 = 48 years 8. The speed of the rain in going from Nagpur to Allahabad is 100 kmh while when coming back from Allahabad to Nagpur, its speed is 1180 km. Find the average speed during the ‘whole journey. (@) 125 (&) 75 (0) 135 (@) 120 ‘Ans. (d) Let the distance between Allahabad and Nagpur be x km. : The average speed 100 2x 100x150 2x100%180 t 250 120 kin. 9. The average’ weight ofa class of 29 students is 40 kg. If the weight of the teacher be included, the average rises by 500 gm. What is the weight of the teacher? (@) 40.5kg, (b) 50.5kg (©) 45kg (@) 55kg Ans: (d) ‘Sum of weight of 29 students = 40 x29 = 1160 kg 10. 1. 12 Let the weight ofthe teacher be xg, then 1160+ x 30 | = x= 40.5x30-1160 = 1215-1160 40+0. p i : | The average of 8 numbers is 17 and that of } the first two is 16, Find the third number \ @ 6 (&) 16 | © 6 @ 19 Ans. (d) ; Sum of 8 number = 3x17 = 51 sum of the frst two members= 2 x16 = 32 | . The third member = 51-32 = 19 ‘The age of Mitali and Poojais in the ratio 2:6 t Alter 5 years, the ratio of their ages wil b become 6 : 8. Find the average of their ages after 10 years. t (@) 12 ) 13 © 7 (6) 24 t Ans. (a) i Letthe present age of Mitali and Pooja be 2x and 6x years respectively. 2x+5 6 Alter Sypars, SE = 5 = 16r+40=36x+30 = 20r=10 1 , xed - Hence Present ages are 1 and 3 years After ten years, their ages will be 11 and 13 years. 2. Average of their ages 1418 10 oars 4 2 Find the average of the first 97 natural numbers. (@ 47 ©) 4 (b) 37 @ MADE EASY * Reasoning & Aptitude | 30 MADE EASY * Average | 31 Ans. (d) Ans. (d) ‘Sum ofthe frst 97 natural members ‘Sum of 50 numbers _s1er+ = 98 x50 = 1900 19. Find the average of all prime numbers between 30 and 60. (a) 39.8 (b) 38.8 () 378 (0) 41.8 Ans. (a) ‘The prime number between 30 and 50 are 31,7, 41, 43, 47 Their average 81497 441443+47 5 14, The average of § consecutive number isn. If the next two numbers are aldo included, the average wil (@) increase by1 (6) remainthe same ©) increase’by 1.4 (d) increase by 2 ‘Ans. (a) Letthe five consecutive numbers of x, x + 1, xe QxtS andre 4, 9.8 xextltrtQtreSext4 Given, ven, z Sonex? The average after including the next two numbers CEAEH Hert 2teeBextderySox46 a a nl ex43=n-243 +4 Henoe the average increases by 1 15, The average of 50 number is 38. If two numbers, namely, 45 and 55 are discarded, the average of the remaining numbers is @ 365 (©) 97 © 376 @ 975 ‘Sum of remaining numbers = 1900 — (45+85) = 1800 average of remaining numbers i -a75 16. Average marks obtained by a student in 3 papers is 52 and in the fourth paper he obtains 60 marks. Find his new average. @) 54 (0) 82 (0) 85 (@) 535 “Ans. (a) The new average = SXS2#80 . 54 17. The average weight of 5 men is decreased by Skg when one of them weighing 150 kg is replaced by another person. Find the weight of the new person? (a) 165 (0) 195 (0) 198 (@) 162 ‘Ans. (b) Let the average weight of § men be x kg before replacement of one person, Also, Let the weight of new person be y kg. thon, x-8= 2a 1504Y > Sr-18= 52-1504 => y= 195 kg 18. The average age of a group of men is increased by 5 years when aperson aged 18 years is replaced bya new person of age 38 ‘years, How many men are there inthe group? (@ 3 (b) 4 ©s 6 Ans. (b) Let there be mmen in the group and the average age of the group be x years before replacement. MADE EASY © Reasoning & Aptitude | 31 Ten x45 = = 8 => ne+ Sn =nx+ 20 = ned 19. The average age of the Indian cricket team playing the Nagpur test is 30. The average age of Sol the playersis 27 and that of another set of 5 players, totally different from the frst five, is 29. If its the captain who was not included in either of these two groups, then find the age of the captain. (a) 75 (b) 55 () 50 (a) Cannot be détermined Ans. (c) Let the age of the captain be x years Then, 30= BxS24 29% > 890 = 185 +x 4145 x = 330 ~ 280 = 50 years 20. A bus goes to Ranchi from Patna at the rate cof 60 km per hour. Another bus leaves Ranchi for Patna at the same time as the first bus at the rate of 70 km per hour. Find the average speed for the journeys of the tio buses combined ifit is known that the distance from ranchi to Patna is 420 kilometers. (@) 64615kmph —_(b) 64.5 kmph (c) 63.823kmph (@) 64.82kmph Ans. (a) ‘The average speed 2x420_ _ 2x60x70 _ 840 20, 0-130 13 60 70 = 64,615 kmph 21. Out of three numbers, the first is twice the second and three times the third. The average 23. of the three numbers is 88. The smallest number is (@ 72 (0) %6 @2 @) @ ‘Ans.(d) Let the first number be n. then the other two noon numbers will be > and ' 4 «The smalest number = 2 “= ag 9. The average weight of 6 personsis increased by 2°5 kg when one of them whose weight is 50 kg is replaced by a new man. the weight of the new man is (a) 65kg, (b) 75kg . () 76kg (d) 60 kg ‘Ans. (a) Letthe average w.bexkg beforereplacement Let the wa. of new men be y kg. Then, x42,5= S2=S0+Y 6 S6r+15=6x-504y a y=65 The average age of three boysis 15 years. t their ages are in the ratio 3:5: 7, the age of the youngest boy is (@) 21 years (b) 18 years (©) 1Syears (0) 9 years Ans. (4) MADE EASY ‘© Reasoning & Aptitude | »32 MADE EASY Let the ages be 3x, Sx and 7x years Then, t= StSet Te 9 15= 5x, x28 the age of youngest boy = 3 x3 = 9 years 24. a,b, ¢, d and e are five consecutive odd ‘numbers, then their averge is (@) S{a+b) (0) (abodeys (©) S(atbtc+d+dee) (d) None ofthese Ans. (d) Average = a+(a+2)+(a+4)+(a+6)+(a+8) 5 Bae zatdec 25, The average of frst five multiples of 3 is @ 3 ©) 9 © 1 ("15 ‘Ans. (b) ‘Average: Messe aS 26. The average weight ofa class of 90 students is 40 kg. If, however, the weight of the teacher Is included, the average become 41 kg. The weight of the teacher is (a) 31kg (©) 62kg (©) 71kg (d) 70kg Ans.(c) Lot the wt. of teacher be x kg. Then, 41= 40x90+x at => x= 41x31 1200 = 1271 ~ 1200 = 71 kg 9 27. The average weightof a school of 40 teachers is 80 kg. If, however, the weight of the principle be included, the average decreases by 1 kg, What is the weight ofthe principle? Average | 30 Let the weight of principle be xg ‘then, 79 = 4080+ =) x= 79x 41-3200 = 3239-3200 = 39 kg 28. The average temperature of 1%, 2 3 December was 24.4°C. The averag temperature of the frst two days was 24°C, The temperature on the '° of December was (@) 200 (b)250 (©) 2520 (d) None of these Ans. (0) Let the temp. on 3" of December be x © Then 24.4= = 25.2°C 2x24+x 3 29. The average of 20 results is 30 and thal ol 30 Wiar@ results is 20. For all he results tan together, the average is (a) & (0) 90 © 1 (@) 2 Ans. (4) The required average 20x30+30%20 _ 1200 _ 9, $0 50 ‘80. The average temperature on Monday, Tuesday ‘and Wednesday was 41°C. and on Tuesday, Wednesday and Thursday it was 40°. If on Thursday twas exactly 39°C, then on Monday, the temperature was (@) 420 © ae Ans. (a) ‘The sum of temp. on Tuesday and Wednestay (= 3 x40-39 = 81°C ‘The sum of temp. on Monday, Tuesday, anc! Wednesday = 3 x41 - 81 = 42°C (b) 46% (6) 260 @) 109kg (@) 29kg (0) 39kg (@) None of these Ans. (c) Sooo MADE EASY ‘© Reasoning & Aptitude | <3 34 | © Average MADE EASY Average {ind the average age of the family atthe bith of the youngest member. Pract 1 (@) 24years (©) 25 years ractice Exercise: (©) 20years @ zsyeers 1. The daily eamings of a taxi driver during @ ‘week are : RS, 60, Rs. 65, Rs. 70, Rs. 52.50, Rs. 63, Rs. 73 and Rs. 68, What's his average daily earning for the week? (@) Rs 74.50 (b) Rs54.50 (©) Rs 64.50 (a) Rs-84.60 2. Tneaverage ot 1Onumbersis 7. What willbe the new average if each of the numbers is ‘multiplied by 87 @ 45 (©) 82 (c) 56 (0) 5 38. There are 36 students in a hostel. If the number of students increased by 7, the expenses of the mess were increased by Rs. 42 per day while the average expenditure per head diminished by Rs. 1 Find the original expenditure ofthe mess (@) Rs. 480 (0) Rs. 440 (©) Rs. 820 (@) Rs. 420, 4, An aeroplane travels distances 2500 km, 1200km and 500 km at the rate of 500 km/h, 400 km, and 250km/hr, respectively. The average is (@) 420km/hr (b) 410 korvhe (©) 405 kmh (d) 575 kenshe 5. The average weight of 24 students of section A of a class is 58 kg whereas the average weight of 26 students of section B of the same class is 60.5 kg. Find the average ‘weight ofall the 50 students of the clas. (@ 57.4Kg. (0) 69.3 kg (©) 58.9kg (¢) 59.7 kg 6. The average age of 5 members is 21 years. Ifthe age of the youngest member be5 years, 7. The average monthly salary of a staff of 9 persons is Rs. 2450. One member of the staff whose monthly salary is Rs. 2650 is transferred. Find the average salary of the remaining 8 persons ofthe staf, (@) Rs, 2425 (b) Rs, 2625 (©) Rs. 9025 (@) Rs, 2825 8. Theaverage of five consecutive even numbers starting with 4, is @6 7 8 (75 9. 3 years ago the average age of a family of Smembers was 17 years. With the birth of a ‘new baby the average remains the same even today. Find the age of the baby, (@) 1 years (b) 3years ©) 2} years (0) 2yeare 10. The average of 17 numbers is 10.9. If the average of first nine number is 10.5 and that the list nine number is 11.4, the middle no. is (@) 118 (o) 11.4 (© 109 (d) 147 11. Abatsmen in his 17th innings makes a score of 85, and thereby increases his average by 3. What is his average after the 17" innings? He had never been ‘not out’, @ 4 (b) 37 (©) 39 (@) 4 12/The sum of three numbers is 98, Ifthe ratio, between fist and second be 2 : 3 and that between second and thitd be 5 : 8then the second number is (@ 20 (©) 20 (0) 58 (a) 48 MADE EASY * Reasoning & Aptitude | 34 MADE EASY + Average | 35 13. A number x equals 80% of the average of 5,7, 14 and a number y. Ifthe average of x and y is 26, the value of vis @ 8 (b) 26 © 99 (2) None of these 14, It rained as much on Wednesday as on all fs days of the week combined. Ifthe rainfall for the whole week was 3.om, how much did it rain on Wednesday? (a) 2.625cm (b) Som, (@) 10.50m @ 15cm 15. The average age of 8 men is increased by 2 years, When two of them, whose ages are 20 years and” 24 years respectively, are teplaced by two women, Whats the average (a) years (b) 30 years © 40years (oF a2 years 16. The average speed of @ train running at a ‘speed of. 30 km/hr during the first 100 kilometers, at 40 kin during the second 100 kilometers and at 50 km/hr during the last 100 klometers is nearly (@) 385kminr ——(b) 38.3 km/hr (©) 40.0kmimr = (d) $9.2 kmh 417. The average weight of 3 men A, B and Cis 84kg. Another men D joins the group and the average now becomes 80 kg. If another men E, whose weight is 3 kg more than that of D, replaces A, then average weight of B, C, D and € be 9 kg, The weight of A is (@ 70k (©) 72kg (©) 75kg (@) 80kg 18. Of the three numbers the first is twice the second and the second is thrice the third. f nc the second is thr thie average of the three numbers is 10 the numbers are 19. The average weight of 36 students is 50 kg. Iwas found later that the figure of 37 kg was misread as 73 kg. What is the correct, average? (@) 49kg (©) 51 kg (©) 50.5kg (@) None of these 20. The average of marks obtained by 120 candidates was 5. If the average of marks of passed candidates was 39 and that of failed candidates was 15, the number of ‘candidates who passed the examination is (@) 100 (@) 110 (© 120 (6) 160 21. Ina class, there are 20 boys whose average ageis decreased by 2 months, when one boy aged 18 years in replaced by anew boy. The age of the new boy is (@) 14 yrs. 8 months (b) 1S years (©) 16 yrs. 4 months (@) 17 yrs. 10 months 22. The average of two numbers is 62, If 2 is added to the smaller number, the ratio between the numbers becomes 1 : 2. The smaller numbers @) 6 (©) 84 28. The average dally wages of A, B and C is Rs, 120. IfB earns Rs. 40 more than C per day and A earns double of what C eams per day, the wages of A per day is (@) Rs. 60 (b) Rs. 120 (©) Rs. 160 (@) Rs.100 24. With and average speed of 40kmjhr a train reaches its destination on time. Ifit goes with an average speed of 35 kmihr, itis late by 15 minutes. The total journey is (b) 30 (@) 40 (@) 18,3,9 (b) 9,3, 18 (2) 30 km (b) 40 km, (©) 3,9, 18 (6) 18,93 (©) 70km (6) 80m, MADE EASY * Reasoning & Aptitude | 35 36 | * Average 25. In a competitive examination, the average marks obtained was 45. It was later discovered that there was some error in computrisation and the marks of 90 candidates had to be changed from 80 to 50 and the average came down to 40 marks. The total no, of candidates who appeared in examination is @) 520 (©) 540 (b) 550 (a) 525 26. Aman whose bowling average Is 12.4 takes 5 wickets for 26 runs and thereby decreases his average by 0.4. The number of wickets, taken by him before his last match is (@) 85 (©) 78 © 7 (@) 64 27. There were 35 students in a hostel. If the number of students increase by 7, the expenses of the mess increase by Rs. 42 per day while the average expenditure per head diminishes by Rs, 1. Find the original ‘expenditure of the mess. (a) Rs 480 (b) Rs. 420 (0) R820 (0) Res. 460 28, The average of these consecutive numbers isn. if thenext two consecutive numbers are also included, the average ofthe five numbers wil (@) remain the same (b) increase by 0.5 (6) increase by 1 (@) increase by 15 29. On the three numbers, second is twioe the first and is also thrice the third. Ifthe average of the three numbers is 44, the largest number is 3. Ans. (d) ‘and he was replaced by a member aged 29 years, The average age of the present committee is (@) 39 years (©) 36 years (b) 38 years (0) 35 years Solutions 1. Ans. (c) Total earning for 7 days = Rs (60 + 65 + 70 + 52.50 + 63 + 73 + 68) = Rs 451.50 Average deily eating MADE EASY 30. The average age of a committee of8 members is 40 years. A member, aged 55 years, retired 2. Ans. (c) The averagaot 10number =7 Total of 10 numbers = 10 x7 = 70 New toial of 10 numbers after each given numbers is multiplied by 8 = 70 x8 = 560 1 +. New average = 522 = 56, 10 Let the original expenditure = Rs x Original average expenditure = == x442 New average expenditure = == «+ Original expenditure = Rs 420 4. Ans. (a) (a) 24 (©) 3 ‘The total time taken can be calculated as (©) 72 (@) 108 shown below: MADE EASY © Reasoning & Aptitude | 36 | MADE EASY * Average | 37 Distance] “Speed | Time he average salary of the remaining 25km | 500 kmh | 5 hrs. Persons Ta00km | 400kmihr | hrs = 5 194 _ As, 2405: BOOkm | 250 krmihr | hrs. 8 Total 4200 km iohrs. 8 Ans. (c) ‘The five even consecutive numbers are Average speed = 4299 — 420 km/hr. 4,6,8, 10.and 12 0 Their average 5. Ans. (b) 4 10+12 ‘Average weight of 24 students of section ne A= S8kg Total weight of 24 students of section 9. Ans. (d) ‘A258 x24 = 1392 kg Present age of 5 members Average weight of 26 students of section = (5 x17 +3 x5) years. B= 605k9 = 100 years. Total weight of 50 student = (1392 +1573) kg, Present age of 5 members and a baby = 2965 kg = 17 x6 = 102 yrs Average weight of students of students in the class. 2965 = FR -s03k0 6. Ans. (c) Total age of § members 21 x5 = 105 year Total age of 4 members at the birth of the younger member, hat is, 5 years ago =105 ~(6 x5) = 80 years Before the birth of the youngest member, the family consisted of only 4 members. ‘Average age of 4 members 5 years ago 0 P= 20 years. 7. Ans. (a) ‘Average salary of 9 persons = Rs. 2450 Total salary of 9 persons = Rs, 2450 x9 = Rs. 22050 Total salary of the person who is transferred = Rs, 2650 ‘Thus, the total salary of remaining 8 persons = Rs, 22050 ~ 2650 = Rs. 19400 2. Age ofthe baby = (102- 100s =2years. 10. Ans. (a) ‘Sum of fist nine numbers + sum of last nine 10.5 x 9-11.4 x9 197.1 Hence, the middle number 2197.1 -17 «108 = 197.1- 185.3 = 118. 11. Ans. (b) ‘Average score before 17th innings =85-3x17 = 34 :, Average score after 17th innings =9443=97, 12, Ans. (a) Let the number be x, yand z. Then, xeysz= 08 508 and L=8 Ay 15 or, <2 = 98 or, y = 30. MADE EASY * Reasoning & Aptitude | 37 38 | * Average MAUE EAST 43. Ans. (e) Average of §, 7, 14 and y -STeiy ‘Therelore, x = 80% of SH+7+14+y 80 26+y 4 joo a i) xy Also “3 = 26 Ai) From (i) and (i), we get 52-y= Bey ay =39. 14, Ans. (c) Let the ainfall on Wednesday be x em so that on the other 6 days, the total is also x. Since average rainfall fo the week = Sem, 3x7 of x =108 om ort 45. Ans. (b) Let the average age of 8 men be x years :. Sum of the ages of 8 men = 8X years Now, according to the condition of the question, average age of (6 men+2 women) = (x +2) years <2. Sum of the ages of (6 men + 2 women) = 8 (x +2) = 8x + 16 yoars Hence, itis clear that on replacing two men by two women, sum of their ages increased by 16 years Therefore, sum of the ages of two women = (20 + 24) + 16 = 60 yrs 1. Average ageot he women 9230 ys 16. Ans. (b) Time taken to cover frst 100 kilometres Time taken to cover second 100 kilometres 100_ 41 “ao 72h Time taken to cover last 100 klometes 12 ons Total time taken dint 105 47 Sgt2gtea yest Shs. Total distance covered = 300 km -. Average speec 200, 7 Averag 76 300%6 a = 38.3km/hr 17. Ans. (e) Weight of D = (80 x 4-84 x8) kg = 68kg Wright of € = (68 + 3) kg = 71kg (B+C+D+Eys Weight = (79 x4)kg = 316 kg 2(B+CY's weight = [316 (68 +71)]kg =177kg * Hence, A's weight = [(84 x3}-177]kg = 75 kg 18. Ans. (d) ‘go by option 19. Ans. (a) Correct average 5096-79437 _ 1764 ae ae OD 20. Ans. (a) Lotthe numberof candidates who passed =x ‘Then, 39 xx +15 x(120-x) =120 x35 Ode MADE EASY © Reasoning & Aptitude | 38 { MADE EASY 21. Ans. (a) Total decrease = (20 x2) months = 3 years 4 months ‘Age of the new boy = 18 years -3 years 4 months = 14 years 8 months, 26. 22. Ans. (d) Let the numbers be xand y, x > = ae Proportion ‘The equality of two ratios is called proportion it ale Brg then a, b, © and d are said to be in proportion and we write a:bi:e:d This is read as ‘ais to be as cis tod’ For examples, since 2-15 we write 52 3:5 proportion, 45 : 25 and say 3, 5, 15, 25 are in a b proportion. a,b, cand d are respectively the rst, ‘second, third and fourth proportional. Here a, d are known as extreme s and b, ¢ are known as means. Each term ofthe ratio = and $ is called a ‘Some Basic Formulae 1 If four quantities are in proportion, then product of means = Product of Extremes for example, in proportion a: b::¢:d, we have bxceaxd Means rq a:bsre:d Ly Entremes From this relation we see that if any three of the four quantities are given, the fourth can be determined Fourth Proportional Itarb x, xis called the fourth proportional of a, b,c. b Thus; fourth proportional of a, b,c, is a Example: Find the fourth proportional to the. numbers 3, 8, 6 Solution: Letyrbe the fourth propértional, then 3.6 3:8::6:x0r B=— 6x8 _ ee Third Proportional la: b::b: xis called the third proportional ofab 16. b 2 or a Wehave = Thus, third proportional of a, MADE EASY * Reasoning & Aptitude | 42 SOR eee er pe er er Example: Find a third proportional to the numbers 3, 6 Solution: If third proportional is xthen 3:6:6:x Mean Proportional lta: x::x:b, xis called the mean or second proportional ofa, b x oy £ or x? sab z at x=vab Mean proportional of a and b is ab We also say that a, x, b are in continued proportion Example: Fd themean proportional between 4and 64. Solution: Let xbe the mean proportional. Then 4x x: 640r or x =4x64,x= 16 then (328 ach. £44 (componendo) (Oividendo) Example: The sum of two number is ¢ and their quotient is P/Q . Find the number. Solution: let the number be x, y Given x+y=0 ( ti) pq {using (i)} ooo Partnership Torun a business or any organisation, we need to have partnerships. There are two types of partners. 1. Working Partner: Those partners who invest the money and manage the business or affairs of organisation are called working partners. 2. Sleeping Partners: Those partners who merely invest money and do not involve in business affairs are called sleeping pariners. Mainly there are four types of conditions related to distribution of profit in business. Condition |: When time period of investment is constant then profit of partners will be divided into ratio My:M, Ms... My, where M,, My, M,:.m, are investment of respective partners. Ex.1 What willbe ratio of proftif three partners, A,B and C are investing 20000, 45000 and Rs. 60000 for one year. Sol.: Ratio of profits of A, B and C will be 20000 : 45000 : 60000 > 4:9:12 Ans. Condition II: When amount invested is same for different partners but time period is diferent then profit will be divided into ratio of their ime invested ie. ty tga ty MADE EASY + Reasoning & Aptitude | 43 44 |= mrawo 6 Froporton MADEEASY Ex.2 MthreeparnersA, BandCareinvestng gy. Investment Rs, 20000 each for period of 1 year, a A 8 ¢ 2 years and 3 years respectively then year | 120K = OK 0K find the ratio of their profits. year | 480K 420K = SOK Sol: Ratio of their profits willbe lit year | +100K 450K ~—=—«s100K 3s EEE Etfective Investment a e c Condition i: + year | 120K 80k 160K When amount of investment and time period IP year | 200K 100k 100k both are different for different partners, tt year | 300K 150K 200k then prof will be cvided into folowing ratio. Teak —aa0K BK Myty :Mgty: Mgly .. : May where m,;m,.... M, are the respective amounts and t, tp... t, are the respective time period of their investment. Ex.1 Three partners undergo a partnership. Their investments are 30000 Rs. 40000 Rs and 50000 Rs respectively for time periods of 5 years, 2 years and 3 years respectively. Find the ratio of their profits. Sol: Ratio of their profit will be ty My My = 90x5:45x2: 120x3 Ex, Ratio of their profit will be => 620:330:450 => 62:33:45 Ans. 2 Three partners X, Y and Z undergo a partnership with initial investment of Rs, 120000, Rs, 50000 and Rs. 150000 respectively. Next year A invested Rs. 50000 more in the business. In the third year B and C invested Rs. 50000 eck in the business, They remain in partnership for five years. Find the ratio ‘of their profits at the end of five years. Sol: Investment = 5:3: 12Ans AA BoC Condition IV: ‘Slyear [120K 50K 120K Multiple investment for different time period. lIndyear | 50K =~ In this case profit will be divided into following ildyear| - +506 450K ratio. Wyear | - - - Em: Empty :Emgly... Eyl, vyear | - = - : Effective Investment Ex.1 Three partners undergo a partnership A Bc with initial investment of Rs, 120000, “Riyear [BOK GOR 150K Fs. 80000 and Rs. 150000, Next year A Und yoar {170K 50K 1804 and 8 invested Rs. 80000 and Fs. 20000 ited year |1708° 100K 200 respectively whereas C withdraws IWyear | 170% 100K 200K Fs. 50000 from the business. In third year Veer [170K 100K 200K of ther partnership A, B and C invested wi = Fis, 100000, Rs. 80000 and Rs. 100000 Ratio of their profit will be respectively. , = 800: 400:900 = 8:4:9/Ans Find the ratio oftheir profits MADE EASY * Reasoning & Aptitude | 44 C) Solved Examples 1. Divide Rs. 500 among A, B, C and D so that ‘A and B together get thrice as much as C and D together, B gets four times of what C gets and C gets 1.5 times as much as D. Now the value of what 8 gets is (@) 300 (0) 75 (©) 125 (@) None of these Ans. (a) A+B4+C+D=500 Here A+B = 3 (C+D) $04(C +D) = 500 C+D= 125 A+B=375 also B = 4C and C= 1.5D C+D 125 x 25D= 125 0=50,C=75 2. B= 900 2. If x? + 6y® = 13xy, what is the ratio of xtoy? @ 2:3 (©) 3:2 © 4:5 1:2 Ans. (a) and (b) both 6x2 + 6) = 13xy Here using options (a) and (b) will satisty the given condition xry22:3 Next > Man. day hour = 160 x2 (twice as earlier) 2.20. h = 160 x2 hee In amixture of 40 litres, the ratio of milk and wateris 4: 1. How much water must be added to this mixture so that the ratio of milk and water becomes 2 : 3. (@) 20 litres (b) 32 ites (6) 40litres (6) 30litres Ans. (c) Let water is x 4x: x= milk: water 32:8 = 4x: x(since total 40 liters) 32.2 xe 40 litre Bex If three numbers are in the ratio of 1:2: 3 and half the sum is 18, then the ratio of squares of the numbers is: (@) 6:12:18, (b) 1:2:4 (©) 36: 144:324 — (¢) Noneof these Ans.(o) Letnumbers are x: 2x: 3x also itis given that $(x+ 2x + 3x) = 18 6x= 36, x=6 6:12:18 ratio of squares 36 : 144 : 324: or also 1:4:9 The ratio between two numbers is 3 : 4 and their LCM is 180. The first number is: 3. It4examiners can examine a certain number @ © us ‘of answer books in 8 days by working Shours os on day, for how many hours a day would 2 fans. (0) examines have to work in order to examine tot number are Se and 4x twice the numberof answer books in 20 days oe iso é : @ o Clearly HCF will be. ane. (0) 122 =x x 180.0 = 15 Man, Day, Hour = constant humbers will be 45 : 60 =4.8.5= 160 MADE EASY * Reasoning & Aptitude | 45 46 | * Ratio & Proportion MADE EASY 7. The incomes of Aand B are in the ratio 3: 2 and their expenditures are in the ratio : 3. If each saves Rs. 1000, than, A\s income is (a) Rs. 3000 (b) Rs. 4000 (©) Rs. 6000 (@) Rs. 9000 Ans.(c) Let incomes are 3x: 2x expenditures are Sy : 3y 1000 (1) also 1000 (2) from (1) and (2)we get 6x- 10y = 2000 3000 A's income is 6000 8. It the ratio of sines of angles of a triangles is, 1:1: YB, then the ratio of square of the gredtest side to sum of the squares of other two sides is (a) 3:4 () 2:1 (134 (@) Can'tbe determined ‘Ans. (c) sinA:sinB:sinC = 4:1: V2 sin A:sinB= 1:1 angle are equal So itis a right angled triangle A fs 45) c B the AB: (AC + CB)? = 4:1 9. Divide Rs, 680 among A, B and C such that Agets 2/9 of what B gets and B gets 1/4th of what C gets. Now the share of C is? (@) Rs. 480 (b) Rs. 300 (©) As. 420 (6) None of these Ans. (a) | 680=A+B8+C 2 1 23 ad = A= GB andB= 7C, 4B=C 2, A= 2B 0x48 c60= 28 +8448 00 = Ze, B= 120,¢ = 480 10. The students in three batches at Made Easy are in the ratio 2: 3: 5, If 20 students are increased in each batch, the ratio change to | 4:5: 7. The total number of students in the three batches before the inorease were. @) 10 () 90 (©) 100 (@) 160 Ans, (c) Let studefits are 2x: Ox: Sx now ‘According to the given condition 2x +20: 3x420: 5x +202 4:5:7 x=10 20:90:50 = 2:3: Sand after adding 20 40:50:70 11. The speeds of three cars are in the ratio 2:8: 4, The ratio between the times taken by these cars to travel the same distance Is (a) 2:3:4 (b) 4:3:2 (©) 4:3:6 (d) 6:4:3 Ans. (d) Speeds are in the ratio 2:3: 4 id ratio of ime taken wil be 3 75 MADE EASY ‘© Reasoning & Aptitude | 46 f 12 Ateranincrementot 7 inboth the numerator Ans. (¢) ‘and denominator, a fraction change t0 3/4 x: Bx: 4x: Tx k Find the original fraction. are four numbers also f (a) 5/12 (b) 7/9 X+ 3x4 4x4 7x= 105 | (©) 26 (@) 38 18x = 105, x= 7, 7x= 49 E Ans. (¢) 16. Ifthe rato of the ages of Maya and Chhaya f xeT 3 is 6: Sat present and fifteen years from now, f yar 4 the ratio will get changed to 9 : 8, then find 4x4 28 = 3y421, dx+7=9y, Mayals present age f only x: y = 2/5 satisfies the given condition. (a) 24 years (b) 30 years ¥ 13. The ditference between two postive numbers oO o (©) Styoere : is 10 and the ratio between them is 5:3. Find tet that ex: Sxthen, i t the product ofthe two numbers. at nei ges age Gx: Sxtnen, Its given t (@ 375 (0) 175 that . (©) 25 (@) 125 6x+15_9 [ Ans. (a) Sx+i5 8 k n ° a 48x + 120 = 45x + 195 £ xiy=8t:3t.@) beat xe 8 according to (1) and (3) 6x: x= 0:28 i St-3t = 10 Maya age is 30 years i 2-10, t=5 17. If Rs. 58 is divided among 150 children such ' x= 25, y= 15 that each git and each boy gets 25 p and 50 p Product of two numbers = 375 respectively, Then how many gis are there? k 14, The present ratio of ages of A and B is (a) 82 (b) 54 i 4:5, 18 years ago, this ratio was 11: 16. (©) 68 (0) & Find the sum total of their present ages. Ans. (c) k (@) 90years (0) 108 years Lot the number of giris and boys are xand y ) (6) 110 years (2) 80 years then Ans. (a) 0.25x+05y=58 (1) E Let ages are 4x: Sx then it is given that also xty= 150 (2) f anni tt from (1) and (2) we get ' Sx-18 16 O5x+y= 116 64x 288 = 55x- 198, x+y = 160 } k= 90, x= 10 $0, 05x = 34, x= 68, y = 82 4 ‘Ages are 40 and 50 sum = 90 ; 18. 11991: yananas was distributed among three y 15. Four numbers in the ratio 1:3: 4:7 add up mmorkeysin tne ratio 12 28. 8/4 Now many to give a sum of 106. Find the value of the taranes di the fist monkey ger? I biggest number. 1m ‘oi (a) 42 (0) 35 I o# oe (© 12 (¢) 104 I L MADE EASY * Reasoning & Aptitude | 47 yy & 48 | _* Ratio & Proportion MADE EASY Ans. (a) Ans. (c) Let number are 5x and 8xthen itis given that 5y+9_ 8 Bananas are in ratio 6x: 8x: 9x @x+9 11 Itis given that 55x +99 = Gar +72 6x+ 8x + 9x = 391 27 = or, x23 23x 301, x= 17 Number are 18 and 24 first monkey get 102 bananas 22. ifx varies as y, and y = 7 when x =18, find x 19. Amixture contains milk and water in the ratio when y= 21 5:1. On adding 5 litres of water, the ratio of @ % (0) 5 milk to water becomes 5 : 2. The quantity of (©) 72 (18 imik in the mixture is: Ans. (b) @ 16iites (©) 25 litres xis directly proportional to y ©) 325 litres (@) 22.75 tives $0,x=ky ‘Ans. (b) 18 = 7k Let mik and water are Sx: x 18 $0, k= 8 now, X+8_ 5 7 xe5 2 18 = 10x = 5x +25 now, x= x21 = 54 \ 5x= 25, x=5 milk: water = 25 : 10 20. Vijay has coins of the denomination of Re. 1, 50p and 25 p in the ratio of 12: 10: 7. The total worth of the coins he has in Rs. 75. Find the number of 25 p coins that Vijay has @) 4 (o) 72 (c) 6 (6) None of these ‘Ans. (d) Coins are 12x: 10x: 7x 1,7 Itis given that 12x + 10x 543% = 75 48x420x47x _ ane 76x = 4 x75, 75 xed 21. iftwo numbers are in the ratio of 5S: Band if9 be added to each, the ratio becomes 8: 11, Now find the lower number. @ 5 (©) 10 (©) 15 (@) None of these 23. 24. ‘A varies jointly as B and C; and A = 6 when B =3,C = 2; find A when B = 5, C=7. (a)175, (b) 35 (ce) 70 (d) 105 | ‘Ans. (b) | Let A = k (BC) then 6 (3.2) So, k = 1 then A=k(BC) (5x7) =35, It xvaries as y directly, and as z inversely and x=14, when y = 10; find z when x=49, y = 46 (@) 1410 (©) 10 (© 1014 (d) Cannot be determined | Ans. (d) xeky MADE EASY * Reasoning & Aptitude | 48 According to given values 142 k,10 z 14 8, = ed So, ky = 142, But value of k, is not given hence Z cannot be determined. ‘A.cask contains a mixture of 49 litres of wine ‘and water in the proportion 5 : 2. How much water must be added to it so that the ratio of wine to water may be 7 : 4? @ 35 (b) 6 ©7 (d) None of these Ans. (b) Let wine and water are Sx and 2xiitres 5x4 2x= 49 , 8x2 95, 2x=14 3__7 iaew 4 98 + Tw = 140 Tw = 42,w=6 ooo Ratio & Proportion Practice Exercise: | 1. Find a fourth proportional to the numbers 60, 48,30. | (@). 96 © 48 (b) 24 (@) None of these 2. Finda third proportional to the numbers 4, 42. () 441 (b) 541 (©) 64 (d) None of these 3. 10, In aratio 11 : 14, if the antecedent is 55, the consequent is. (@) 70 (&) 90 (©) 6 (a) None of these ‘The mean proportional of 0.25 and 0.04 is () 0.01 () 04 ©) 10¥10 (d) None of these The ratio of two numbers is 3: 4 and their ‘sum is 420. The greater of the two numbers is (@) 980 (©) 180 ‘Amit, Sumit and Puneet share an amount of Rs, 660 in the ratio of 3: 4: 5. What is the share of Puneet? (@) Rs. 375 (©) Rs. 575 (b) 240 (a) None of these (0) Rs. 275 (@) None of these ‘The prices of a scooter and a television set are in the ratio of 3: 2. if a scooter costs Rs. (600 more than the television set, then the price of television set is: (@) Rs. 1800 (o) Rs. 1200 (©) Rs. 2400 (0) None of these HA:B=7:5andB:C=9:11,then A:B:C is equal to (@) 95:45:63 (©) 69:45:55 (©) 45:63:55 (6) None of these IGA = 48 = SC, then A: B: Cis (@) 16:20:18 (©) 15:20:16 (©) 20:15:12 (6) None of these The ratio of money with Anju and Sanjuis 4 :5 and that with Sanju and Manju is § 6. If Anju has Rs. 280, then the amount of money Manju has (a) Rs. 320 (©) Rs. 640 (©) Rs. 420 (@) None of these MADE EASY * Reasoning & Aptitude | 49 ou | awe riuperiun 11, The sum of three number is 124. If the ratio, between the first and second be 2: 3 and that between the second and third be 7: 9, then the third number is: (a) 54 (b) 64 (©) 48 (@) None of these 12, The ratio between two number is 2:3. each number is increased by 4, the ratio becomes 5:7, The number are (a) 8,16 (0) 24,82 (©) 16,24 (@) None of these 13. The ratio of present ages of Suresh and Mahesh is 7:5. itafter 6 years their ages will be in the ratio of 4 : 3, the present age of Mahesh is (a) 32 years (©) S0years (0) 96 years (@) None of these 14, Two numbers are in the ratio of 6:7. If 25 be subtracted from each, they are in the ratio of 35:59, Find the difference of the two numbers, @ @ (b) 52 (©) 24 (@) None of these 15. The value of k that must be added to 7, 16, 43, 79 s0 that they are in proportion, is @7 (0) 5 ©9 (d) None of these 16. The number that must be added to each of, the numbers 8, 21, 13 and 31 to make the ratio of first two numbers equal tothe ratio of last two number is @5 o9 17. The incomes of A and B are in the ratio 3 : 2 and their expenditures in the ratio 8 : 3. If each saves Rs, 1000, A's income is (@) As. 5000 (b) Rs. 6000 (©) Rs. 8000 (2) None of these (b)7 (d) None of these 18. A mixture contains alcohol and water in the ratio of 12: 5. On adding 14 litres of water, the ratio of alcohol to water becomes 4 : 3. The quantity of alcohol in the mixture is (@) 18iitres (b) 42 litres (©) 26itres (4) None of these Solutions 1. Ans. (b) Let xbe the fourth proportional, then 60:48:38:x 07,02 = 20 Bx 30x38 AMS = 24, xe 2. Ans. (a) Let x be the third proportional, then 42 xor, 4 2 4:42 = 42% 42 xe = 4 aa 3. Ans. (a) 11 11K5 14” 14x5 Consequent = 70. 4. Ans. (b) Let xbe the mean proportional. Then, 0.25: x:x:0.04 3, b = 4and x= 420, 2. The first number MADE EASY ‘* Reasoning & Aptitude | 50 and, the second number bx _ 4x 420 a+b 344 = 240. 6. Ans. (b) Here, a= 3,b = 4,0 = Sand x= 660, ox arb+e «, Share of Puneet: 5660 34445 7. Ans. (b) Hote, a= 3, b = 2, and x = 600. The price of a television set bx _ 2x600 = 2X60 as. 1 a-b” 3-2 1200 8. Ans. (b) Here, n, = 7, n= 9, d, = 5 and d,=11 A:B:C = (n,xn,): (dyxn,) : (d,xd,) 7x9): (5 x9): (5x11) 63: 45: 55. Rs275. 9. Ans. (¢) WehaveA:B=4:3andB Here, n, = 4, n,=5, dy=3 and d= 4 ABC = (nyxmg): (d,xn,) : (d,xd,) 4x5): (3x8) :(3 x4) 20:15:12, 10. Ans. (b) We have, A:B=4:5andB:C=5:6. d,= Sand d= 6, Here, n LBC = (nyxng): (dyxn,): (d,xd,) = (4x5): (5 x5) :(5 x6) = 20:25: 3001, 4:5:6. Thus, ratioof money with Anju, Sanju and “Manju is 4:5 : 6, Sinoe Anju has Rs. 280, the amount of money Manju has = Ox =s.a20, 11. Ans. (a) We have, A: 8 =2:3and8:C=7:9. Here, n, d= 3 and do= 9, AsB:C = (nyxnq): (4,xn,): (xd) 2x7):(3x7):(3x9) = 14:21:27. Since the sum of the numbers is 124, the third 27 — et tea 54, number is 77r54¢57 * 124 = 54 12. Ans. (b) Wehave, a:b =2:8,c:d=5:7and x= 4 ax(o-d) The frst number = —W°—SF je frstrumber = Se 4x(5-7) 22x4x(5-1) ig (2x7=3x8) bx(c-d) and, the second number= 3x4x(5-7) © (@x7-3x8) 2 13. Ans. (c) Wehave,a:b=7:5,¢:d=4:3and x=6. bx(c-d) The present age of Mahesh= oF 14. Ans. (c) We have, a:b =5:7,¢:d = 35:59 and x= 25. ax(d~c) ‘ad=be _5x25x(59-95) _ © (6x59-7%35) The first number 60, bx(d-0) and, the second number = >= MADE EASY * Reasoning & Aptitude | 51 “ 52 | * Ratio & Proportion _7x25x (59-35) _ “"(6x59-7x35) The difference of two numbers = 84-60 = 24 16. Ans. (b) Here, a= 7, b = 16,0 = 43 and d = 79. ya benad * “= Grd=(bF0) __16x43-7x79 (7+79)-(16+43) 16. Ans. (a) Here, a = 8, b = 21,c = 13 andd = 31, The required number _ __be-ad @d=(F0) 21x13-8x31 3)-@r 13) ~ 17. Ans. (b) We have, a:b=3:2,¢:d=5:3and s= 1000, a8(d-c) Asincome = at _3x1000x (3-5) ""(8x3=2x5) Rs.6000. 18. Ans. (b) The two given ratios are 12: 5 and 4:3. Inorder to equate the antecedents of the two ratios, we write the second ratio as 12:8 Now, we have, a:b=12:8,0:d=12:9 and x= 14 The quantity of alcohol in the mixture MADEEASY Partnership Practice Exercise: | ‘Aman and Pranjal enter into @ partnership investing Rs, 50000 and Rs. 40000, respectively. They agree to share profits in the ratio of their capitals. Find the share of ‘Aman in a profit of Ris, 22500 atter one year. (@) Rs, 12500 (©) Rs. 9500 (6) Rs. 10500 (6) None of these 2. Amit, Nitin and Ravindra entered into a partnership. Amit invested Rs. 16000 for 9 months. Nitin invested Rs. 12000 for 6 months and Ravindra invested Rs, 8000 for 12months, At the end ofa year there was 2 profit of Rs. 26000, Find the share of Nitin inthe profit (@) Rs, 8000 (©) Rs.'6000 (©) Rs. 7500 (0) None of these A starts business with Rs. 3600 and § months after B joins A as his partner. After a year the Profits are divided in the ratio of 2: 3. How much did B contribute? (@ Rs. 7000 (©) Rs. 11000 (@) Rs. 9000 (@) None of these . Arvind began a business with Rs, 650 and was joined afterwards by Naveen with Rs, 330, When did Naveen joinit the profits atthe end ‘of the year were divided in the ratio 10: 32 @) After 4 months (b) After 6 months (©) After 4.5 months. (d) None of these A, B and C invested capitals in the ratio 3: 5:9; the timing of their investments being inthe ratio 2:3: 1. In what ratio would their profits be distributed? ax 12x14 (@) 2:5:3 (b) 3:2:5 e-b 12-5 Its (© 7:5:3 (2) None of these ooo00 MADE EASY ‘© Reasoning & Aptitude | 52 kk 6. A,B and C start a business. If the ratio of their periods of investments are 2: 3:6 and their profits are in the ratio of 4: 5:6, then the ratio of capitals of A, 8 and C is (@) 6:8:10 (0) 12: 10:6 (©) 10: 12:6 (@) None of these x A, B, C and D enter into partnership. A 1 aa pt ot subsoribes = of he capital, B =, Cz and the rest. What is the share of D out of a profit of Rs, 60007 (@) Rs. 2000 (©) Rs. 1600 (0) Rs. 1200 (6) Rs. 1300 8. A and 6 started 2 business with initial investments in the ratio: 7. I after one year their profits were in the ratio 1 : 2 and the Period for A's investment was 7 months, B invested the money for» (@) 6months (b) 2% (¢) 10months (@) 4 months © ‘A,B, Center into a partnership with shares in thoratio Z 4 2 Atter 4 months, A increase his share by 60%. I the total profit tthe end of one year be Rs. 21600, then Bs share in the profitis (a) Rs. 2100 (c) Rs. 3600 10. Bis a sleeping partner ang A working. A puts. in Rs. 5000 and B puts in 6000. A received 12.5% of profit for managing the business and rest is divided in proportion to their capitals. A's share of profit in-a profit of Fs, 880 is (@) As, 350 (6) Rs, 420 (b) Rs. 2400 (@) Rs. 4000 (b) Rs. 400 (2) Rs. 460 11. Astarts business with a capital of Rs, 1200. B and C join with some investments after ‘3.and 6 months, respectively. If at the end of a year, the profit is divided in the ratio 2:3: respectively, what is 8's investment in the business? (@) Rs. 2400 (0) Rs. 1800 (©) Rs, 3600 (8) Rs. 6000 Solutions Ans. (a) Here, C= 50000, C,=40000 and P = 22600, 4 Cy +C2 = 50000 + 40000 = 90000. CyxP C+C, = 3000022500 90000 Nikita's Share = = 222600 = Rs.12500. Ans. (c) Here, C= 16000, C,=12000, C, = 8000, t=9,)=6, ty= 12 and P = 26000. Nitin’s share in the profit = Op &ty xP City + Coty + Calg - 120006 26000 16000x9+ 12000 x6+8000% 12 = 1872000000 = Rs. 6000 312000” R* 600 |. Ans. (c) We have, C,xt; = 9500 x 12 Opxth=xx7 Profit for A “Profit for 8 The MADE EASY * Reasoning & Aptitude | 53 54 | * Ratio & Proportion MADE EASY 4. Ans. (b) LetNNaveen remaininthe business for xmonths. We have, C, xt; = 550 x 12 = 6600 C, xt, = 990 xx = 830%. Awind's share of profit _ Cx! Naveen's share of profit ~ G, xtz 8600%8 _ ¢ months. 33010 5. Ans. (a) Ratio of capitals of A, B and C are 3: 5:9. Let the capitals of A, B and C be 3x, 5xand 9x, respectively. Ratio of timing of their investments are 2:3: 1. Let A, B and C invest their capitals, for 2y, By and y months, respectively Then, profit of A : profit of B : Profit of C =O, xt, :Cyxt,: Cy xt Bx x2y : Sx xBy : 8 xy =6:18:907,.2:5:3. 6. Ans. (b) We have, P,:P,:P,=4:5:6andt, ity:t =2:3:6. Py Pp. P. Required ratio = 2:2 :-2 thot ts oy, 12: 10:6 Thus, A, B and C invested their capitals in the ratio 12: 10: 6. 7. Ans. (d) D's Capital = Profit ratio of A, B, C, Dis 20: 18:12:13 3 Share of D= — x Rs,6000 = Rs.1300. 10, 1". coon Ans. (¢) Let investments of A and 8 respectively be ‘Sxand 7x and period of B's investmentbe y months. = 105: 40:36 5 Let the initially invest Rs. 105, Rs. 40 and Rs, 36, respectively. Ratio of investments = [105 x4 + (150% of 105) x8) (36 x12) = 1680: 480: 432 = 35: 10:9 (40x12) Bs share= rx (21600x%2) =Rs.4000 . Ans. (d) ‘As shate for managing the business 2.5% of Rs. 880 = Rs. 110 Remaining proift = Rs. 770. Profit ratio of A and B = 5:6 As shate= 5 of Rs, 770 = Rs. 350. As total profit= As. 350 + Ris.110 = As. 460. Ans. (a) Profit ratio of A, B and Cis (1200 x 12) : (e x9) (y x6)=2:3:5 4200x12 _ ox 2 3 ss x=Rs. 2400, MADE EASY ‘+ Reasoning & Aptitude | 54 The term percent means “for every 100’. It is derived from french word ‘Cent’ which is 100 for trench. The basic utility of percentage arises from the fact that it is one of the most powerful tool for ‘comparison of numerical data and information. it is also one of the simplest tol for comparison of data In the context of business and economic performance, it is specifically, useful for comparing data such as profits, growth rates, ‘magnitude and so on. Calculation Since percentage represent the value obtained Out of every 100, calculation can be done in following manner Value obtained So x1 Total value *'°° Percentage = For example, let in IIT JEE examination Sanjay scored 135 marks out of 300 marks, Then Marks obtained by Sanjay in % term is equal to 135 oa 300 7% ‘As discussed earlier, percentage is amongst the ‘most powerful tool for comparison, et for example Percentage ‘Sweta scored 120 marks out of 150 and Rahul scored 360 marks out of 500 then Percentage 120 12 = 80% score of Sewta = 725 = 80' 360 Percentage score of Rahul =o = 72% ‘Thus, it clearly shows that Sweta’s performance is better than Rahul's performance. Other Comparison Let in Annual Examination Megha obtained 200 marks aut of 300 and Sangeeta obtained +100 marks out of 300, Now to compare marks of Megha and Sangeeta two question arise 1. Megha's marks is how much percentage greater than Sangeeta’s marks? 2. Sangeeta's marks is how much percentage less than Megha’s marks obtened? Megha 200 Sangeeta 100 Clearly Megha’s marks is 100 greater than ‘Sangeeta’s marks. its means difference of marks _ 100 eS = = 100% ‘Sangeeta marks — 100 Now Sangeeta’s marks is 100 less than Megha’s it means MADE EASY * Reasoning & Aptitude | 55 56 | + Percentage 1 MADEEASY difference of marks _ 100 Megha's marks 200 From this we can come at the result that Megha’s marks is 100% greater than Sangeeta’s marks while Sangeeta's marks is 50% less than Megha's marks, 50% Some Useful Short cut Methods 1.(a)__ IF Ais x% more than that of B then B is less than that of A by x [ass ca} IFA is x% less than that of B then 8 is more than that of A by 1.(b) [xm] Example: 1. If Shailendra’s salary is 20% more than that of Surendra, then how much percer salary of Surendra less than that of Shailendra. Solution: Here x = 20 Required Answer: x 100} x 100+ 20 = jp * 100% = 16.66% Example: 2. If Ais income is 30% less than that of B's income, then how much percent is 8's income more than A's income, Solution: Here x = 30 700-x Required Answer = ( x 100) 30 ~(Get00)x = 220% 2. IFAis 2% of C and B is y% of C then =x100% of B y Example: If Ais 20% of C and Bis 25% of C then what percentage is A of B. Solution: Here x = 20, y = 25 A=2x100= 22x 100=80% of 8 y 25 3.(a) Ifthe price of a commodity increase by P%, then the reduction in consumption 0.8 not to Increase the expendiue is) Pig (bas oo} It the price of a commodity decrease by P% then the increase in consumption s0 as not to decrease the expenditure is 3.(b) Pp (seca) Example: 3(a) If the price of sugar increase by 25%. Find how much percent its consumption be reduced so as not to increase the expenditure. Solution: Reduction in consumption Pp ~ (saga) 25 73g” 100% = 20% Example: 3(b) If price of commodity decrease bby 25%, find how much percent its consumption be increased so as not to decrease the expenditure. Solution: Increase in consumption P (00-P 100% 5g * 100% 4, If a number is changed (increased decreased) successively by 2% and y% then MADE EASY ‘* Reasoning & Aptitude | 56 ee x net% change is given by (=v a \which represent increase or decrease in value according as the sign in (+)ve or (ve. !ixor y indicates decrease in percentage then put (-Wve sign before x or y, otherwise (+)ve sign, Example: 4.(a) If salary or a personis increased bby 10% and 20% successively then, what is the change in his salary? Solution: Here x= 10. y=20 The net % change in the salary 10x20 700 Example: (4b) The price of a commodity first increasd by 20% then decreased by 10% then what is the net change price of commodity. Solution: here x = 20, 10 then net percent change in price is, =(10+204 = 30% [z0-104 28-1) Here sign is (+ve) hence the net is (+ve) increase in price, Example: (4c) Ifprice of a commodity decreased first by 20% and then by 30% then find the net change in price? Solution: Here x = 20, y = 30 Net % change'is x-30 [-20-s0+ 700 assem Since sign is (—-) ve the net change is reduction in price 5. Ifthe present population of a town (or value of an item) be P and the population (or value of item) changes in 1% per annum, then (2) Population (or value of an item) after n ry years ago =e +35) (6) Population (or value of an item) n years Pp a (38) Where ris (+ve) or (—)ve according as the Population (or value of item) increase or decreases. ago = Example: The population of a town increases 10% annually If its present population is 120000, what will it be in 2 years time. Solution: Here P=120000, r=10, n=2, :, Population after 2 years ry 1 =o 35) = 120000]1+ 110, 110 = 120000% 555 *95 Example: The population of a town increase at the rate of 20% annually due to excessive migration. f present populations 144000 find population two years ago. Solution: Here = P= 144000, 1=20 Population of the town two years ago 144000 720, 120 00 "100 6. Ita number A is increased by x%, y% and 2% Successively then final value of A will be ade) 5) 55) = 100000 MADE EASY * Reasoning & Aptitude | 57 58 | * Percentage In case a given value decrease by an percentage, we will use a (—)ve sign before that. Example: (6) The income of Rameshincrease by 10%, 20% and 30% successively in throe years find change in his income if present income is 150000 Rs. per annum Solution: Here x = So, percent change in income Vy) a(t al" Al aI 10 20 30. «sand o22)(1628)( 1-23] 10 ce 100 * 190 =160000 x 00 8.257400 1. To convert fraction into percent L to convert any fraction =~ to rate per unit ‘multiply it by 100. For example 1. 2 = 3x 100 = 60% 55 5 For example 2. 6 x 100 = 83.33% 2. To convert a percent into a fraction To convert percent into fraction, drop the % sign and divide the number by 100. 2 Example: What fraction is 165% 50 Solution: 164% = <3; 50 300 3. Tofind a percentage of given number 1 % of given number (N) = 700 xN Example: 1.65% of 300=7 65 Solution: = *900 = 795 Example: 2 Find a number whose 4% is 72. 4 : Here 72 = 2= ape 2x25 = 1800 Example: 3 What percentage of 30 kg is 2.59. = 100= bx 100=8.33% Solution: Bx 00 = 8.33: oooo | Solved Examples 1. Which of the folowing isthe largest number? (@) 20% of 200 (b) 7% of 500 (©) 1300% of 3 (d) 600% of 7 Ans. (d) 20% of 200 = 40 7% of 500 = 35 1300% of 3 = 39 600% of 7 = 42 2. If 25% of a number is 75, then 45% of that ‘number is: @) 145 (0) 125 (©) 150 (a) 135 Ans. (d) 25 Bxxar wor = 75%100 |, _ 499 45% of 300 = 135 MADE EASY * Reasoning & Aptitude | 58 ee BA kk eel tren a cos ‘So quantity of sugar that can be purchased, aE aT Ey = 160% of x 16, The price of rice falls by 20%. How much rice ‘can be bought now with the money that was sufficient to buy 20 kg or rice previously? (a) Skg (b) 15 kg (©) 25kg (©) 90kg Ans. (c) Fallin price by 20% now with same price one can purchase 100% more. 20 700-20 25% of 20 = 5 kg So total sugar = 20 + 5 =/25 kg 100% = 25% more, 16. 30% of a number when subtracted from 91, gives the number itself. Find the number. @ 6 (0) 65 () 70 {(@) None of these Ans. (c) 30, 10% of number x 00 30 Now 91- axe x ‘ 130, Mayen 100" *=70 17. The population ofa vilage is 1,00,000. The rate of increase is 10% per annum. Find the population at the start ofthe third year? (@) 1,33,100 (©) 1.21,000 (©) 1,20,000 (d) None of these Ans. (b) here P = 100000, r=10%, m= 2 year (At the start of third year Le. after two year) 121000 10 =1 + cooee( + ay . The population of the village of Rampur is 10,000 at this moment. It increases by 10% inthe first year. However, inthe second year, ‘dye to immigration, the population drops by '5%. Find the population atthe end of the third year if in the third the population increases by 20%. (@ 12,340 (b) 12,540 (©) 427,540 (@) 12,340 Ans. (b) P=10000, x=+10%, y=—5%, 220% Population at the end of third year “ellos vn Bh) 2540 ). In an examination, Mohit obtained 20% more than Sushant but 10% less than Rajesh. If the marks obtained by Sushant is 1080, find the percentage marks obtained by Rajesh if the full marks is 2000, (@) 86.66% (0) 72% (©) 78.33% 42) None of these Ans. (b) Marks obtained by Shushant is 1080 120 Mohits marks = 755% 1080 = 1296 Rajesh Marks, R=» 22-xR'= 1296. 100 R= 1440 MADE EASY + Reasoning & Aptitude | 61 62 | © Percentage MADE EASY 20. Ram spends 20% of his monthly income on his house hold expenditure, 15% of the rest of books, 30% of the rest on clothes and saves the rest. On counting, he comes to know that he has finally saved Rs. 9620. Find his monthly income. (a) 10000 (b) 15000 (©) 20000 (@) None of these Ans. (c) Lethe earns P Rupees x= -20, el tal tal za] Sle 9520 Pr. 476 = 9520, P=20000 -15, 2 = -30 1520 21. The population of a village is 5500. If the number of males increases by 11% and the number of females increases by 20%, then the population becomes 6330. Find the population of females in the town, (@) 2500 (b) 3000 (©) 2000 (6) 3500 Ans.(a) x is population of male (5500 ~) is female population xx111 , (6500. )x120 100 100 = 6330 22. Vicky's salary is 75% more than Ashu's. Vicky {gota raise of 40% on his salary while Ashu got a raise of 259% on his salary. By what percent is Vicky's salary more than Ashu's? (@) 96% (0) 51.1% (©) 90% (2) 52.1% Ans. (a) Lets Ashu’s salary = 100 Rs. Vicky's salary = 175 Rs. Vicky's salary increased by 40% i.e. 140 = 79g *175= 245 Rs Ashu's salary increased by 25% ie 125 Vicky's Salary is 120 more that Ashu's in % 120 = x100=96% term 5 23, An ore contains 25% of an alloy that has 90% iron. Other than this, in the remaining 75% of the ore, there is no iron. How many kilograms of the ore are needed to obtain 60 kg of pure iron? (@) 250kg (b) 275kg (©) 300kg (@) 266.68 kg Ans. (d) Let amount of ore is 100 kg 25% oF 25 kg has 90% iron 25x90 700 75% of 75 kg has no iron = Okg To obtain 22.5 kg 100 kg ore is required Soto obtain 60 kg 100 pg *00= 266.65k9 = 22,5kg iron 24, The population of a villages is 1,00,000. Increase raté per annum is 10%. Find the population at the starting of the fourth year. (@) 1,33,100 (©) 1,21,000 (©) 1,33,000 (d) None of these Ans. (a) Here P= 100000, r= 10, n= 3 Population at the starting of fourth year is vo200o[ 473] 133100 MADE EASY + Reasoning & Aptitude | 62 ee ¥ Pee Ce Dee Be 3. What is 20% of 50% of 75% of 70? (a) 8.25 (b) 675 (©) 7.25 (@) 25 Ans. (a) 20% of 50% of 75% of 70 2, Bx x70=5. 700 “iD 0-528 4, Mr. Rajesh is worried about the balance of his monthly budget. The price of petrol has increased by 40%. By what percent should he reduce the consumption of petrol so that heis able to balance his budget? (@) 9333 (b) 28.56 (0) 25 (6) None of these Ans. (b) We know that % redyction required is 100 here x=40 aorz ox 4 gp 100 28.56% 5, Ram sells his goods 25% cheapér than Shyam ‘and 25% dearer than Bram. How much percentage is Bram’s goods cheaper than Shyam's? (a) 93.33% (0) 50% (©) 66.66% (d) 40% Ans. (d) Let Shyam sell good at Rs. 100 ‘So Ram sell good at 75 Rs. Now this 75 Rs. is 25% dearer than Bram ie. 125 Bramx 12> =75 ram x5 Bram’s price = 60 Bram's good is 40 Rs. cheaper than Shyam in % term Bx 100 i ie. 40% 6. Inanelection between 2 candidates, Ravikant gets 65% of the total valid votes. If the total votes were 6000, what is the number of valid votes that the other candidate Shallendra gets if 25% of the total votes were declared invalid? (@) 1625 (o) 1575 (©) 1675 (6) 1525 Ans. (b) Total votes 6000 invalid votes = 25% of 6000 = 1500 Total valid votes = 4500 Ravikant gets 65% Socther candidate gets 35% 35% of 4500 =1575 In a medical certificate, by mistake a candidate gave his height as 25% more than normal. nthe interview panel, he clanied that his height was 5 feet 5 inches. Find the percentage correction made by the candidate {tom his stated height to his actual height. (a) 20 (b) 28.56 (c) 25 (d) None of these Ans. (a) Height increased =25% here x = 26 to reduce to normal % correction required is 25 8 x100= 700+25 * 00= 20% A number is mistakenly divided by 5 instead of being multiplied by 5. Find the percentage change in the result due to this mistake. (@) 96% (©) 95% (©) 240% (@) None of these Ans. (a) Let number is 100, Itis divided by 5 we get 20 Nowactual result should be x10 = 500 So % change is result 500-20 100= Bap *100= 96% . Rajiv wanted to subtract 5 from a number. Unfortunately, he added 5 instead of subtracting. Find the percentage change in the result, MADE EASY * Reasoning & Aptitude | 59 60 | * Percentage MADE EASY (@) 300% Percentage change in the volume of the (b) 66.66% cuboid. (©) 50% (@) 7% (b) 75% (@) Cannot be determined (0) 88% (0) 98% Ans. (d) ‘Ans. (d) This cannot be determined because after adding or substracting 5 to different numbers variable results are obtained. 10. In a mixture of 80 litres of milk and water, 25% of the mixture is milk. How much water should be added to the mixture so that milk becomes 20% of the mixture? @ 20litres (©) 1Slitres (©) 25 litres (@) None of these Ans. (a) Total mixture 80 litre 25 i Milk is 25% ie, <> x80 =20 litre tomake it 20% amount of water required 60+2 toadd = 55 x 100 = 80% +e 6000+ 100x= 6400 + 80x 20r = 400, x = 20 11. landowner increased the length and the breadth of arectanguler plot by 10% and 20% respectively. Find the percentage change in the cost ofthe plot assuming land prices are Uniform throughout his plot (0) 33% (0) 35% (©) 22.22% {(@) None of these Ans. (d) here x =10 ory = 20 ay he =| x a sangenace (x+y) 10x20 =10+204 32 12. The length, breadth and height of a room in the shape of a cuboid are increased by 10%, 20% and 50% respectively. Find the Let / b, h be length, breadth and height of the cuboid Volume v = 1b.h Now, |b, h are increased by 10%, 20% 50% respectively 10 20 50. vend of 2 ano = 1.98Lbh % change = 98% 1, The salary of Amit is 30% more than that of Varun. Find by what percentage is the salary cof Varun less than that of Amit? @) 26.12% (b) 23.07% (©) 21.28% (d) None of these Ans. (bb) Let Varun’s salary is 100 ‘Amit salary = 30% more that Varun's Salary = 190 in % galary of Varun less than Amit 30 Jap * 100= 23.07% 14. The price of sugar is reduced by 25% but inspite of the decrease, Aayush ends up increasing his expenditure on sugar by 20%. \Whatis the percentage change in his montly ‘consumption of sugar? (@) + 60% (©) -10% (©) +33.33% (d) 50% Ans. (a) Let price of sugar be x& expenditure E Now itis reduced by 25%. So itis. os now ‘expenditure of sugar is also increased by 20% ie 1260 SE MADE EASY * Reasoning & Aptitude | 60 25. Acow anda calf cost Rs. 2000 and As. 1400 respectively. It the price of the cow and that of the calf is increased by 20% and 30% respectively then the price of 1 dozen cows, and 2 dozens calves is: (a) 72,480 (b) 71,360 (©) 74,340 (d) None of these Ans. (a) Price of cow = Rs. 2000 Prige of calf = 1400 Price of 1 dozen cow & 2 dozens calves at reased rate eto e( 122 130 «105 ( 2a) ato 224 = 72480 26. Ram sells his goods 20% cheaper than Bobby and 20 dearer than Chandilya. How much percentage is Chandilya's goods cheaper/ dearer than Bobby's. (@) 33.33% (&) 50% (c) 42.85% (d) None of these Ans. (a) Let Bobby's sale price is 100 Rs. Ram sale price = 80 Rs. Ram sale price is 20% dearer than chandilya, So, Chandilya’s price = CP Now Chandilya good is $3.33 Rs. cheaper than Bobby's i. 33.63 = pp 100 93.89% 27. During winters, an athlete can run ‘x metres ‘on one bottle of Glucose, But in the summer, he can only run 0.5 xmetres on one bottle of Glucose. How many bottles of Glucose are required to run 400 metres during surnmer? (@) 800% (b) 890% ©) % (0) 454K Ans. (a) During summer to run ~ 5x one bottle of glucose required 1 1km —— bottles then to run t km === bottles < to-run 400m aux too= bottle 28. Out of the total production of iron from hematite, an ore of iron, 20% of the ore gets wasted, and out of the remaining iron, only 25% is pure iron. If the pure iron obtained in ‘year from a mine of hematite was 80,000 kg, then the quantity of hematite mined from that mine in the year is @ 5,00,000%g ——(b) 4,00,000kg (©) 4,50,000kg —(d) Noneof these Ans. (b) Let xkg ore is there 20% washed away so remaining is 80% i.e. 4/5 1 oat $zsiapueton ds 1 3* Kg is obtained from x kg then 1 kg is obtained from 5 kg ++ 80000 kg is obtained from 5 80000 = 400000 kg 29. In an examination, 80% students passed in Physics, 70% in Chemistry while 15% failed in both the subjects. If 325 students passed in both the subjects. Find the total number of students who appeared in the examination. (a) 500 (b) 400 (©) 300 (4) 600 Ans. (a) ‘* Reasoning & Aptitude | 63 | © Percentage 15% student failed in both subjects So total passed = 85% 80+ 70=85+x, x= 65% 65% of total = 325 total = 500. . 30. Ram spends 30% of his salary on house ent, 30% of the rest he spends on his children’s education and 24% of the rest salary he spends on clothes. After his expenditure, he is left wth Rs. 2500. What is Ram's salary? (@) Ps.6713.2 (b) Rs. 20,000 (¢) Rs. 10,000 (@) Rs. 18,000 Ans. (a) Let total salary is x Rs. 30% on house rent 30% of remaining on Childrens education =y 24% of remaining on clothes = z tte el Ze of ell dolls 2500 30], 30][,_ 24 [-3]-2 [-4] = 2500 {(-ve sign because of spending] 70 }f 70} 76 AR ek = 2500 P= As. 6713.21 oooo Percentage Practice Exercise: | 1 1. What percentage is equivalent to 57 (@) 525% (0) 625% (0) 425% (2) None of these 2 10. ot expressed as a fraction in its lowest term is 2 1 @g OF | os (2) None o th 20 (d) None of these What percent of 12 is 42 (a) 800% (b) 600% (c) 700% (@) None of these ar} of Rs. 48 is (@) Rs. 20 (0) Fis. 16 (© Rs. 18 (6) None of these 75% of 480 = (2) x 15 (@) 12 (b) %6 ! (©) 24 (@) None of these tt srhs of a number is 45, then ari% of the number will be @ 115 (&) 135 (o) 105 (d) None of these x% ol y + y%ofx =? (2) 3% of xy (0) 2% of xy (c) 5% of xy (B) None of these. x% ot y +7% of x=x% of (e+ y) @ x+y (b) x @y (2) None of these 25% of 25% =? (@ 625 (©) 0.0825 (©) 0625 (0) None of these’ 20% of 30% of 20% of Rs. 850 is ‘ (@) Rs. 9.50 (b) Rs, 10.20 (©) Rs. 10.50 (8) None of these MADE EASY * Reasoning & Aptitude | 64 11. 40% of 209% + 30% of 25% + 50% of 28% is ‘equivalent to (@ 29.5% (b) 28.5% (¢) 30.5% (d) None of these 12. Ifa number is 20% more than the other, how much percents the second number less than the first? 1 2, (a) 125% (b 165% ©) 163% (6) None of these 13, the given twonumbers are respectively 796 ‘and 28% of a third number, then what percentage isthe frst ofthe second? (a) 20% (b) 25% © 18% (@) None of these 14, The price cooking oll has ingreased by 15%. The percentage of reduction that a family should effect in the use of cooking oil so as not oincrease the expenditure on this aecount is 2 1 @ 6e% &) 13 35% © 13% (@) None of these 15. A man’s wages were decreased by 50%. Again, the reduced wages were increased by 50%. He has a loss of (@) 35% (c) 20% (0) 25% (d) None of these 16. The difference between a discount of 35% and two successive discounts of 20% and 20% on a certain bill was Rs. 22. Find the ‘mount of the bil. (@) Rs. 3200 (b) Rs. 2200 (© Rs, 1800 (@) None of these 17..Two shopkeepers sel a radio of similar brand and type at the same list price of Rs. 1000. The first allows two successive discount of 20% and 10% and the second allows the successive discount of 15% and 15%. Find the difference in discounts offered by the two shopkeepers. (@) Rs. 3.60 (b) Rs. 1.50 (©) Rs. 2.50 (2) None of these. 18. The taxon a commodity is diminished by 10% and its consumption increases by 10%. Find the effects on revenue. (@) 1% (b) 2% (©) 3% (d) None of these 19. The radius of a sphere is increased 10%. The surface area increases by (@) 21% (0) 31% ©) 41% (d) None of these 20. If the side of a square is increased by 30%, its area is increased by (@) 49% (b) 69% (©) 79% (2) None of these 21. In measuring the sides of a rectangle, one side fs taken 10% in excess and the other 20% in deficit. Find the error percent in area caloulated from the measurement, (@) 12%deficit ——(b) 10% deficit (©) 12% excess —_(d) None of these 22, Water tax is increased by 20% but its consumption is decreased by 20%. The increase or decrease in the expenditure is (@) 4%decrease (0) 4% increase (©) B%decrease — (d) 8%increase 23. The population of a city increases at the rate of 10% annually. Its present population is 90.51 lacs. The population 3 years ago was nearly (@) 72iacs (b) 68 lacs (©) 80 lacs (d) None of these MADE EASY ‘* Reasoning & Aptitude | 65 66 | * Percentage MALE EASY 24, The value of a machine is Rs. 6260. It decreases by 10% during the frst year, 20% ‘uring the second year and 30% during the third year. What will be the value of the machine after 3 years? (a) Rs. 2650 (b) Rs, 3050 (c) Rs. 3150 (@) None of these 25, An army lost 10% its men in war, 10% of the remaining due to diseases and 10% of the rest were disabled. Thus, the strength was reduced to 729000 active men. Find the original strength. (a) 1000000 (©) 1500000 (e) 120000 (d) None of these 26. In an examination, 30% and 35% students ‘respectively fallad in History and Geography while 27% students failed in both the subjects. It the number of students passing the ‘examination is 248, find the total number of students who appeared in the examination. (a) 425 (b) 380 (©) 400 (@) None of these 27.1n an examination, there were 2000 candidates, out of which 900 candidates were boys and rest were girls. f 32% of the boys 38% of the girls passed, then the total percentage of failed candidates is (@) 353% (©) 64.7% (0) 68.5%. @ 70% 28. Ifthe price of gold increases by 30%, find by how much the quantity of ornaments must be reduced so that the expenditure may remain the same as before? 2 1 (@ 275% ©) 2355 (©) 90% (@ 19% 29, The price of an article is cut by 20%. Torestore itto its original price, the new price must be increased by Solutions Vd (@) 20% (b) 22.5% ' (©) 2% (@) 40% i Inatraction, numerator's increasedby 25% I and the denominator is diminished by 10%. The new fraction obtained is 2 Theoriginal | fraction is i 2 8 @¢% Os I 3 | Og (2) Noneottrese ) Ans. (c) Let x%-of 12 700% of 12is 84. Ans. (c) 1 15 1% of RS.48 = 4 =Rst 377% of R548 = 48x = Ra 8. Ans. (c) Let 75% of 480 = «15. Then, 7 75x 480 79074002 15k ore eae MADE EASY * Reasoning & Aptitude | 66 Ans. (c) Let the number be x, then x a x 700"! 700%" = Fog 4) a aytar=x(ety) 9 ax=s2 ax . Ans. (b) 25% of 25% 25 25 625 * 70000 * 70000 = Ans. (b) 20% and 30% of 20% of B50 40.208 40% 0 20% = SOx ZO = = 8% 4 75,1 71% of x= 459 Bx x xe 815% l= M59 Ex Tera 45 ya gs v= 1528 «49, 8 1 754 “873% of 12 <4 x120=105. 2 00 Ans. (b) y 36 of yy% of am swt yy ctan( oe) (oe) 2 = oxy 22% of ay. Gap =a of . Ans. (b) r Let the missing figure be a We have, 90, 25 _ 75 ex 75% 30% of 25% = 309 * 00 * 300 50. 28 f 28% =e x 26 and, 50% of 28%= =F xT 1. (40% of 20%+ 30% of 25% + 50% of 28%) 9647 5%+1495229.5%. 12, Ans.(b) Here, x = 20 Required answer (air) 20 2, = 100) = 162 (ara co} 165% 13. Ans. (b) Here, I= 7, andm = 28. First number 100% ofsecond number = 3 x 100% of second number Or 25% of second number. 14, Ans. (b) Reduction in consumption P = 100% (wae 3} 15 1 (sedzagxt00}x oF 1355 % 15. Ans. (b) Here, x = -60 and y = 50. The net % change in wages ‘Reasoning & Aptitude | 67 MADEEASY 68 | © Percentage =m = 78 4000 = 144, 100% =[rng a) 27% of 1000 = 77x 1000 =Rs. on 50 -(4 50+50- ee or 25%. Difference in discounts offered bythe 100 two shopkepers = Rs, 280 - Rs.277.50 Since the sign is -ve, he has a loss of 25%. = Rs. 2.50. 16. Ans. (b) 48. Ans. (a) The equivalent discount of two successive discounts of 20% and 20%. zm =(vtye at) oe or -36%, -(0- -20+ Given: 36% - 35% = Rs. 22. Amount of the bill = 22x100=As. 2200. 17. Ans. (o) The equivalent discount of two successive discounts of 20% and 10% ~([vtye gd} 2x10) or 28% -(- 20-10 Discount on the list price of radio offered by the first shopkeeper 28 = = x1000=R: 28% of 1000 = +55 1000 = Rs. 280. Also, the equivalent discount of two” successive discounts of 15% and 15% YL =(sy+3 a) 18x1 -(- 15-15% or aie Discount on thelist price of radio offered by the second shopkeeper. Since tax xconsumption = revenue Not % change in revenue =(s9-d) (10% 10D) (here x = 10 and y = 10) =-1% ‘The revenue decreases by 1%. 19. Ans. (a) Since 4x xradius xradius = surface area Net % change in area -ose es 10x10 ={t (10+10¢ 700 20. Ans. (b) Since side xside = area Net % change in area “(ora 30x30 100) = (s0» 30+ = 60%. :. The area is increased by 69%. 21. Ans, (a) Since side,xside, = area 1 ‘ xy Error % 9% wr xnaea-(s0ye22) MADE EASY * Reasoning & Aptitude | 68 1% = 21%, r i} i i y h x X ® x x 1 x f 40x21 100 ~(10-20- (Here, x = 10 and y 12%, Le. 12% deficit. 22. Ans. (a) Since tax xconsumption = expenditure Net % change in expenditure afx+yr 2 («erat )s =09 99 -20%20 -(20 20-— 3 b=208y=-20] = 4%, Expenditure decreases by 4%. 23. Ans. (b) We have, P = 90.51, r = 10nd n = 3. ‘The population 3 years ago P 90.51 24, Ans. (c) Here,A = 6250, x = -10, y= -20 and z= -30. Value of the machine after 3 years alr ssl( bella) = 6250(1-22.)(1-20.)/,_.30. =o wa)! wl! wm) 6250x 90x 80x70 400% 100x100 = 5.3150. 25. Ans. (a) Let A be the original strength. The t+ ue niece ied = 729000 (Given) Here, x = -10, y = ~10 and z= ~10. 10 10 10 A(+-z05]-aae) (70) = 729000 __ 729000 x 100x100 x 100 ‘90x 90%90 = 1000000 men. 26. Ans. (c) Percentage of students passing the ‘examination 100 ~ (80 + 85 ~27))% (here, x = 90, y = 35 and z= 27] = 62(100- 38)% = 62%, Let the total number of students appearing in the examination x. Given: 62% of x = 248 =A €2 248x100 ey ea 2a 0, GHp = 288 oF Therefore, 400 students appeared in the examination. = 400. 27. Ans. (b) Boys = 900, Girls = 1100 Passed = (32 % of 900) + (38% of 1100) = 288 + 418 = 706 Failed = 2000 - 706 = 1294 (SHep0) = 4.7% Falled % =| 3665 28. Ans. (b) Reduction = 100% = 20-.% 30 400+30 29. Ans. (c) New price must be increased by MADE EASY * Reasoning & Aptitude | 69 maveeasy | 70 | + Percentage 2 (% of y + y% of x) =? ( 5% 00] = 25%, (@) x%ofy (0) y%ofx 100 (¢) 2% of xy (6) 27% of 8 30. Ans. (a) 7. IF 90% of A = 30% of Band B = xbof A, x then is equal to: Letthe fraction be (@) 900 (©) 200 (©) 800 (@) 600 x+0.25x i Then, oy 8. 40 quintals is what percent of 2 metric ) yee tonnes? | (@) 20% (0) 2% (©) 200% (@) 150% nooo 9. Astudent has to secure 40% marks to pass. He gets 178 marks and fails by 22 marks. The maximum marks are: (@) 200 (b) 500 Percentage © o0 (a 100 Practice Exercise: ! 10. Of the total amount received by Prema, 20% was spent on purchases and 5% of the remaining on transportation. Ithe is eft with 1. What is 25% of 25% equal to? Fis, 1820, the intial amount was: (a) 6.25 (0) .625 (a) Rs. 2800 (b) Rs. 2000 (c) .0625 (2) .00625 (©) Rs. 2400 (@) Rs. 1600 4 2. One fourth of one third of twofithofanumber 4. The price of jute has been reduced by 20%. ‘8 18. What will be 40% of that number? if the reduced price is Rs. 800 per quintal, (a) 120 (0) 360 the original price per quintal was (e) 270 (d) 180 (a) As. 900 (b) Rs. 640 3. What wl be 160% of a number whose 200% (©) Rs. 960 (2) Rs. 1000 is 140? 12. Amit has some apples, He sold 40% more (@) 200 (©) 160 than he are, Ihe sold 70 apples, how many (©) 140 (@ 112 did he eat? 4. IFthree fifth of anumber is 40 more than 40% @ 5 (b) 90 of the same number. What is the number? © 8 (9) 42 {a) 100 (b) 150 13. 5/9 part of the population in a village are males. () 20 (a) 400 If 30% of the males are married, the 5. IfXis 90% of ¥. What percent of Xis Y? percentage of unmarie females inthe total (a) 101.1 (©) 190 population is (©) 90 (a) 114.4 (@) 70% (b) 40% © 272% (©) 20% MADE EASY * Reasoning & Aptitude | 70 ee ee ee ee ee ee i 1 M 14, Aschool has only tree classes which contain 40,50.and 60 students respectively The pass percentages of these classes are 10, 20 and 10 respectively. The pass percentage ofthe schools @ 133 (0) 15 @x @ 168 3 15. 20% of As salary is equal to 20% of = th of Bis salary. If 85 salary is Rs. 2400, what is As salary? (a) Rs. 1880 (b) Rs. 1000 (©) Rs. 960 (@) Rs. 2160 16. Ifx% of ais the same as y% of b, then 2% of bis F @ Brot © ae ofa © oe ofa (@) None of these 17. A’s marks in Biology are 20 less than 25% of the total marks obtained by him in Biology, Maths and Drawing. If his marks in Drawing be 50, what are his marks in Maths? (a) 40 (bo) 45 (©) 0 (d) Can't be determined 18. From a container having pure milk, 20% is teplaced by water and the process is repeated thrice. At the end of the third operation, the milk is: (2) 40% pure (c) 51.2% pure (b) 60% pure (6) 58.8% pure 19, The salaries of A and B together amount to es, 2000. A spends 95% of his salary and B, ° 85% of his, If now, their savings are same, what is A’s salary? (@) Rs. 1500 () Rs. 750 (0) Rs. 1250 (a) Rs. 1600 20. 300 grams of sugar solution has 40% sugar in it, How much sugar should be added to make it 50% in the solution? (@) 10.gms (©) 40.gms (c) 60 gms (@) 80. gms 21, The price of an article is cut by 20%. Torestore itto the former value, the new price must be increased by (@) 20% (0) 25% (©) 162% (a) 24% 22. In an examination, there are 3 papers of Mathematics of 100 marks each. A boy secures 60% in the first paper and 70% in the second paper. In order to secure 70% in the aggregate the percentage of marks he should secure in third paper will be: (@) 90% (©) 80% (0) 75% (0) 70% 23. Two numbers are less than a third number by 30% and 37% respectively. How much percent is the second number less than the first? (@) 10% (0) 4% (0) 7% (2) 3% 24, Inan examination, A got 10% marks lass than 8, B got 25% marks more than C and C got 20% less than D. If A got 360 marks out of ‘500, the percentage of marks obtained by D was (a) 70 (b) 75 ©) & (2) 85 26. If the numerator of a fraction be increased by 16% and its denominator be dimimished by MADE EASY, ° * Reasoning & Aptitude | 71 72 | * Percentage MADEEASY 15 8%, the value ofthe fraction +The original fraction is 3 3 @s Oe 3 2 OF Or 26. Arvind spends 75% of his income. His income is increased by 20% and he increased his ‘expenditure by 10%. His savings are increased by: (a) 10% (0) 25% © ar (6) 50% 27. In an examination, 35% candidates failed in ‘one subject and 42% failed in another subject ‘while 15% failed in both the subjects. f 2500 candidates appeared at the examination, how many passed in ether subject but notin both? (@) 325 (0) 1175 (©) 2125 (@) None of these 28, The boys and girls ina college are in the ratio 3:2. If 20% of the boys and 25% ofthe girls vare adults, the percentage of students who are not adults is (@) 58% (c) 78% (b) 67.5% (a) 825% 29, The price of sugaris increased by 20%. Asa result, a family decreases its consumption by 25%. The expenditure of the family on sugar will be decreased by: (@) 10% (b) 5% (©) 14% (6) 15% 30. A building worth Rs. 133,100 is constructed on land worth As. 72,900. After how many years will the value of both be the same if land appreciates at 10% p.a. and building depreciates at 10% p.a.? 31. A reduction of 21% in the price of wheat enables a person to buy 10.5 kg mae for Rs, 100. What is the reduced price per kg? (a) Rs. 2 (b) Rs. 2.25 (c) Rs. 2.30 (0) Rs. 2.50 82, Salaries of A, B and C are in the ratio 1:23 Salary of B and C together is Rs. 6000. By what peroantis salary of C more than that of A? (a) 300 (b) 600 (©) 100 (A) 200 33, The length of arectangles increased by60%, By what percent would the width haveto be decreased to maintain the same area? (b) 60% (@) None @ ar3% © 75% 34. For a sphere of radius 10 om, the numerical value of surface area is what percent of the ‘numerical value ofits volume? (a) 24% (b) 26.5% (©) 30% (@) 45% 35. A cricket team won 40% of the total number ‘of matches it‘played during a year. If itlost 50% of the matches played and 20 matches were drawn, the total number of matches played by the team during the year was: (a) 200 (b) 100 () 50 (a) 40 Solutions 1, Ans. (c) 2525 1 2596 of 25% = 755% 3gp = 7g = 0628. 2. Ans. (d) 142 Lot pgx gene 16. (@) 25 (©) 2 ©) 15 @3 MADE EASY + Reasoning & Aptitude | 72 { ow Then, x = 15 x30 = 450 Now, 40% of 450 = 42 x 450 = 180, ° 100 ~ . Ans. (d) Let 200% of x = 140, 200, 8 . Then, pg X#= 140 0rx=70. 160 Now, 160% of 70= 55x 70= 112, . Ans. (c) 3 3,40 22-40% of x= 403 2x-O x= 40, Br 40% Of x= 409 Ex To = AO. Required percentage ¥ 40 ($100) = (42100) 14. 80.100, 400" 30 300 = F987 300% of A = 900 8 Ans. (c) Note that 1 metric tonne = 10 quintals. Required percentage 40. = 100 |= (25 0} 200% 9. Ans. (b) 40% of x = 178422 40. 100), Fog" = 200 orx (200% 2) 600. 10. Ans. (b) Purchases = 20% of x= = Balance = jalance = x-F == Transportation = 5% of ax. 1 198 = 4500 wp y = 1520%25 2 = 2000. 11. Ans. (d) 80. 0% of x= 800 x= 80% of x= 800 = Tex = 800 100 = x= 800% G5 = 1000. 12. Ans. (a) Suppose he ate x apples. Then, apples sold =140% of * Reasoning & Aptitude | 73 74 | * Percentage MADE EASY 18. Ans. (c) + 100). (x -(an"@)o-(2) Let, total population = x. Males = y . Married males = 30% of tot oe(x ofa y é Married females = z 17. Ans. (d) Let B+M+D =x. Then, B = (25% of x- 20) Total females = (-2) i -(% Sx Unmarried females -( : ) % Required percentage 5 [Betetoo)gearta 6 9 =(Bxttco)n= ards ‘ So, marks in Maths cannot be determing 14. Ans. (a) 48. Ans. (c) Number of passed candidates Let total quantity of original milk=1000 om 0 0 Milk after fist operation =( Bextor 2 xa0+ toceo)° 80% of 1000 = 800 gm, 100 co 10% Mik &ter second operation = (4410 +6)=20 = 80% of 800 = 640 gm. 2 4 Milk after thied operation Passed percentage= “9 100 = 80% of 640 = 512 9m (40+80+60) © Strength of fina mixture = 51.2%, 20 1 = (22 x100) = 134% 19. Ans. (a) (ie } 3 Let salary x, Then, B= (2000-2) 15. Ans. (c) 5% of A =15% of Bie. Y 35.,30A_ 20.3, 6 30% A= 20% of 3B. 0A. 20.35 20% of 58 = 00 ~ 4005 o FE (e000-) or += 1590 y 20 3. 109, oy 20.3100; 100" *-30 20. Ans. (¢) 1 2 ° (Z<2u00) = 960. Sugar = (Bxs00)ams #120 oms, I 16. Ans. (0) - water =180 gin’. ° : E Letx gm sugar may be addéd, I MADE EASY * Roasoning & Aptiude | 74 . ‘ «. 120+x 100 = 4 Then, Sag py *100= 50 240+ 2e =300+x= r= 60 21. Ans. (b) Let original price = 100. New price = 80. Increase on 80 = 20 Increaseon 100 = (2100) = 25% 22. Ans. (b) 60+70+= 2 «300 or r= 80%, -2 x 23. Ans. (a) Let, third number be x. Then, Ix First number = 70% of 63x Second number =63% of x = => Required Percentage Tx 10 «(xB 100) = 10% 24, Ans. (c) Ba aio and c. Px360 = 400, C= 2x 400=320 5 &D==x320 = 400. Percentage of D (s Bexto0)» = 20% 25. Ans. (b) Let the given fraction be xy MADE EASY 26. a7. 28. 115% of x _ 15 THEN, 93% of y Ans. (d) Let income = 100, Expenditure = 75 & Savings = 26. New income=20, New expenditure 1102) _ 165 -(Bx73) : vw mio 78 Increase in savings = Increase percent (2,1, . -(Bx3 100} =s0%. Ans. (b) 38 -(28 =875, Failed int subject ( S x20) 875. Failed in 28 uve x 2500) = 1050, 15 Falled in both {3 x 2500) = 375. Failed in 1® subject only Failed in 2" subject only: Passed in 2% only + Passed in ‘st only = (675 + 500) = 1175 Ans. (c) Suppose boys = 3x and girls = 2x. 80 75 Not adults ={ 22. Bx (Bxa)}s( Bx) * Redsoning & Aptitude | 75 76 | © Percentage MADE EASY 79. 2100 =Ro{ 22 2100 jig = Rs.2/i Rs, aie} a= Rs.2/i9 Required percentage 32. Ane. (@) Box 1 LetA =x, B = 2vandC = 3x, = (Sx gxio0)n=708 Thon, 2x + 3x = 6000 =x =1200. ‘A= 1200 and C = 3600. 29. Ans. (a) Required excess Let original consumption= 100 units & original price = Rs. 100/unit. 2400 ) = (7422, 10096 = 200%, Original expenditure = Rs. (100 x 100) (26 %100 J% = 200 = As, 10000. New expenditure= Rs. (120 x75) = Rs. 9000. 33. Ans. (a) Decrease in expenditure 1000 ~(Jp*100)6 108, 30. Ans. (d) 10y 10)" JOY 4 Jo rea00{ 1+ 2) 0100 3) (2)-(2)-=2 1331 30) “(a) * 7es00 ~ 729 GGT =m Let length = J and breadth = b. Let the required decrease in breadth be x%. 460, (100-2) Tx) xb = Ib TREN. F099! 00 = 160(100~ x)= 100x100 10000 _ 125 oF 100-x = Ea 125 4 = + (10002) -a73. 34, Ans. (c) Surface area nh? = 31. Ans. (a) Let original price = Rs. x/kg. Reduced price =3xVolume R 79 -(22 When R =10, we have (2)n0 Sy (2.x 100)% of v=30% of V 100 100 10000 _ 100 40" =\10* ove Fee 105 tos oxy 78x 100 35. Ans. (a) 10000-7900 = 10.5 «79% 40% of x + 80% of x + 20=x _ 2100 0 x xt SertQ0=r orr=200. 505x79 700" * 700 x Reduced price ooo MADE EASY * Reasoning & Aptitude | 76 Profit and Loss Profit and loss are part and parcel of every commercial transaction. In fact, the entire economy and concept of capitalizm is based on the s0 caléd ‘profit and loss Business transactions have now-a-days become common feature of life. Whett a person deals in purchase and sale of any item, he either gains or loses some amount generally. The aim of entire business is to earn profit The commonly used term in dealing with questions involving sales and purchase are: Cost Price The cost price of an article is the price at which an article has been purchased. It is abbreviated as CP. Selling Price ‘The selling price of an article is the price at which an article has been sold - It is abbreviated as 8. Profit or Gain Ifthe seling price of an article is more than the cost price, there is a gain or profit. Thus, Profit or Gain = SP. -C.P. Loss It the cost price of an article is greater than the selling price, the seller sulfers a loss, Thus, Loss = CP. - SP. Profit and loss are always calculated with the respect to the cost price of the item Profit Profit% «400 OF. Loss Loss% = 288 x1 088% = Ze x 100 Example: By selling an article at 500 Rs, Mohan incurs SORs. gain then find cost price of that article CP.=$P Gain CP. = 500-50 = 450 Rs Example: Ramesh purchased a radioset at Rs. 1500 and sold it at Rs. 1200. Find loss incurred by him? Loss= C.P, - SP {500 - 1200 = 300, Also in this case we can calculate Thus he incurred 20% loss. MADE EASY * Reasoning & Aptitude | 77 78 | * Profit & Loss Basic Formulae 1. When SP and Gain% are Given then 100 wooteme 3. When C.P. and loss% are given then 100-Loss% = WOT hOSS p s 100) cp. 4, When S.P. and loss percentages are given oral 100 mo) x00 5. If the cost price (C.P.) of m articles is equal to selling price of n article, then % gain or loss =. (=). 100 Itmon, itis % gain and if men, itis % loss Example: If the S.P. of 12 articles is equal to the cost price of 18 articles, what is profit%?. Solution: Herem=18,n=12 protit% = =") 100 7 = 18212 100-5 «100 = 50% 72 2 Example: Ifthe S. P. of a dozen apple is equal to cost price of 9 apples find gain or loss? Solution: Here m=9,n=12 2 x100 = (252x100 n MADE EASY 100 = -25% ale (-ve)sign indicates loss. 6. When two different articles are sold at the same S P,, getting gain/loss of x% on the first ‘and gairv/oss of y% on the second, the overall 4% gain or % loss in the transaction is given by 100(x+y) + 2xy pee ‘The above expression represent overall gain or loss according to its given (+)ve or (ve). 7. When two different articles are sold at the same selling price getting gain of x% on the first and loss of x96 on the second, then the overall % loss in the transaction is given by +Y a)% (3) Example: Michad! sold two TV. sets for Rs. 9600 ‘each gaining 20% on one and loosing 20% on the other. Find the total gain or loss percent Solution: Here x= 20 2 2 x 20) So, overall loss () % (3) 9% = 4%, 8, A merchant uses faulty measure and sells his goods at gain/loss of x%, Theroverall % gain or loss (g) is given by 100+g_ True measure 700+x Faulty measure Note: If merchant sells his goods at cost price then x = 0. MADE EASY * Reasoning & Aptitude | 78 Example: A dishonest shopkeeper professes to sell his goods at the cost price but use faulty measure. His 1 kg weight measures 950 gms only. Find his gain percent. Solution: Here, True measure= 1000 gms False measure = 950 gms ‘Sine the Shopkeeper sells the goods at cost price. x=, overall gain % is given by True measure _ 100+9 Faulty measure 100+x Discount 9. \If two successive discoynt of an article are m% and n% respectively, then a single discount equivalent to the successive m+n-2 |e discount will be 700 Itean also be calculated as (100-m) 100-100: { 500 Example 1: Two successive discount of 10% and 20% is equal to a single discount of 10x20 100 )=28 (10+20- Which is less than 30%, nooo Ex.1_ Find the single discount which is equivalent to a successive discounts of 50% and 40%. Sol.: Single discount will be equal to Ex.2._ Find the single discount which is equal to three successive discounts of 10%, 20% and 30%. Sol.: Here firstof all we will determine the single discount, which is equal to two successive discounts of 10% and 20%. 10x 20 700 = [ro+20- => 28% Now, we will ind a single discount which is equal to two successive discounts of 28% and 30% 28x30 > [20+00- “00 ls => 49.6% Ans. Ex.3 Finda single discount which is equivalent to three successive discounts of 50%, 40% and 20%, Sol: Single Discount equivalent to 50% and 40% is equal to ths 30 + 40 - = [soso ca = 70% Ans. ‘Now, we will find single discount which is equal totwo successive discounts of 70% & 20%, MADE EASY. * Roar * Apttnae | 79 80 | * Profit & Loss > [ro+20-0x20)y 400 = 76% Ans. £x.4_ Find asingle discount which is equivalent to three successive discounts of 20%, 30% and 20%. Sol.: Single discount equivalent to 20% and 30% is = [z0+20-22220), 700 => 44% Now, we will find single discount which is equivalent to two successive discounts of 44% and 20%, 44x20), 44420-: % - [ oo | => 55.2% oooo a 1. Byselling watch for Rs. 495, a shopkeeper MAUE EAS 2. By selling a cap for Rs. 34.40, a man gains 7.5% percent. What willbe the CP of thecap? (a) Rs. 32.80 (b) Rs. 32 (0) Rs.32.40 (d) Rs. 28.80 Ans. (b) SP. (ioo+ Gainey"? 34.40 = ppg toons ‘A shopkeeper sold goods for Rs. 2400 and made a profitof 25% in the process. Find his profit percent if he had sold his goods for Rs, 2040. (a) 6.25% (b) 7% | (©) 6.20% (A) 6.5% Ans. (a) SP = 2400, Profit% = 25 sp 2400 CP Tapa *100 = FE «100 = 1900 It sold at 2040, profit = 120 Rs. \ 120 san X 100 = 6.25 Profit % => 7920 x ! A dligital diary sold for Rs. 935 at a profitof 40%, What would have been the actual profit or loss on it, iit had been sold for Rs. 8107 | incurs a loss of 10%. Find the cost price of (@) Rs. 45, (b) Rs. 40 . the watch for the shopkeeper. (c) Rs. 48 (@) Rs. 50 (@) Rs. 545 (0) Rs. 550 ‘Ans. (b) (©) Rs. $55 (@) None of these SP = 935, Profit %= 10% ‘Ans. (b) 935 Here S.P.=495 CP = 5g % 100= 860 Loss = 10% Diary if sold at 810, Loss = 40 Rs. oP, = aay 100 By selling bouquets for Rs. 63, a florist gains (100-Loss%) 5%, At whal price should he sel he bouquets 495, to gain 10% on the cost price? OP = FP x 100 = 650 Rs @) Rs. 66 (bo) Rs. 69 (©). Rs. 72 (@) Rs. 72.50 MADE EASY * Reasoning & Aptitude | 80 ; = 19 | epmndy y Buuoseey « Asva 3QVN 0004 = 001 x Ot = % 04, %8= OLX“ NO ‘88 08 = dO UO Wold _ 008 = 0801 = 06 0021 ='dS JUNOOSIP %O}. TE Pjos 02 0001 = oo xD" oz = div (q) ‘suy %9 (9) KEL (0) 48 (0) OL (®) uso.ed youd ay) uly “coud peryew oui uO 9%0} Jo yUNEESID TE pos 3111 “20ud S09 eUN anOgE 5402 s1 “ youm'ooet ‘susieigere jo eoud perew oul “OL o4)_ for ey {@) x } abeuaored 580] pu sso|:nou| sheme am s9seo YoNS Ul (p) suy %OL 108807 (0) %y JOueD (e) uuonoesuen 3}OUM oun UO yUBOIed LIEB 0 S80} sIY PUL} ‘602 10 $80) B suByNS oy J0\IO OU UO PUB [002 jo youd ® s@yeus ay auo uO “eoud ewes eu) ye selddnd omy syjes seumo doys 1ed y “6 Sb 10.880 (p) $80] Ou youd ony (Q) SU SLL Uuonypuod usn6 0} Buipioooy SY Beans 10.d0 MON, SU 8g 88NOM JO. 4) 187 (0) ‘suy 002 "sx (P) Subse (©) Ost ‘sy (a) Szi se (e) 1y98N02, 2u JO 801d 1800 64) Puy 'sHeSNOA 94) UeY) 810Ui %Z} SISOO LIS OU J) “4.Z ‘SU S| Jesnon pu Uys B JO BoUld 1809 ey, [Kusee Aion panos eq ueo sia wer fevogoey ul) e902 i991 199'| = €8'02- 0822 = 0822 =dS H Wold %8 = OOLx % WO = =001xb= ©8802 = SH gy = O0LXG> = dO SZ=dS 5602 = % Old (p) suy %8 (0) %SL (0) 499'9 (9) %9 (e) aveuasenul ejoym ay) uo Ued!0d woud iy sey eiUINb 18d og'Ze ‘Sy 18 [200 OY sis 64 4) TeuInb sed gz “sy Ye je09 Bunjes 4g %02 40 Youd e seyeU yUEYOIeU! [209 y & seo Ee = 001 x S42 = % yor sede = 001 KE = % Wola ‘SY S20 = WlOld StI L = dS MON SH GL= SF = ereIO00U9 | 10 dd 6 ‘SH 6 = 812}000Y9 ZI 10 do (q) ‘suy %d@ (P) ‘re (0) swleiee (@) ——_se(a/t}99 Ce) gquadied youd siy sem rey “yore | “otf Je WeLA JO I pIOs ayy) ‘Uazop sad 6 sue Seve}000y9 OFZ 1yBnog radeeydous y ‘SU 99 18 PlOs 64 pinous 1 60} uleB 01 0g so 4 09 =001x 2b = 5 09 = O01 Xe = dO G = % 101d '€9 = dS (e) suy 9 82 | © Profit & Loss 11. A dozen pairs of gloves quoted at Rs, 80 are avaliable at a discount of 10%. Find how many pairs of golves can be bought for Rs. 24. (a) 4 ) 5 ©) 6 @s Ans. (a) 12 pairs of gloves are available at 8ORs. (with 90 1% — =72 Rs 10% discount) 80x =F = 72 A In 72 Rs. 12 pairs are available then in 10 pas 12 In 24Rstix24=4 In 24 Ras x2 12, Find a single discount equivalent to the discount series of 20%, 10%, 5%. (@) 30% (o) 31.6% (©) 68.4% (a) 35% Ans. (b) Single discount { (00. = 100-68.4= 31.6% 80 90 | 95 (100% $x 20 $5) 13. How much percent more than the cost price shoulda shopkeeper mark his goods, so that after allowing a discount of 12.5% he should have a gain of 5% on his outlay? (@) 9375 (b) 16.66% (©) 20% (A) 25% > Ans. (c) Let the cost price be x Rs. and printed price bey Rs. Hence, price alter giving a discount of 12.5% 14. 16. MADE EASY 87.5 _ 105 yx 2d 2 yx I 700 ~~" 400 105 105 175 Pe ray-re ee 875 a5 875 Required Percentage 578 5.400 = 20% a75 x 100 In order to maintain the price line, a trader allows a discount of 10% on the marked price of goods in his shop. However, he still makes ‘a gross prolit of 17% on the cost price. Find the profit percent he would have made onthe selling price had he sold atthe marked pric. (@) 2307 (b) 30%. fo) 21.21% (6) 25% ‘Ans. (b) Let the cost price be xRs. and marked price be y Rs. Given, y-yx tO = xt xxl Yo) 106 yx Manet yall, 500 =** 700° Y= "a0 100 G0 Awwpole-seller allows a discount of 20% on the list price to a retailer. The retailer sells at '5% discount on thelist price. ifthe customer paid Rs. 38 for an article, what profitis made by the retailer? (@ Rs. 10 (b) R88 (0) Rs. 6 (d) None of these Ans. (c) Let the list price be x As. Then, x= 2823 = 98 MADE EASY ‘* Reasoning & Aptitude | 82 ; t 95 TT x= ORs ‘The buying price for the retailer 20 =40-40x 20 = 0 — 40x55 = 92 Rs. Hence profit made by retailer = 98-92 = 6 Rs. 16. The cost of production of a cordless phone set in 2002 is Rs. 900, divided between material, labour and overheads in the ratio 3 4:2. the cordless phone set is marked ata price that gives a 20% profit on the ‘component of price accounted for by labour, What is the marked price of the set? (@) Rs. 980 (b) Rs. 1080 (©) Rs, 960 (@) None of these Ans. (a) Cost of phone accounted for By labour = x90 = 400 Rs B4ae2 Component of price accountant for by labour 20 = 400+ 400x <2 = 480 Rs 00+ 400% <5 = 480 Rs Marked price of the set = 480 + (900 - 400) = 980 Rs. 17. If subsequently in 2003, the cost of material, labour and overheads increased by 20%, 30% and. 10% respectively, calculate the cost of manufacturing in 2003, (@) Rs. 1150 (b) Rs, 1050 (©) Ps. 1080 (d) Rs. 1100 Ans. (d) In.2003, cost of material 20 =900+300%-55 = 360 Rs. Cost of labour 30. = 400+400x—— 00+ 400% 18, 19, Cost of overhead = 200+.200x 12. 700 220 As, Hence, cost of manufacturing = 360 + 520 + 220 = 1100 As. The cost price of 0 mangoes is equal to the selling price of 40 mangoes. Find the percentage profit? (@) 20% (b) 25% (e) 30% (@) None of these Ans. (b) Let the cost price of each mango be Rs. 1 Then cost price of 40 mangoes = 40 Rs. Selling price of 40 mangoes = 50 Rs. Percentage profit x 100 = 25% 50-40 “a0 ‘Acowns a house worth Rs, 10,000. He selis it to B at a profit of 15%. After some time, B sells it back to A at 15% loss. Find A’s loss or gain percent (a) 2.25% gain (©) 17.64% gain Ans. (d) Buying price of the House by B (b) 6.25% gain (@) 17.25% gain 18 = 1 => = 11500 Rs. 10900 + 10000% 75 = 11800 Price at which A buys house from 8. 18 = 11500-11600x 12 = 11500-1725 11500- 11500x 55 = 17500 = 975 Re. Hence A's gain percent _ 11500-9775 “0000 Anil bought an article at Rs. 200 and sold it at aprofit of 10%, What would have been the increase in the profit percent if it was sold for Rs, 230? 100 = 17.25% »MADE EASY © Reasoning & Aptitude | 83 84 | * Profit &Loss MADE EASY @ 8% (b) 10% Ans. (c) (©) 15% (6) None ofthese Let the cost price oft article is 1. Ans. (a) Cost price of 20 articles = 20 Re. Amount of profit when article is soldat profit Selling price of 20 article = 30 Fs of 10% +. Profitpercent = 22=20 409=50% | 10 = 200% 7755 = 20 Re ‘Amount of profit when article is sold for Rs. 280. = 280 - 200 = 30 Rs. Hence, increase in profit percent 30-20 100=5% 200) 21. A reduction of 10% in the price of sugar ‘enables a housewife to buy 6.2 kg more for Rs, 279. Find the reduced price per kilogram. (@ As.5 (©) Rs. 45 (©) Rs. 4.05 (d) None of these Ans. (b) Let original rate = Rs. x per kg New rate = 90% of 90 9x vae( ano Original quantity for Rs, 279 = 222 10 _ 310 New quantity = 279% === 2020 gy 2ao2 xe 31 =x 62 sxe =AS.4.5 per kg 22. If the cost price of 30 article is equal tothe selling price of 20 article, find the profit Reduced Price 23. 24, 25. ‘A shopkeeper sells sugar in such away that | the selling price of 950 gm is the same as the cost price of one kilogram. Find his gain percent, (@) 100/17% (b) 150/17% (©) 5(6/19)% (A) 119% Ans. (c) Let the cost price of 1 gm. is 1 Rs. Then cost price of 950 gm = 950 Rs. Selling price of 950 gm = 1000 Rs. Hence, gain percent (000 - 950 1000 OE 950 400 5 x100= 555% A dealer sold two TV sets for Rs, 2400 each, gaining 20% on one and losing losing 20% ‘on the other set. Find his net gain or netloss. (@) Rs. 300 loss (b) As. 200 loss (@) Rs. 200 gain (d) Rs. 300 gain Ans. (b) Selling price of two TV sets = 240042400 = 4800 Rs, Cost price of two TV sets 00 400 = 102400 4 12. $9 2400+ 5 % 2400 = 2000 + 3000 = 5000 Re. | Net loss = 5000- 4800=200Rs. | Aman sels an article at 5% above its cost price. if he had bought it at 5% less than k percent. what he paid for it and sold it for Rs. 2 less, (@) 33.33% (&) 40% he would have gained 10%. Find the cost price (c) 50% (d) 60% of the article. MADE EASY € Reasoning & Aptitude | 84 k F k © (a) Rs. 500 (@) Rs. 425 Ans. (d) Lot the cost price of the article = x Rs. Price when it is bought at 5% less than cost price (b) Rs. 360 (@) Rs, 400 105,» 95, 95,10 Gwen, F00*- 2" F00% 400" * 700 20.22% 00" "" 000 * = 400 Rs, = 2% 1000 5 . Abriefcase was sold at a profit of 10%. Ifits cost price was 5% less and it was sold for Rs. 7 more, the gain would have been 20%. Find the cost price of the briefcase. @) Rs. 175 (b) Rs. 200 (© Rs. 225 (a) Rs. 160 Ans. (a) Let the cost price = x Rs. 95 Price 6% less than cost price= => * Seling price when sold for Rs. 7 more =i. 47 110 10, 7-2. Given, 3og**7 “795 00" 100 eer 7600 OO 20 175 Rs. | 27. Aman sells a plot of land at 6% profit. he had sold it at 10% profit, he would have received Rs. 200 more. What is the selling price of the land? (a) Rs. 5000 (©) Rs. 4800 Ans. (b) Lt the cost price of land = x Rs. oven xt axeac Sean (©) Rs. 5300 (d) Rs, 5500 Selling price of land 6 5000+ 5000 ==> = 6300 Ris 28. A man buys two cycles for a total cost of Rs. 900. By selling one for 4/5 of its cost and other for 5/4 of its cost, he makes a profit of As. 90.0n the whole transaction. Find the cost price of lower priced cycle. (@) Rs. 360 (6) Rs. 250 (©) Rs. 300 (0) Rs. 420 Ans. (c) Let the cost of the two oycles be xand y Rs. Then, x+y = 900 “) Ai) Solving (') & (i), we get x = 300, y = 600, 29.A trader purchases apples at Rs. 60 per hundered. He spends 15% on the transportation. What should be the selling price per 100 to earn a profit of 20%. (@) Rs. 72 (b) Rs. 81.8 (©) Rs. 828 (6) Rs. 83.8 MADE EASY ‘© Reasoning & Aptitude | 85 86 | * Profit & Loss Ans. (c) Cost of 100 apples after transportation 15 = 60+ 60% 5 = 69 As Selling price per 100 apples to eam a profit of 20% 20 = 69+ 69x75 = 82.8 Rs 30. A dishonest dealer professes to sell at cost price but uses a 900 gram weight instead of 2.1 kilogram weight. Find the percent profit tothe dealer. @ 10% (©) 11.11% (©) 125% (6) None of these ‘Ans. (b) Let the cost price be x Rs. per kg Then cost price of 900 gm = Hence % profit ooo Profit & Loss Practice Exercise: | 1. Cost of 3 cricket balls = cost of 2 pairs of leg pads. Cost of 3 pais of leg pads of gloves. Cost of 3 paits of gloves = cost of 2 cricket bats. cost of 2 pairs MADEEASY 2. What is the loss percentif aman losesRs. 10 on seling an article for Rs, 100? (a) ote (©) 10% © i (2) None When a commodity is sold for Rs. 34.80, there is aloss of 25%. What is the cost price of the ‘commodity? (a) Rs. 46.40 (b) Rs. 26.10 (0) Rs. 43, (@) Rs. 43.20 A retailer buys a radio for Rs. 225. His overhead expenses are Rs. 15, He sells the radio for Rs. 300. The profit percent of the retailer is: @ 10 (b) 25 (©) 80 (d) 82 There would be 10% loss if a toy is sold at Rs, 10.80 per piece. At what price should it be sold to eam a profit of 20%? (@) Rs. 12 (b) Rs. 12.96 (©) As. 1440 () Nene If books bought at prices ranging from Fs.200 to Rs.350 are sold at prices ranging from Rs. 300 to Rs, 425, what is the greatest possible profit that might be made in selling eight books? (a) Rs. 400 (b) Fs. 600 (©) Cannot be determined (d) None of these The cost price of 20 articles is the same as seling price of 18 articles. The profit percent in the transaction is: Ifa cricket bat cosis Rs. 64, what isthe cost © % Ha of a cricket ball? wet (@) Rs. 12 (b) Rs. 14 ©) 335 (9) 50 (0) Rs. 16 (d) Rs. 18 MADE EASY * Reasoning & Aptitude | 86 8. Ifthe seling price of 18 articles is equal tothe CP. of 21 articles, the loss or gain percent is @ 162% gain (b) 142% gain © 162% loss @ abs loss 9. Aman sold 250 chairs and had a gain equal to seling price of 80 chairs. His profit percents: (a) 5% (b) 10% (©) 25% (a) 50% 10. Avendor loses the SP. of 4 oranges on seling 36 oranges. His loss percent is: 1 1 (@) 125% (b) 115% (©) 10% (d) None of these 11. A shopkeeper, on selling @ pen for Rs. 10, loses 1/11 of whatit costs him. His costs price is: (@) Rs.9 (6) Rs.1t (©) Rs. 10 (@) Rs. 12 12. If | purchased 11 books for As 10 and sold allthe books at the rate of 10 books for Rs. 11, the profit percent is: (@) 10% () 21% (0) 11% (6) 100% 13. Ajay bought 15 kg of dal at the rate of s.14.50 per kg and 10kg atthe rate of Rs. 13 per kg, He mixed the two and sold the mixture at the rate of Rs. 15 per kg. What was his total gain in this transaction? @ Rs. 1.10 (©) Rs. 16.50 (©) Rs. 11 (@) Rs. 27.50 14, Pure ghee costs As. 100 per kg. After adult- crating it with vegetable oil costing Rs. 0 per kg, a shopkeeper sells the mixture at the rate of Rs. 96 per kg, thereby making a profit (of 20%. In what ratio does he mix the two? (b) 3:2 (@ 1:2 1 (2) None of these (3 15. A dealer protessing to sel his goods at cost price, uses @ 900 gm weight fora kilogram. His gain percent is: @9 ~ (@) 10 1 @1 (a) Np 16. Toffees are bought atthe rate of 3 for a rupee. To gain 50%, they must be sold at: (@) 2forarupees (b) 1 for arupees (© 4torarypee (d) S fora rupees 17. By selling 45 lemons for Rs. 40, a man loses 20%. How many should he sell for Rs. 24 to gain 20% in the transaction? (@) 16 &) 18 ©) 2 (@) 22 18. Aman gains 10% by selling a certain article for acertain price. fhe sells itat double the price, the profit made is: (@) 20% (&) 120% ©) 100% (a) 140% 19. Aselis a bicycle toB at a profitol 20% and B sells it to C at a profit of 25%. If C pays Rs, 1500, what did A pay for it? (@) Rs. 825 (©) Rs. 1000 (©) Rs. 1100 (@) Rs. 1125 20. Ifthe manufacturer gains 10%, the wholesale dealer 15% and the retailer 25%, then the cost of production of a table, the retail price of Which is Rs. 1265 was: (@) Rs. 632.50 (b) Rs. 800 (©) Rs.814 (A) Rs. 834.34 21. Two mixers and one TV. cost Rs. 7000, while two TV's and a mixer cost Rs. 9800. The value of one TV. is: (@) Rs. 2800 (0) Rs. 4200 (0) Rs. 2100 (d) Rs. 8400 MADE EASY * Reasoning & Aptitude | 87 88 | * Profit & Loss 22. A horse and a cow were sold for Rs. 12000 each. The horse was sold at a loss of 20% * and the cow at a gain of 20%. The entire transaction resulted in: (@) no loss oF no gain (©) loss of Rs, 1000 (©) gain of Rs. 1000 (@) gain of Rs, 2000 23. An Article is sold at a certain price. By sling itat g of that price one loses 10%. The gain percent at original price is: @ 20% @) sod (©) 35% (@ 40% 24. A grocer sells rice at a profit of 10% and uses ‘weights which are 20% less than the market weight, The total gain eared by him will be: @ 30% (0) 95% ©) 37.5% (8) None of these 25. Tworthird of a consignment was sold ata profit (of 5% and the remainder at a loss of 2%, If the total profit was Rs. 400, the value of the consignment (in rupees) was: (@) 20000 {b) 15000 (©) 12000 (d) 10000 26. A truit seller has 24 kg of apples. He sells a partof these at 20% gain and the balance at loss of 5%. Ifon the whole he ears a profit of 10%, the amount of apples sold at loss is: (@) 6kg (0) 46kg (©) 96kg (@ 11.4kg 27. The C. P. of an article is 40% of the S. P. The percent that the S.P. is of C.P. is: MADE EASY 28. If an article is sold at 5% gain instead of 5% loss, the seller gets Rs. 6.72 more. The C.P. of the article is: (@) Rs. 67.20 (©) Rs. 134.40 (b) Rs. 120 (d) Rs. 240 29. Aman bought an artiole and sold it at a gain of 5%, If he had bought it at 5% less and sold it for Re 1 less, he would have made a profit of 10%. The C.P. of the article was: (@ Rs. 100 (©) Rs. 150 (©) Rs. 200 (@) Rs. 600 30. A reduction of 25% in the price of eggs will enable one to buy 4 dozen more eggs for Rs, 96. What is the price per dozen? @ Fs.6 (©) Rs. 8 (©) Rs. 8.50 (6) Rs.9 31, Raghu bought 4 dozen oranges at As. 12per, dozen and 2 dozen oranges at As. 16 per dozen. He sold them all to eam 20% profit, ‘At whet: price per dozen did he sell the oranges? (@) Fs. 1440 (0) Rs. 16 (©) Rs. 16.80, (¢) Rs. 19.20 82. The profit eared by selling an article for Rs. 900i double the loss incurred when the same article is sold for Ris. 450. At what prige should the article be sold to make 25% profit? (@) Rs. 600 (b) Rs. 750 (¢) Rs, 800 (0) Datainadequate 33, A man sold an article for Rs. 75 and lost something, Had he soldit for Rs. 96, his gain would have been double the former loss. The CP of the article is (@) Rs. 81 (©) Rs. 83 34. A single discount equivalent to a discount (b) Rs. 62 (6) Rs. 85.50 (a) 250 (b) 240 series of 30%, 20% and 10% is: © © @ 0 (@) 50% (0) 49.6% (c) 49.4% (d) 51% MADE EASY * Reasoning & Aptitude | 8 35. A table is offered for Rs. 300 with 20% and 10% off. If in addition, a discount of 5% is offered on cash payment, then the cash price of the table is: (@) Ps. 240 (©) Fs. 210 (b) As. 216 (@) Rs. 205.20 36. A tradesman marks his goods 30% above 1 the CP. Ihe allows a discount of 6%, then his gain percentis: ) ale (&) 22% © 2% (@) None 37. The difference between a discount of 40% on Rs, 500 and two successive discount of 36% and 4% on the same amount is: (@) Nil (0) Rs.2 (©) Rs.7.20 (a) Rs. 1.98 ‘38, What price should a shopkeeper mark on an atticle, costing him Rs. 153, to gain 20% after allowing a discount of 159%? (@) Rs. 224 (b) Fs.216 (©) Rs. 184 (0) Rs.162 39, Ifthe S.P. of Rs. 24 results in a 20% discount ‘on list price, what S.P. would result in a:30% discount on list price? (@) Rs. 27 (b) Rs. 21 (¢) Rs. 20 (A) Rs.9 40. A shopkeeper earns a protit of 12% on seling ‘ book at 10% discount on the printed price. ‘The ratio ofthe cost price tothe printed price of the book is: (@ 50:61 (&) 45:56 (©) 99: 125 (@) 55:69 41. A retailer buys 90 articles from awholesaler at the price of 27, If he sells this at their marked price, the gain percent in the transaction is 42. 43, @ ot% (©) 10% 1 2, 112% 165% © 15 © 165 A cloth merchant announces 25% rebate in prices. if one needs to have a rebate of Rs. 40, then how many shirts each costing Rs. 32, he should purchase? @5 (0) 6 O7 (9) 10 ‘A shopkeeper professes to sell all things at a discount of 10%, but increase the S.P. of ‘each article by 20%. His gain on each article is: (@) 6% (0) 8% (©) 10% @ Solutions Ans. (c) 8G =54x2= 108 > G=36, 9P =36x2=72P=24 8C=24x2=48 C= 16 Cost of a cricket ball= Rs. 16. Ans. (d) SP.=Rs. 100, loss= Rs. 10. So, 100 1000 P,=( 122,109} = Rs 10 c ( +100) 5 9 Loss% (10x00) 9%. Ans. (a) S.P.=Rs, 34.80, Loss = 25%, cP ns (xcs oo) =Rs 46.40 MADE EASY * Reasoning & Aptitude | 89 90 | © Profit & Loss 4, Ans. (b) C.P=Rs.(225+15)=Rs. 240, S.P.=Rs, 300 60. 2 Gain % Bosco) e258 5. Ans. (c) 90 _120 90; 10.80 = 120: x0" >a5 = p= 120%10.80 _ 44 49 Hence, S.P. = Rs. 14.40 6. Ans. (d) Least C.P, = Rs. (200 x Greatest SP. = Rs. (425 x8) = Rs. 3400. Required profit = Rs. (3400 -1600) = Rs, 1800, 7. Ans. (c) LetC.P. of each article be Re 1. CP. of 18 articles = Rs. 15 SP. of 16 articles = Rs. 20. 5 1 Gain =(5x 100}%6 = 392% “ (a e 3 8 Ans. (c) Let C.P.of each article be Re 1. CP. of 18 articles SP. of 18 articles = Rs. 21 3 2 ‘ a =(Zx100}x = 165% 9. Ans. (c) Gain = (SP. of 250 chairs) ~ (CP. of 250 chairs) (SP of 250 chairs) -(C.P.of 250 chairs) = SP. of 50 chairs. SP. of 200 chairs = C.P. of 250 chairs Let C.P. of each chair be Re. 1 C.P. of 200 chairs = Rs. 200. SP. of 200 chairs = Rs. 250. MADEEASY 50. =[ xt Gain% (B« 0x 25%, Ans. (c) (CP. of 36) - (SP. of 36)= Loss=5.Pot 4. SP.of 40= CP. of 36 Let C.P. of each mango = Re 1 C.P of 40 mangoes = Rs. 40 SP. of 40 mangoes = Rs. 36 4 Loser = (A xr00} 0% ‘Ans. (c) loss =(0P -SP) Lor cp.-10 eo Yop=t0 11 14x10) = =Rstt oP. ( 7 Rs, Ans. (c) ‘Suppose, the number of books purchased a11x10= 110 ©P.ot 10 books = Ra E110) Res, 100 it SP. of 110 books =A6( Hxt10) = Rs, 121, Profit % = 21% Ans. (d) CP. of 25 kg = Rs, (15 x 14.50 + 10 x 13) Rs. 347.50 S.P.of 25 k Gain = Rs. (375 ~347.50) As, (25 x15) = Rs. 375. 8, 27.50 Ans. (b) { 100, Mean price = ra(F5o x06) = Rs. B0)kg, By the rule of alligation: MADE EASY ‘© Reasoning & Aptitude | 90 Rofl kg Ghee Croft igo 100 > Mean ice co Required ratio = 30 : 20 = 3: 2 15. Ans. (d) 190 1 10. la = 112 Gain % (se*9} 119% 16. Ans. (a) C.P. of 8 toffees = Re. 1 8 P of 8 toons = 150% of Re 1= Rs. For Re, 3. toffoos sold = 3, For Re 1, toffees sold = (2 x. 17. Ans. (b) Let S.P. of 45 lemons be Rs. x 80 _ 120, 80: 40.= 120: x07 = 40120 y x= = 60 For Rs. 60, mons sold = 45 For As. 24, 45 \ =(2x24)=18. lemons sold & 24) 8. 18. Ans. (b) 110: x= (100 +P): 2¢ or 00+p oF 100+P = 220 20. 21 22, Ans. (b) 125% of 120% of 125, 120 A= 1800 = 228, 120 9 = 15 = 700700 ** 90° = (1500% Ans. (b) 125% of 115% of 110% of p=1265. 2 25115 110, * 500% 400 * 700° * 126 253 or Bop = 1265, 1265. 160 (=) s.800 Ans. (c) Let C.P. of @ mixer be As. x and that of @ TV. be Rs. y. ‘Then, 2x + y = 7000 and 2y +x = 9800. Multiplying 2nd equation by 2 and sub- tracting first from it, we get Sy = 19600 -7000=12600 or y = 4200 C.Rofal.V.=Rs, 4200 Ans. (b) Total S. P, = Rs. 24000 C.R.of horse = ao x 12000) Et C.P.of acow 100 =e, (= x1 000) Total C.P, = Rs. 25000. Loss = 24000 - 25000 = Rs. 1000 s.10000. MADE EASY ‘© Reasoning & Aptitude | 91 92 | © Profit & Loss MADE EASY 23. Ans. (c) Let original S. P. be Rs. x, x, loss = 10% 100, 2) _ 20% ore (a) ar 28. Ans. () Suppose, the quantity old ata loss be xkg New, cP,= 22%, sp ars. x, and let C.P. per kg be Re. 1 ar Total C. P. = Rs. 24. ain=(x-2) B Total S. P, = 120% of (24 - x) + 95% of x 6 49x _ 576~24x+19x = 8(24—x)4 19% _ 576 24e+ 18 Gait = (HE x-Z1 x100}n = 25% 5 2 % i 576-5 24. Ans. (c) Let us consider a packet of rice marked 576- 1a, Sam = 110% of 24 Its actual weight is 80% of 1000 gm og S16=5x _ 264 = 800 gm 20” 10 or 876-Sx=528 or Sx = 48 Let C.P. of each gm be Re. 1 .6 kg Then, C. P. of this packet = Rs. 800. S.P. of this packet = 110% of C.P. of 1 kg. of x= 27. Ans. (a) 40 5 790 *5P. 2 SP.=ZOP, 100} of cp. 2% «100 }% =37. 500% oo} 97.5% S.P.=250% of C.P. 25. Ans. (b) vs. (a) 28. Ans. (a) Let the total value be Rs. x Let CP. be Rs. x (105% of 2) ~ (95% of x) = 6.72 Value of 2 = &, Value of rd = = of 10% of x = 6.72 Totals. P, (105% of 24.98% az) 29. Ans. (c) a 3, Let original C.P, be Rs. x Its MADE EASY * Reasoning & Aptitude | 92 400 90, 80, 70 -( BH «100)=504 110 19x _ 209 «Single discount = (100-50.4)% =49.6% New S.P, = th xe = 0 35. Ans. (d) 2 20 1 n Gash price = 95% of 90% of 80% of Re. 20 200 “ 30. Ans. (b) =( BBB x<00) rac 20 Lot original price per dozen be As. x 16: 3x 36. Ans. (c) New price per dozen = oo => Let C.P. be Rs: 100. Then, marked price = Rs. 130 496 Now 98x54 p.=(100-22) ot R130 oF (128-96) = 4x orx = 8. 375 at . -(Bera0}= 121 875 1s. (12 x4 +16 x2) = Rs, 60 21875 _ 4,7 §,P.of 6 dozen orgenes Profits = 21.875 = 27078 = 217% =Rs. (Be<00) =A. 96, 37. Ans. (0) ‘st discount= Res. (40% of 00) S.P. per dozen = Rs. 16 40 =Rs| “2 x500}=Rs. 32. Ans. (b) (a i } 20 Let CP. be Rs. x 2nd discount = (36% of 500 + 4% of 64% 900-1 = 2 (x -450)=> Sx=1800-= x= 600. of 500). CP. = Rs, 600, gain required = 25% 4 64 1500+ fx 84500) sp. =Rs( 125x600) = Rs, 750 490° 100 400 = Rs, 192.80 53. Ans. (0) Difference = Rs.(200 - 192.80) = Ri.7.20 Let CP. be As. x 38. Ans. (b) 240-75) = (96-1) => Sx = 2464 = 82 CP, = Rs, 183, Gain = 20%, 34. Ans. (b) 420 Let marked price be Rs. 100. SP.= Fee199) =A 09 60, Then, S. P. = 90% of 80% of 70% of 100 Lot the marked price be Rs.» MADE EASY ‘+ Reasoning & Aptitude | 93 94 | © Profit & Loss MADE EASY 41. Ans. (c) 189.60 Let the C.P of each article be Re. 83.60% 100 Then, CP. of 30=Rs.27, SP.of 0. = 183.60%100 ig 85 3 1 Garr =( F100} 11g% 39. Ans. (b) Let, the list price be Rs. x 42. Ans. (a) Rs, 25 i the rebate on Rs. 100 Rs, 4O willbe the rebate on Required SP, = 70% of Rs.30 = Rs. 21 Rs. (Bato «Re 160 40, Ane.) Number of shirts purchased tthe GP. be Re. 100 lumber of shirts purchase: Then, SP. = Rs. 112, _ 100 Let the printed price be Rs. x 32 0 43. Ans. (b) Then, 90% of ¥= 1123 ox = 112 Let. P. be Rs. 100. Then, marked price = As. 120 . (nee) 1120 . 90 9 P= 108 = 1 ( ) SP. = 108= 120( (CP): (Printed price) = 10:42 0 000: 1120 « 45:86 2 sP.=Re( 2 x120)=Re 08 Gain = 8% nooo MADE EASY ‘* Reasoning & Aptitude’ | 94 Simple Interest itis calculated on the basis of a basic amount borrowed for the entire period at a particular rate of interest. ‘The amount borrowed isthe pringipal forthe entire period of borrowing. . PxAxT S09 Where P is principal amount Riis % rate of interest Tis the time duration Compound Interest The interest of the previous years are added to the principal for the calculation of the compound interes. Where P is principal amount R is Rate of interest Tis time duration Case: When interests compounded annually then amount A will be aT = [1+ a-[we| Simple Interest & Compound Interest Case2: When interest is compounded half yearly then Case3: When interest is compounded quarterly 1 R/Ay™ anes Case 4: When differential rate of interest is charged ic. if rate of interest is R1% for first year. R2% for second year and 3% for third year then Amount arf | fie Bei Be 700 }*['* 109 }."* F060. 0 Solved Examples Example 1: Calculate the simple interest if an ‘amount of As. 1500is deposited in ICICI Bank at 8% rate for 5 years, MADE EASY * Reasoning & Aptitude | 95 96 | © Simple Interest & Compound Interest MADEEASY Solution: Example 5: An amount become 5 times in PxRxT 20 years. Calculate the rate of interest given? soa Solution: Principal = x Rs. 1500x8x5. ‘Total Amount = 5x Rs., So Sie Ds 600 Sl = 4e Rs Example 2: A sum of 12000 Rs. is deposited now S1=27, 4x = into SBI Bank for 3 years. If the bank is providing 100 5% rate then calculate the amount after the 400 ‘maturity period? Ray = 20% Solution: Example: 6 In how many years an amount become double if rate of interest given is 20%? Solution: Here principal =x ‘Amount = Principal + Si Si= x, R=20% 19800 = 12000 + 1800 PRT Now Si= SE, x Example 3: An amount become double in 8 years 100 100 calculate the rate of interest? 100 Solution: T= 5p 7 5 years Let the amount be xs Example: 7 Since it become double in years 80Slin 5 ceain amounts deposited the bank. bank S years = x Rs. is giving 12% interest then calculate in how many Now gj=PXRxT, ,_ xxRx8 years amount will become 3 times the principal? 400 100 Solution: Here p =x, A= Sr So, SI = 2x R=12 . Example 4: An Amount become 3 times in 12 PRT years calculate the rate of interest? si= FST , 100 Solution: Let the amount be x Rs 200.2 Since it become triple in 12 years so Tap = 165 years. Sil. in 12 years = 2x Rs. exemple: 8 j now sta PBT, op = #XRM12 ‘An amount of 25000 is deposited in to Citi bank 00 100 for 2 years. Calculate the interest incurred ifrate of interest is 10% compounded annually? area Here P= 25000 Rs. | | MADEEASY * Reasoning & Aptitude | 96 y PAT 4. ¥xRx20 y r i ' t a t q ' t i t I I t t t | ( t | | | I I = 30250-25000 = Rs.5250 C] Solved Example: 2 1. Rs. 1200s lent out at 5% per annum simple interest for 3 years. Find the amount after 3 years. (@) Rs. 1380 (b) As. 1290 (e) Rs. 1470 (d) Rs. 1200 Ans. (a) Here P = 1200 Rs. R= 5%,T = 3 years PRT _ 1200x5x3 Si 09 = To” 180 Rs ‘Amount = P + SI = 1200 + 180 = 1380 2. Interest obtained on a sum of Rs. 5000 for ‘years is Rs. 1500. Find the rate percent. (@) 8% (b) 9% (©) 10% @) 11% Ans. (c) Here P = 6000, SI = 1800, T = 3 years. SIx 100 _ 1500x100 “PxT ~ 5000x3 ~ 1% Rs. 2100 is lent at compound interest of 5% =. per annum for 2 years. Find the amount after two years. © (a) Rs. 2300 (6) Rs. 2310 (0) Rs. 2315.25 (@) None of these Find the difference between the simple and the compound interest at 5% per annum for 2 years on a principal of Rs. 2000. @ 5 (b) 105 © 45 (d) None of these Ans. (a) Here P = 2000 Rs., R = 5%, arse B ‘| PRT = 2years 00} ~" |" 100 105 | 105) = [roms ee -} _2000%5%2 100, = 205 - 200 = 5 Rs. After how many years will a sum of Rs. 12,500 become Rs. 17, 500 at the rate of 10% per annum? (@) 2years (©) 3years (©) 4years () Syears ‘Ans. (c) Here A = 17500, P= 12500, R=10% SI=A-P }7500 - 12500 = 5000 $1x100 _ 5000x100 _ PxR — 12500x10 4 years. What is the difference between the simple interest on a principal of Rs. 500 being caloulated at 5% per annum for 3 years and 4% per annum for 4 years? (@) Rs.5 (b) Rs. 10 (©) As. 20 (6) Rs. 40 Ans. (a) ‘* Reasoning & Aptitude | 97 98 | © Simple Interest & Compound Interest years PRT, _PReTp 700 100 =5Rs, Diference= 7. What is the simple interest on a sum of Rs. 700 if the rate of interest for the first 3 years is 8% per annum and for the last 2 years is 7% per annum? (@) 400 (b) 992 (©) 352 (d) 266 Ans. (d) 700x3x8 | 700x7%2 700 * 100 “68 8. Find the compound interest on Rs. 1000 at the rate of 20% per annum for 18 months when interest is compounded halt-yearly, (@) Rs. 331 (©) Rs. 1331 (©) Rs, 320 (d)_ None of these Ans. (a) Here P =1000 Rs., R = 20%, Compound interest when interest is compounded annually . RI2 C= 1000+ 700 1 = 000{ 1+ = 1331 Rs. 400, 9. Find the principal ifthe interest compounded at the rate of 10% per annum for two years is Rs, 420, e @ Rs.2000 © (b) Rs. 2200 (©) Rs.1000 —(d) Rs. 1100 Ans. (a) P=?,Cl=420Rs,,R= 10%, T= 2years, _ 420%100 By = 2000 Rs 10. At what percentage per annum, will Rs. 10,000 amount to 17, 280in three years? (Compound Interest being reckoned) (a) 20% (b) 14% (©) 24% @ 11% Ans. (a) P’= 10000 Rs., A= 17280 Rs., years, R=? R 17250» one] +E 17280" R 3 ee = (1.2) 11. What is the rate of simple interest forthe first 4 years if the sum of Rs. 360 becomes Rs, 540 in 9 years and the rate of interest for the last 5 years is 6%? @) 4% (b) 5% (©) 3% (d) 6% Ans. (b) P= 360Rs., A=540Rs., Si = 180 Rs. MADE EASY ‘* Reasoning & Aptitude | 98 1: 2. Vinod makes a deposit of Rs. 100,000 in the HOFC Bank for a period of 2 years. i the rate of interest be 12% per annum compounded halt-yearly, what amount will he get after 2years, (@) 122,247.89 (©) 126,247.69 Ans. (c) P = 100000, R = 12%, T = 2 years Ri2y" * of Siz] (b) 125,496.79 (@) None of these st As +] = 126247.69 Rs 100000[1 i | '26247.69 Rs. [_ 19. What willbe the simple interest on Rs. 700 at 9% per annum for the period from February 5, 1994 to April 18, 19947 (@) Rs. 12.60 (b) As. 11.30 (©) Rs. 15 (0) Rs. 13 Ans. (a) Feb 5 to April 18, 1994 = 24 431418 = 73 days = 0.2 years PAT _ 700x9x.2 100 ~~ 100 14, Ajay borrows Rs. 1500 from two money- lenders. He pays interest atthe rate of 12% per annum for one loan and at the rate of 14% =126 Rs ‘Ans. (a) 41800, R= 12%, R,= 14%, T= 1 year Sl= 186 Letx Rs. be amount invested at 12% Rate 15. A sum was invested at simple interest at a certain interest for 2 years. It would have fetched Rs. 60. more had it been invested at 2% higher rate. What was the sum? (@) Rs. 1500 (b) Rs. +300 (©) Rs.2500 — (¢) Rs. 1000 Ans. (a) Since SI is directly proportional to Rate of interest 2% higher rate fetched Rs. 60 more it means this sum will give SI of Fs. 60 if invested at 2% rate for 2 years S1x100 _ 60100 RxT 2x2 = 1500 16. The difference between simple and compound interest on a sum of money at 5% per annum is Rs. 25. What is the sum? (@) Rs.5000 ——_(b) Rs. 10,000 (©) Rs. 4000 (@) Datainsutticient Ans. (d) Itcannot be determined. 17. Two equal sums were borrowed at 8% simple interest per annum for 2 years and 3 years respectively. The difference on the interest was Rs. 56. The sum borrowed were (@) Rs. 690 (©) Rs. 700 _ er annum forthe other. The total interest he (©) Rs. 740 (@ As. 780 ays forthe entire year is Rs. 186, How much Ans. (b) does he borrow at the rate of 12%? a) Rs. 1200 (b) Rs. 1300 PRT, _PRT2 (©) Rs. 1400 (@) Rs. 300 700 ~ 700 DE EASY * Reasoning & Aptitude | 99 100 | * Simple Interest & Compound Interest 8x3-8x2 ofessee2 =56, P=700 18. Ifthe difference between the simple interest ‘and compound interest on some principal ‘amount at 20% per annum for 3 years in Rs, 48, then the principle amount must be (@) Rs. 550 (b) Rs. 500 (©) Rs. 375 (A) Rs. 400 Ans. (c) Here P = 7, R= 20%, T = 3 year Difference 48 Rs. R PRT 7 onr[e se] -P- 100, 700 19. Raju lent Rs, 400 to Ajay for 2 years, and Rs. 100 to Manoj for 4 years and received together from both Rs. 60 as interest. Find the rate of interest, simple interest being calculated. (@) 5% (©) 6% (©) 8% @ 9% Ans. (a) PRAT PePoTe = 69 100 *” 100 4002xR , 100X4xR _ 6 700 100 12R = 60, = 5% 20. In what time will Rs 8000 amount to 40,000 at 4% per annum? (Simple interest being reckoned) (@). 100years —(b) SO years (©) *10years — (d) 160 years Ans. (a) P = 8000, A= 40000 MADE EASY R= 4%, SI = 32000, T=? S1x100 T=" BxP 32000 100 _ =" e000%4 10° years 21 sumof money becomes 4 times at simple interest in 10 years. What is the rate of interest? @ 10% (&) 20% (c) 30% (a) 40% Ans. (c) Let sum be x then A = 4x, SI = ax RT Se 0 x10xR gee *= 500 So, R = 30% 22. A sum of money doubles itself in 5 yeers. In how many years will it become four fold (it > 4 = [pus] > B22 lL > 3 T=10Yts. MADE EASY * Reasoning & Aptitude | 100 23. A sum of money placed at compound interest doubles itself in 3 years. In how many years will it amount to 8 times itself? (@) 9years (0) 8 years (c) 27 years (@) 7 years Ans, (a) Let sum =x ° Then, 2 = { + al 100. T =9¥6s, 24, Divided Rs. 6000 into two parts so that simple intereston the frst part for 2 years at 6% p.a. ‘may be equal to the simple interest on the second part for 3 years at 8% p.a {(@) Rs. 4000, Rs. 2000 (b) Ris. 5000, Rs. 1000 (©) Rs. 2000, Rs. 3000 (2) None of these Ans. (a) Let one part be x As. then xx2x6 _ (6000~x)x3x8 100 100 12x = 144000 - 24x 36x = 144000 x= 4000 25. A sum of money becomes 7/4 of itself in 6 years at certain rate of simple interest. Find the rate of interest. (@) 12% (b) 12.5% (©) 8% (a) 14% Ans. (b) Let sum be x aly 3 Aaa , So, Slnje =Gyears, A= pe S'x100 _ $xx100 PxT ~ xx6 = 12.5% 26. Sanjay borrowed Rs. 900 at 4% p.a, and Rs. 1100 at 5% p.a. for the same duration. He had to pay Rs. 364 in all as interest. What is the time period in years? (@) Syears (b) 3years () 2years (d) 4years Ans. (d) PRT | PoRoTe “00 * 100 * 96% 900x4T | 1100%5T 700 100) 367 +557 = 364 T=4Years 364 27. It a certain sum of money becomes double at simple interest in 12 years, what would be the rate of interest per annum? 1 @ 85 (e) 10 () 12 (a) 14 Ans. (a) _ xx12xR 100 100 2° °3 28. A sum of Rs. 600 amounts to Rs. 720 in 4 years at simple Interest. What will it amount to ifthe rate of interest is increased by 2%? (@) Rs. 648 (0) Rs. 768 () Rs. 726 (d) Rs. 792 MADE EASY * Reasoning & Aptitude | 101 102 | © Simple Interest & Compound Interest. MADE EASY! Ans. (b) P=600Rs, A=720Rs. T=4years, R=? 31x10 _ 120x100 PxT 600x4 at 7% Rate 600x7x4 BOOx7x4 _ 4 sy 68, A= 600 + 168 = 768 ooo Simple Interest Practice Exercise: | 1. Atthe rate of 6% p.a. simple interest, a sum ‘of Rs, 2500 will eam how much interest by the end of 5 years? (@)-Rs. 150 (b) Rs. 700 (6) Rs. 750 (6), Rs. 3250 2. If Alends Rs, 3500 to B at 10% p.a. and B lends the same sum toC at 11.5% p.a., then the gain of B (in Rs.) ina period of 3 yearsis: (@) 107.50 (©) 115.50 (©) 187.50 () 177.50 3. Avinash borrowed Rs. 5000 From Sanjay at simple Interest, After Syears, Sanjay got Rs. 300 more than what he had given to Avinash. What was the rate of interest per annum? (@) 2% (©) 5% (0) 8% (@) 10% 4, Rakesh took a loan for 6 years at the rate of 5% p.a. Sil If the total interest paid was Rs. 1230, the principal was: (@) As.4100 ——(b) As. 4920 (©) Rs.5000 —(d) is. §300 10, Rs, 800 amounts to Rs. 920 in 3 years at simple interest. If the interest rate is increased ‘by 3%, it would amount to how much? (a) Rs, 992 (b) As. 1056 (©) Rs.1112 (d) Rs. 1182 The simple interest on a sum of money at 5% is Rs, 48 for 4 years. The simple interest on the same sum for 5 years at 4% will be (@) Rs. 40 (0) Rs. 48 (©) As. 50 (0) Rs. 60 ‘A sum of money at simple interest amounts to Rs. 2240 in 2 years and to Rs, 2600 in 5 years. What is the principal amount? (@) Rs, 1520 (b) As. 1880 (©) Rs.2120 —(d) None ‘The simple interest on a certain sum of money at the rate of 5% p.a. for 8 years in Rs. 840, ‘At what rate of interest the same amount of interest can be received on the same sum after 5 years? @) 6% (b) 8% (©) 9% (2) 10% ‘A sum of money was lent at simple | 2 years respectively If the difference ininterests for two period was Rs. 412.50, the sum is: (@) Rs.3250 —(b) Rs. 3500 (©) Rs.3750 (qd) Fs. 4250 interest at 11% p.a. for 34 years and 4 Prabhat took a certain amount as a loan from a bank at the rate of 8% p.a. S.|. and gave the same amount to Ashish as a loan at the, rate of 12% p.a,Ifat the end of 12 years, he made a profit of Rs. 320 in the deal, what was the original amount. (a) Rs. 2000 (b) Rs. 3000 (¢) Rs. 4000 (d) None of these MADE EASY * Reasoning & Aptitude | 102 OR ee eer 11, Rahul borrowed Ri. 830 from MrLal at 12% pa. Sil for 3 years. He then added some more money to the borrowed sum and lent it to Shobha for the same period at 14% p.a. rate of interest. If Rahul gains Rs, 93.90 i the whole transaction, how much money did he add from his side? (@) Rs. 95 (b) Rs. 55 (©) Rs. 80 (@) Rs. 105 12, The simple interest on Rs. 1820 from March. 9, 1994 to May 21, 1994 at 73% rate willbe (@) Rs. 29 (b) Rs. 28.80 (©) Rs.2730 — (d) As. 22.50 13. Mr. Roopchand finds that an increase in the rate of interest from adato 53% per annum increases his Jearly income by Rs, 25. His investment is: @) Rs. 10,000 (b) As. 12,000 (©) Rs. 15,000 (4) Rs. 20,000 14, The rate at which a sum becomes four times of itself in 15 years at S.1. will be: @ 16% © 73% (©) 20% (6) 25% 16. The simple interest accrued on a sum of i money atthe end of four years is = thot its principal. Whet is the rate of interest per annum? (@) 4% © 6% 16. A sum of money triples itself in 15 years months. Inhow many years wouldit double itset? (a) 6 years 3 months (b) 7 years 9 months (0) 5% (6) Data inadequate 17, 18. 19. 20. 24. (©) 8 years 3 months (6) 9 years 6 months i The simple interest on a sum of moneyis = of the sum. The number of years is numerically equal to the rate percent per annum. The rate percent per annum is: 1 @ 93 © s 2 62 () 83 @ 10 ‘A sum of Rs. 10 is lent to be returned in 11 monthly installments of Re. 1 each, interest being simple. The rate of interest is: 1 @ 97% (0) 10% ©) 11% (@) 25% i The rate of simple interest on a sum of money is 6%'p.a, for the first 3 years, 8% p.a. for the next 5 years and 10% p.a. for the period beyond 8 years. ithe simple interest accrued by the sum for a total period of 10 years is Rs. 1560, what is the sum? (@) Rs. 1500 (b) Rs. 2000 (©) Rs. 3000 (@) Data inadequate ‘A monthly installment of Rs. 180 is required to be paid for repayment of an interest free loan in 40 months. I itis decided to pay it in 30 months, how much will be the monthly installment in rupees? (@) 60 (b) 198 (©) 240 (@) 330 ‘A.sum of Rs. 1850 is lent out into two parts, ‘one at 8% and another one at 6%. Ifthe total annual income is Rs. 106, the money lent at 8% is (a) Rs. 650 (©) Rs. 840 (0) Rs. 720 (@) Rs. 900 MADE EASY ‘* Reasoning & Aptitude | 103 104 | © Simple Interest & Compound interest 22. A sum of Rs. 1550 was lent partly at 5% and partly at 8% p.a. simple interest. The total interest received after 3 years was Rs. 300. The ratio of the money lent at 5% to that lent at 8%. (@) 8:5 &) 5:8 (©) 31:6 (6) 16:15 23, What should be the least number of years in which the simple interest on Rs. 2600 at 2 65% willbe an exact number or rupees? @2 &) 3 ©4 @s 24, Aman invests an amount of Rs. 15860 in the names of his three sons A,B and C in sucha way that they get the same amount after 2,3, and 4 years respectively. Ifthe rate of simple interest is 5%, then the ratio of amounts invested among, A, B and C will be: (@) 10:15:20 (b) 22:23:24 (©) 6:4:3 (@) 2:3:4 25. Rs. 2189 are divided into three parts such that their amounts after 1, 2 and 3 years respectively may be equal, the rate of simple interest being 4% p.a. in-all cases. The smallest part is (@) Rs. 702 (b) Rs, 597 (©) Rs. 756 (@) Rs. 1093 Solutions 1. Ans. (c) 3500%11.5%3_9500x10x3 100 100 = Ris, (1207.5 ~ 1050) = Rs. 157.50. |. Ans. (a) 100x300) 5000%3 Rate -( ‘a = 2%, . Ans. (a) Rs.4100. Principal= Rs. (eu) 6x5 Ans. (a) Principal = Rs. 800, S.1. = Rs. 120, Time = 3 years. 100% 120 800x3 asx New rate=8%, Time 3 years. (800x8x3 100) si Re( ) = Resa Re, (800+ 192) = Rs, 992 240x5x 4 ps 240x5%4" si ef 700 = Rs48 Ans. (d) S.I. for = Fis, (2600 - 2240) = Rs. 360. 360 S.1. For 2 years = Rs. “3 2 = Rs.240 Principal = Rs. (2240-240) = Rs. 2000 . Ans. (b) 100x840 = ( 100x840) Rs.2100. Sum ( ea8 ) 15.2100, (oyee 2100%5 Rate required: =8%, MADE EASY '* Reasoning & Aptitude | 104 ; } k 9. Ans. (c) Let the sum be Rs. x. Then, gt 74 (ext Seg ard) = 412.50 22x op 7 41250 = = 41250 = ¥=3750 10. Ans. (d) Let the original amounts be Rs. x. then, 2x12K12 _xx8xt: 100) 100) = += 2000 ns oei67 11. Ans. (d) (890+x)x14x3_ 830x123 100 700 oF 830 x42 + 42 ~ 830 x36 = 9390 42x = 9390 - 4980 .90 4410 _ 2 Money added = Rs. 105. 12. Ans. (c) 05. March, April, May 22 +90 + 21 = 73 days 73. 1 eg oats gears Interest 1 15 oro teox!a®—) 13. Ans. (a) Let the investment be Rs. x, Then, = 2x=20000 = x= 10000, 14, Ans. (c) Let sum = x, Then, S. |. = 3x Rate -( Se = 20% xx15 15. Ans. (b) Let sum =x, Then, Sl-==,Time = 4 years. 5 xd «feos 16. Ans. (b) Let sum =x, Then, Si. = 6 Tine = 2 yeas. 100x2x 2 oe a ( ai 400 Now, sum = x, Slax & Rate {37} % tox, 31) x a00 YS = 7 years 9 months Time 17. Ans. (a) Let sum «3, Then, Se. Let time = n years and rate = 1 yt xn n= 100% x MADE.EASY * Reasoning & Aptitude | 105 106 | © Simple interest & Compound Interest MADEEASY | 18. Ans. (d) Rs. 10+S.1. on Rs. 10 for 11 months = Rs.11+S. |. on Ret for(1+2+3+4+ + 10) months Rs. 10 + S.1. on Re 1. for 110 months. = Rs. 11 +S.1, on Re 1 for 55 months S.1.on Re 1 for 55 months = Re 1. 100x12 155) Rate 9 lanai, 19. Ans. (b) Lt the sum be Rs. x. Then 2XOXS | 2xBX5 | 2102 100 400" 100 or 78x= 156000 or x= 2000 1560 20. Ans. (¢) 180 x40 = x x30 orx = 240 21. Ans. (a) Let the money lent at 8% be Rs. x. then, xx8x1, (1550-2)x6% 100 100 of 2x +9300 =10600 or x = 650. 106 22. Ans. (d) Let the sum at 8% be Rs. x. Then, 2X59 , (1550-2) x8x3 700 700 or x= 800 =300 Money at 5% 800 Money at 8% ~ (1650-800) 23. Ans. (b) 201 st=Re (26002241) 520 =Rs| xT], which is an exact number of rupees So 24, Ans. (c) Lethe amounts iwestedbex.y zrespecvely. x4x 8 700 . then, 2X25 _ yx3x5 400 100 =10k, y= Bk a2 = : So, x: y:Z =10k Bass = 30:20:15 =6:4:3, 25. Ans. (b) Let these parts ie. x, y and [2189-(x+y)] Then, xxtx. 700" X2x4 100) _[2189-(«+y) 3x4 100) =2 oF 2y. <1 2yrctxd _(2189-dy)x3x4 100 100 or day = 2189 x 12, 218912 2189%12) _ 597, ( 44 Jes ‘Smallest part = Rs. 697. oooo MADEEASY * Reasoning & Aptitude | 106 Compound Interest Practice Exercise: | 1. The amount of Rs. 7500 at compound interest at 4% per annum for 2 years, is: (@) Rs. 7800 (b) Rs. 8100 (©) As. 8112 (@) Rs. 8082 2, Thedifference between the compound interest and the simple interest on a sum of money for 2 years at 12.5% per annum in Rs. 150, The sum is (2) Rs. 9000 (© Rs. 9500 3. The difference in compound interest, and simple interest on a certain amount at 10% per annum at the end.of the third year is Rs, 620. Whatis the principal amount? (@) Rs.40000 —(b) As. 12000 (©) Rs.10000 —(¢) As. 20000 4. ifthe compound interest on a certain sum at (b) Rs. 9200 (d) Rs, 9600 162% for 3 years is Rs. 1270, the simple interest on the same sum at the same rate and for the same period is: (@) Rs. 1200 (b) Rs. 1165 (0) Rs. 1080 (d) Rs. 1220 5. The compound interest on a certain sum at ‘5% per annum for 2 years is Rs. 328, The simple interest for that sum at the same rate and for the same period will be: (a) Rs, 320 (0) Rs. 322 (©) Rs. 325 (@) Rs. 326 6. Whatis principal amount which earns Rs, 132 ‘as compound interest for the second year at 10. "1 12. ‘A sum of money at compound interest amounts to Rs. 578.40 in 2 years and to Rs. 614.55 in 3 years. The rate of interest per annume is: (@) 4% (©) 5% 1 1 (© 63% (@) 85% A sum of money amounts to Rs, 4624 in 2 years and to Rs. 4913 in 3 years at compound interest. The sum is: (@) Rs, 4096 (©) Rs. 4260 (©) Rs. 4335 (@) As. 4360 ‘A sum of money at compound interest amounts to thrice itself in 3 years. In how many years wil it be 9 times itself? fa) 12 (b) 9 6 ) 8 In how many years will a sum of Rs. 800 at 10% per annum compounded semiannually become Rs. 926.10? 1 1 @) 25 ) 15 1 1 © 23 @ 15 To find out the total compound interest accrued on a sum of money after 5 years, which of the following informations given in the statements P and Q willbe sufficient? P: The sum was Rs. 20000. Q:: The total amount of simple interest on the sum after 5 years was Rs. 4000. (@) Only P is sufficient (©) Only Q is sufficient (c) Either P or Q is sufficient (@) BothP & Qare needed (e) Both P and Q are not sufficient. ‘The least number of complete years in which ‘a sum of money put out at 20% compound 10% per annum? interest will be more than doubled is: (@) Rs. 1000 (b) Fs. 1200 (3 4 (©) Rs.1320 (¢) Rs. 1188 os 6 MADE EASY ‘* Reasoning & Aptitude | 107 108 | -¢ Simple Interest & Compound interest 4 18, Atree increase annually by 5 thot its height. By how much will it increase after 2 years, if itstands today 64 cm high? (@) 720m (b) 740m (©) 750m (0) Btom 14. The compound interest on a sum for 2 years is Re. 832 and the simple interest on the same sum for the same period is Rs. 800. The difference between the compound interest and the simple interest for 3 years willbe : (@) Rs. 48 (0) 66.56 (©) Rs.98.56 —(d) None of these 15. A sum of money becomes Rs. 13380 after 3 years and Rs. 20070 alter 6 years on ‘compound interest. The sum is: (@) Rs.8800 ——(b) Rs. 8890 (©) Rs.8920 (a) Rs. 9040 16. A sum of Rs. 1100 was taken as a loan. This is to be repaid in two equal installments. If the rate of interest be 20% compounded annvally, then the value of each installments, is: (@) Rs. 842 (©) Rs. 720 (&) Rs. 792 (6) Fs. 700 17. The compound interest on Rs. 8000 at 15% per annum for 2 years 4 months, compounded. annually is: (@) Rs.2980 —(b) Rs. 3091 (©) Rs.3109 —(@) Rs. 3100 18. What annual payment will discharge a debt of Rs. 7620 due in 3 years at 162% per ‘annum compound interest? (a) Rs. 2540 (o) Rs. 3430 (0) Rs. 3260 (d) is. 3380 MALELASY 4 19. A sum of money invested at compound interest amounts to Rs. 800 in 3 years and As. 840 in 4 years. Whatis the rate of inierest per annum? (a) 2% (0) 4% © 5% (a) 10% 20. A sum of money becomes 8 times of itself in 3 years at compound interest. The rate of interest is: (@) 100% (0) 8% (©) 1% (@) Data inadequate Solutions 1. Ans. (0) . . it = Rs] 7500 x} 1+—— san mr 26 | 26 = eoox2222) rosie 2. Ans. (d) Let the sum be Rs. x. Then, 3. Ans. (d) Let the sum be Rs. x, Then, MADE EASY © Reasoning & Aptitude. | 108 She S09" 0 or x= 20000, Hence, the principal amount is Rs. 20000. }. Ans. (c) Let the sum be Rs. x. Then, so_ =| xx(t - oleate) ( -»)= tar “(216 216° 127x AER a 127 2 12 1270x216 127 Thus, the sum is Rs. 2160. or x = 2160. 50 1 St Re(2160%52x9x-4,) = Rs, 1080, Ans. (a) Let the sum be Rs. x. Then, + orai(ing fs SL=Rs (Ses) 700 6. Ans. (b) Let the principal at the end of first year be Rs. x. xx10x4 700 Now, let the original principal be Rs. P. Then, amount after 1 year Then, =182 or x= 1320. apy Px10x1 _1P 100 10 41P UP at 77 1320 1920x10 TP = Re. 1200. 7. Ans. (d) Interest on Rs. 578.40 for 1 year = Rs, (614.55 - 578.40) = Rs. 36.15. 100x36.15 1 Rate = (SBS)x 267% 8. Ans. (a) Interest on Rs. 4624 for 1 year = Rs, (6083.50 - 5290) = As. 793.50 100x793.50 1 Rate ={ 2X78 20 le, ae ( 4624 x4 Ye Now {t+ or xx et. agog 16 “16 '* Reasoning & Aptitude | 109 110 | © Simple Interest & Compound Interest MADE EASY So, n=4 years. 13, Ans. (3) 1 1 =(4x100}%= 122%. Increase % ( 00) 3 Let oie) =" (+ 4) =9. Height atter 2 years, 25 feos) fo -(001329)onesem ayT_(,, 8° BI] OB) 0) so soe eg Diff. in C.1. & S.1. for 2 years = Rs. 32. Hence, n=6 years, Sl. for 1 year = Rs. 400. 10. Ans. (b) Shon Rs.400 for 1 year = Fs.32. Rate=5% per half year. Let times 2n half years = n years. . Rate = (SA) wa . 400x1 Then, o00( +55) = 926.10 Hence, dif. in C., and S.. for 3d year 100, = 81. on Ret 832 oy ( 21)" 928.10 _ 9261 70) ~~ 800 8 =Rs, (exe 799% ‘) |= Rs.66.56. ‘Total difference = Rs. (32 + 66.56) # 2=8 orns = Rs, 98.56 11. Ans. (d) 15. Ans. (c) Clearly, both P and Q together are needed. Let the sum be Rs. x. then, Using P and Q rate can be calculated as Fi xf1+2) 19980 700, 100% 4000 Rate (San) =4% : Now,C. |. can be calculated. a (eB) = 20070 12. Ans. (b) ay On dividing, yet | 1+—— ncn. at (i 8) MADE EASY * Reasoning & Aptitude | 110 20070 _3 73380 2 3 3x5 = 19380 = x= s9200x2) = 020, Hence, the sum is Re. 8920. Ans. (c) Let the value of each installment be Rs. x. Then, (PW. of Rs. x due 1 year hence) + (P.W. of Rs, x due 2 years hence) = Rs. 1100. 400 5x | 25x or +E = 1100 85x = 96% 1100 +-(22@) OU 6.720. Ans. (c) Time = 2 years 4 months = 24 years 1 = 2-years. years 2 Che 1 18. Ans. (b) Let each installment be Rs. x. Then, — 50 50 ¥ (+5eha) (+535) ( 50, 6 3x 100, 6x 36x | 216x Tag t gag 762. 294x + 252 + 216x = 7620 x343 7620x343, Ten 9480 ‘Amount of each installment = Rs.3430. 19. Ans. (0) Interest on Rs. 800 for 1 year = Rs, (840-800) = Rs. 40. 100 40) ‘800% Fate=( = 5 6y 43° t+ Rs. sonos{14-15 x] 145 | - 8000 aerial R o Be . 23 23 21 he toon z5 2 soto) o R=100 = Re. 3108, ooo MADE EASY ‘ Reasoning & Aptitude | 111 In real life situations we come across practical problems of accomplishing the given project in prescribed time limit, Since elticiency of diferent person is different, the management has to lake Proper note of it before distributing the task to subordinates or executives. We discussed all ‘such problems under the heading Time and work’ Theory Let a person A can finish a task in x days and person B can finish it in y days. Then Aan finish — Part of work in a day and 8 can 1 finish partof workin a day both wll be working together to finish the task as soon as possible then in @ day they can finish 1 x Example 1: A can finish the task in 8 day and B san finish the same task in 12 days. How many days are required to finish the task if both are ‘working together? Time & Work Solution: A can finish the task in 8 days 1 S0 in a day A can finish = part of work 8 can finish the task in 12 days 80 in a day 8 can finish poet of work it bboth are working simultaneously then ina day they can finish. 1_ 0 5 1 = he 727 Bid 7 Patt of work so the 24 whole task will be finished in = days ie 4.8 days. Example 2: x can complete a work in 8 days, y can do the same work in 24 days. If both are working together then the task will be finished in how many days? Solution: Here x= 8, y= 24 Total number of days required MADE EASY * Reasoning & Aptitude | 113 114 | * Time & Work When A and B work in alternate days. Case | ‘starting from A. Let A can finish the task in 8 days and B can finish the same task in 10 days. How many days, are required to finish the task i both are working inalternate days.? Solution: ‘A can finish the task in 8 days 1 So A's workin a day = 5 part B can finish the task in 10 days $0.88 work ina day = part Both are working alternately starting from A. 14 then in two days they can finish 5 +75 part i ie. In first day A will finish 5 part 1 In second day 8 will finish 75 part 8+10_ 18 80 80 Itthey continue in such a manner together in 8 days they can finish 2 part of work. Stl eo 4 26 % 7p Pats remaining which is to be 1 finished by A in Sth days so, to finish = part Aneeds days. = So, together they can finish the task in of days. Case Il: In the above question calculate number of days MADE EASY ‘required to finish the task by A and B if both are working alternately starting from B Solution: The explanation remains the same from ist day to th day. In 9th day 3 still remains which has to be finished by 8. 1 4 a B. Bean nish 75 partot workin = 1 day So, together they can complete the task in 9 days, Concept of Negative Work ‘Supposing A and B are working to build a wall while C is working to break the wall. In such case the wallis being bul by A and B while itis broken by C. Here if we consider the work as building of the wall, we can say that C is doing negative work. Example: A can build a wall in 8 days and B can build itin 12 days while C can completely destroy it in 24 days, Itthey start working at the same time, in how many days will the work be completed, Solution: The net combined work per day here is As work + B's work - C's work + 8°12 24° 24 Hence the work will be completed in 6 days, The concept of negative work cortimonly appears as a problem based on pipes and cisterns, where there inlet pipes and outher! leaks which are working against each other. Ex.1._ A tortoise can climb upto 5m in first hour. Next hour it climbs down to 4m. Itis trying toreach a plateform of 15m. Find number of hours require to the tortoise to climb up the plateform, MADE EASY ‘* Reasoning & Aptitude | 114 Sol Itis very olear that tortoise can climb up to.Sm per hour. Effective climbing in the last hour willbe Sr. Remaining time in which tortoise will climb Up to plateform can be calculated by simple logic. First hour climb up 5m next hour climb down ky 4m Effective work within 2 hour = tm. : Imolimbing requires 2 hours. 410m Climbing will require 20 hours. Total time required by tortoise to climb up to 15m is 20 + 1 = 21 hours. Ex2_A snails trying to reach at the top level cf wooden stick which is 95 om long, Snail can climb up to Som in frst hour but climbs down to 3 cm in next hour. Find the time required by snail to reach the top of wooden stick. Sol: Snail climb Som in last hour. Remaining time in which snail will climb stick can be calculated by simple method. Sem climbing up in first hour & cm climbing down in next hour ‘+ Effective work done by snail wil be 2om in2 hour. 90em can be climed in 90 hours. Total time required by snail will be 90+ 1 = 91 hours. Pipes and Cisterns . Pipes are connected to a tank or cistern and are "used to fill or empty the tank Tank Inlet: is a pipe which is used to fil the tank. In the given figure pipe A is inlet Outlet: is @ pipe connected toa tank andis used for emptying the tank. In the figure pipe is outlet 1. tfainlet can completely fill the empty tank in x hours, the part of the tank filed in 1 hour 2. If an outlet can empty the tank in y hours then the part of tank emptied in 1 hour = Q Se = AB x a5, time period, Similarly if efficiency is low, it Together they can finish requires more time to finish the same task than +405 4 that of stipulated time eta 7 Nady We will observe following examples to have a glimps of problems based on concept of So, it requires 12 days to finish the task efficiency, together Ex.1 Ais twise as efficient as B, and finish the Ex.4 Ais three times as efficient as B, and together they finish a task in 30 days, Find number of days required by B to finish the same task individually task 20 days earlier than B. Find number of days required to finish the task by A. MADE EASY * Reasoning & Aptitude | 117 118 | Time & Work Sol.: Let A requires x days then B requires 3x days ‘According to given condition 144 tet et x Bx 30 >» 4.1 ar 30 = RL, rea 3 x = 120 Ans. x5 Aisthrice as efficient as B and can finish the task 16 days ahead of B. Find number of days required to finish the same task, if both are working alternately starting from A. Sol: Let A requires x days then B requires 3x days ‘According to given condition Brox = 16 => 2x= 16 > x=8 Oe = 24 IF they are working alternately then in two days they can finish 4,421 324 > Pat So it requires 12 days to finish the task. | SOLVED EXAMPLES 1. Vinod can do 25% of a piece of work in 5 days. How many days will he take to complete the work ten times? (@) 150 days (b) 250 days (©) 200 days (©) 180 days Ans. (c) 25% part or ; part in 5 days then 1 part in 7 = 20 days alala days required to complete ten times work = 20x 10 = 200 days . men can do piece of work in 12 days. How | many men are needed to do the workin 18 days? (@) 9men (©) 6men (©) 4men (d) 2men Ans. (c) Here Man . Day = 6 x12 7 mx18= 72, m=75=4 A. can do a piece of work in 20 days and can do it in 15 days. How long will they take if both work together? @ 9 }covs © (3) days © 9f2}e08 (a) None of these Ans. (b) Here x = 20 and y = 18 days required = ry _ 20x15 900 44 4 x+y 20415 95 07 %¥ Nishu and Archana can do apiece of work in 10 days and Nishu alone can doit in 12 days, In How many days can Archana do it alone? (@) 60 days (b) 30 days (©) 50 days (6) 45 days ‘Ans.(a) ‘One day work of both = $ MADE EASY + Reasoning & Aptitude | 118 and Nishu's day work = $ ‘So, Archana's one day work 24 20 60 So, 60 days required to finish the work by Archna 5. 4men and 3 women finish a job in 6 days. And 5 men and 7 women can do the same {pbin 4 days. Howlongwil man and 1 women ‘take to do the work? @ %(Z)oa6 25{3 Jove © off}oar — s2f Z)oare And. (a) Let man completes m part in a day and woman completes w partiin a day then also 0 Sm+7w=> @ ater simplifying we get zoms t5w= 2 20m+28w=1 1 So13w= é i 78 from (1) and (3) we get —s 156 w @) S47 mewe Sy. 156" 78 cotreaie aay 22? . WB boys and 12 women can do a piece of work in 25 days. In how many days can the work be done by 6 boys and 11 women working together? (@) 15 days (0) 10 days (©) 12 days (@) Cannot be determined, Ans. (d) 1 88+ 12w= 5 () now to calculate 6B+tiw=? Itcan not be determined because we have 1 equation but two valiables, A can do a piece of work in 10 days and B can do the same work in 20 days. With the help of C, they finish the work in 5 days. How long will ittake for C alone to finish the work? (@) 20 days (0) 10 days (©) 35 days (@) 15 days ‘Ans. (a) JALAL 5 10 20 20 This is C’s one day work so 20 days are required for C alone to finish task ‘ACan do apiece of workin 20 days, He work at it for 5 days and then B finishes it in 10 more days. In how many days will A and B together finish the work? (@) 8 days (©) 10 days (©) 12 days (6) 6 days ‘Ans. (a) 1 ANS one day work = y= 20 1 1 ‘5 day jlete —x5 = —part in day a can complet + x5 = ‘pa i 3 remaining work = 7 part MADE EASY © Reasoning & Aptitude | 119 120 | © Time & Work MADE EASY B finish 3 part in 10 days 9. Twenty workers can finish a piece of work in 30 days. After how many days should 5 workers leave the job so that the work is completed in 35 days? (@) 5 days (o) 10 days (©) 18 days (@) 20 days ‘Ans. (c) Man x Day = Man . Day 20 x30 = 600 Let 5 workers leave after x days then 20r + 15 (35-2) = 600 Sr + 25 = 600 Sr = 75,x= 15 10. Subhash can copy 60 page in 10 hours; subhash and Prakash together can copy 300 ages in 40 hours. In how much time can Prakash copy 30 pages? (@) 13h (o) 12h (© 1th (@ 9h Ans. (b) ‘Subhash in 10 hours can copy 50 pages 50, in thour 225 int hour > = 5 pages Both can copy 800 pages in 40 hours So, in th = = 40 Clearly prakash can copy 2.5 pagesin 1 hour 80.30 pages in z= x30 = 12 hours 11. Sashi can-do a piece of work in 25 days and Rishi can do it in 20 days. They work for 5 days and then Sashi goes away. In how many more days will Rishi finish the work? (@) 10 days (©) 12 days (©) 14 days (d) None of these Ans. (d) Shashi's days work Rishi's days work = together they can finish z +z 2 9 Patina day 9 45 » Work in § days =55%5=755 part 55 Jap Pat Is remaining, Rishi can finshit in (i) 400. (ws) 20, 12. Priya can do 1/2 of the work in 8 days while Preeti can do 1/3 of the work in 6 days. How Jong will it take for both of them to finish the work? 1¥days 88, 144 (a) soy ©) 7 das © Baays (6) 8 days Ans. (b) @) , 2 Priyals days work = Preeti’ days work MADE EASY 1 ASL. 4 days 6 18° Together they can finish in 144 Fr days 13, Mano takes twice as much time as Ajay and thrice as much as Vijay to finish a piece of work. Together they finish the work in 1 day. What is the time taken by Manojto finish the work? (@) 6 days (©) 2days Ans. (a) M:A:V 2:14:28 or 6:3:2 Let Mano] takes 6x days Ajay takes Sr days 144 and vi then 44 vilay 2 days then + <4 5 (0) 3 days (d) None of these Sor=t Manoj will take 6days 14, Apurva can do a piece of work in 12 days. ‘Apurva and Amit complete the work together and were paid Rs. 54 and Rs. 81 respectively How many days must they have taken to ‘complete the work together? (@) 4 days (0) 4.5 days (©) 48 days (0) 5 days Ans. (c) Payment ratio = 54:81 1 days work = 75 54:81 HT 12x8 together they will take = °8 % 1a+8 4.8 days 16. Raju is twice as Vijay. Together, they finish the work in 14 days. In how many days can Vijay alone do the same work? (@) 16 days (0) 21 days (©) 32 days @) 42 days Ans. (d) Let Raju can do the work in xdays then vijay in 2 days together they can complete in xxx = 14 days x= 21 days, 2 = 42 days 16. If 12 men and 16 boys can do a piece of work in 5 days and 13 men and 24 boys can doit in 4 days, compare the daily work done by aman with that done by a boy? @ 1:2 (&) 1:3 @ 2: @) 3:1 Ans. (c) 12M+16B= (a) 13M+24B= (2)also MADE EASY © Reasoning & Aptitude | 121 122 |» lime & work 17. Acan do a work in 10 days and B can do the same work in 20 days. They work together for 5 days and then A goes away. Inhow many more days will B finish the work? (a) 5 days (b) 6.5 days (©) 10 days @ shcays Ans. (a) In 5 days the can complete 3 worm zoe 4 Remaining is + part B wiliish tin 1 = 5 days 1720 18. 15 men could finish a piege of work in 210 days. But at the end of 100 days, 15 additional ‘men are employed. In how many more days will the work be complete? (a) 80 days (b) 60 days (©) 85 days (d) 50 days Ans. (c) Man x Day = Man . Day 15.210= 3150 Softer 100 days 15. 100 = 1500 will be completed ‘emaining work = 1650 Man . Day Man . Day 1650. 30. Day = 1650 Day = 55 19. In a fort there was sufficient food for 200 soldiers for 31 day. After 27 days 120 soldiers left the fort. For how many extra days will the rest of the food last for the remaining soldiers? (@) 12 days (b) 10 days (©) 8 days (0) 6 days ‘Ans. (b) Man xDay = Man.Day 200 x31 = 6200 at. After 27 days *200 x27 = 5400 Man. Day is finished ). A cistern is normally filed in 5 hours. However, it takes 6 hours when there is leak i its bottom. I the cistern is ful, in what time shall the leak empty it? (@) 6h (o) 5h (©) 30h (@) 15h Ans. (c) In one hour 2 pati tiled now leak can ‘empty in x hour then += 30 hours Pipe A and 8 running together can fila cistern in 6 minutes. If B takes 5 minutes more than Atoll the cistern, then the time in which A and B will fil the cistern separately will be respectively? (@) 15min, 20min (6) 15 min, 10min (©) 10min, 15min (4) 25 min, 20 min Ans. (c) Let pipe A can fil in x and pipe B in x+5 minutes MADE EASY © Reasoning & Aptitude | 122 22. Acister is normally filled in 6h but takes 4h onger to fill because of a leak in its bottom. If the cistern is full, the leak will empty it in how much time? (@) 15h (db) 16h © 20h (6) None of these ‘Ans. (a) Let leak empty in x hours Now 23. If three taps are open together, a tankis filed in 10 fh, One of the taps can fill in 5 h and another in 10h. At what rate does the 3rd pipe work? (@) Waste pipe emptying the tank in 10h (0) Waste pipe emptying the tank in 20 h (c) Waste pipe emptying the tank in 5 h (Q) Fills the tank in 10h Ans. (c) Le. pipe can empty in 5 hours 24, There are wo pipe in a tank. Pipe Ais for filing the tank and Pipe B is for emptying the tank. if can fllthe tank in 10 hours and B can empty the tank in 18 hours then find how many hours will it take to completely fil a half empty tank? (@) 30hours (©) 1Shours (©) 20hours (0) 33.33hours ‘Ans. (b) 14054 10 15° 160” 30 So it takes 30 hours to fill the tank and 15 hours to fil half the tank. 25. There are three Taps A, B and C in a tank. They can fill the tank in 10 hrs, 20 hrs and 25 hrs respectively. At first, all of them are ‘opened simultaneously. Then alter 2hours tap Cis closed and A and B are kept running. After the 4th hour, tap B is also closed. The remaining work is done by Tap A alone. Find the percentage of the work done by Tap A by itselt (@) 32% (b) 92% (©) 75% (d) None of these Ans. (d) ATS ]e ee nous |} 35 | a5 lop Pat | i741 if wt? hows |= | 5 | 5 |p Dat tft oe thous || 55 1x] ae | pean Tt 3” [48 Wvrous | «| = [Bom So remaining part after 4 hour is 32 op 73:2 hours a 10 % of work done by A (10+10+10+10+32) cess) «100 = 72% a0 x 100 = 72 Oooo Time & Work Practice Exercise: | 1. 10 men can complete a piece of work in 15 days and 15 women can complete the same work in 12 days. If all the 10 men and MADE EASY ‘* Reasoning & Aptitude | 123 124 | © Time & Work MAUE EADY 18 women'work together, in how many will the work get completed? 2 1 62. 4 (@) 63 days (o) 8 days 2 (©) 73 days (d) None of these 2. Acan do 1/9 of a work in § days and B can do 2/5 of the work in 10 days. In how many days both A and B together can do the work? @ 192 days (e) 92 days © 182 days (d) Noneof these 3. Ais thrice as good as B and is therefore able to finish a piece of work in 60 days less than B. Find the time in which they can do it, working together, (@) 228 cays (0) 22% days (©) 24 days (0) None of these 4. Ramesh takes twice as much time as Mahesh ‘and thrioe as much time as Suresh to complete job. It working together, they can complete the job in 4 days, then the time taken by each of them separately to complete the work is (@) 36, 24 and 16 days (b) 20, 16 and 12 days (©) 24, 42 and 18 days (@) None of these 5. Smencancompletea workin 2 days, 4 women can complete the same work in 3 days and 5 children can do tin days. 1 man, 1 women and 1 child, working together, can complete the work in 10, ‘AandB can finish a piece of work in 30 days, B and C in 40 days while C and A in 60 days. A, B, C together can do the workin @ 2d days (b) 20k days 2 (©) 265 days (6) None of these ‘Ajay and Sunil can do a piece of work in 10 days, sunil and Sanjay in 15 days and Sanjay and Alay in 20 days. They together ‘work at it for 6 days and then Ajay leaves ‘and Sunil and Sanjay go.on together for 4days ‘more, If Sunil then leaves, how long will Sanjay take to complete the work? (@) 12days {b) 10 days (©) 16 days (6) None of these Bansal, Gupta and Singhal together can complete a work in 4 days. if Bansal and Gupta together can complete the work in at days, Gupta and Singhal together can do itm 8 days, then Gupta alone can complete the work in (@) 16 days, (©) 20days Bindal can finish a work in 10 days. Jindal is twice as efficient as Bindal. If they work together, in how many days, the work willbe finished? (b) 12 days (d) None of these 1 2 (@) 33 days (b) 55 days 1 (© 43 days (¢) None of these Ais 4 times as fast as B and is therefore able to finish a work in 45 days less than B. ‘Aand B, working together, can complete the (@) 6 days, (b) 4 days workin (©) B days (@) None of these (@) 12 days (b) 16 days (0) 8 days (@) None of these MADE EASY * Reasoning & Aptitude | 124 11. Working 7 hours daily 24 men can complete apiece of workin 27 days. In how many days would 14 men complete the same piece of work working 9 hours daily? (@) 36 days (©) 30 days (©) 32.days (@) None of these 12, 45 men complete a piece of work in 30 days working 12 hours a day. In how many days will 69 men complete the work working 10hours a day? (@) 27 days (b) 30 days (@) 24 days (A) None of these 18. 4men or 6 women can finish a piece of work in 20 days. In how many days can 6 men and 11 women finish the same work? (@) 9 days (b) 6 days (©) 7days (0) None of these F 14, Acan do a piece of work in 10 days, while B alone can doitin 15 days. They work together for 5 days and the rest of the workis done by Cin 2days. I they get As. 450'for the whole work, how should they divide the money? (@) Rs. 250, Rs. 100, Rs. 100 (0) Rs. 225, Rs. 150, Rs. 75 (©) Rs. 200, Rs. 150, Rs. 100 (@ Rs. 175, Rs. 175, Rs. 100 15.A does half as much work as B in three- fourths of the time. If together they take 18 days to complete a work, how much time shall B take to do it? (@) 30 days (0) 35 days (¢) 40 days (a) 66 16. Twomen undertake to do a piece of work for is, 600. One alone could do it in 6 days and the other in 8 days. With the assistance of @ boy they finish itin 3 days. Boy's share should be 17. 5 men and 2 boys working together can do four times as much work per hour as @ man and a boy together. The work done by a man and a boy should be in the ratio. (@ 1:2 (o) 2:1 (© 1:3 (@ 4:1 18. A and B working separately can do a piece of work in 9 and 12 days, respectively. If they work fora day altenately, A beginning, in how many days the work will be completed? i 1 4 tot (2) 105 days (b) 107 days 1 © 108 days (q) 103 days 19. A and B can do a piece of work in 45 and 40 days, respectively. They began the work together, but A leaves after some days and B finished the remaining work in 23 days. Alter how many days, did A leaves? (@ 6 days (6) 8 days (©) 9 days @ 12 days 20. A can do a piece of work in 30 days, 8 in 50 days and C in 40 days. If Ais assisted by B on one day and by C on the next day alternately work will be completed in 32 2 (@) 173 days () 195 days 31 ©) 16Stdays 183 days Solutions 1. Ans. (a) Here, x = 15 and y = 12 Working together, 10men and 15 women will complete the work in (@) Rs. 300 (b) Rs. 225 (©) Rs. 75 (d) Rs. 100 days xty MADE EASY + Reasoning & Aptitude | 125 126 | * Time & Work = 18x12 ig, 20 or, 62 da “qe 12" “Sg Fs OS 2. Ans. (b) can dothe complete work in 5 x3 = 15 days 5 10 x2 10 x5 B can do the complete work = 25 days. Here, x = 15 and y = 25 ‘Aand B working together, can complete xy the work in x+y days = pe days ie. & or 93 cays 3, Ans. (b) Let B takes x days to do the work. Then, ‘takes (x 60) days to complete the work Since ratio of work done by A and B is 3:1, ratio of time taken by A and B is 1 : 3. => 3 (x-60)=x oF, :. Time taken by B to finish the work = 90 days and time taken by A to finish the 90 790 days. ‘AandB, working together, will complete work the work in 2 days xy 90x30 90+30 45 days, ie. © or, 224 days, 225 4. Ans. (a) Let Ramesh takes x days to finish the work ‘Thea, Mahesh takes 3 and Suresh takes = days to finish the same work, MADE EASY :, Ramesh, Mahesh and Suresh, working together, will complete the work in — a spt yetex OY ad +ixt = 8 bas peat 22°3 3 2 P16 x ie. or, ~ days ee Given: 5 = 4,» x= 24 6 Ramesh takes 24 days. Mahesh takes 4 & or 12 days and Suresh takes = = 8 days to finish the work, Ans. (b). 1/Man can complete the work in 5 x2 = 10 ‘days, 1 woman can complete the work in 4 x3 = 12 days. And 1 child can complete the work in5 x3 = 15 Hays 1 man, 1 woman and 1 child, working together, can complete the work in — days reer 10x12%15 10x12+12x15+15x10 4 days . Ans. (0) Hore, x= 30, y = 40 and z = 60. ‘A.BandC together will finish the work in (325) days payee. _{_2x80% 40x60 = 30%40+40x60+60%30) 19Y° MADE EASY © Reasoning & Aptitude | 126 144000 Eggi 268 days 7. Ans. (b) Ajay, sun and Sanjay, working together, can complete the work in (<2 wits) -( 2x10 15x20 ) 10x 15+ 15%x20+20x10 6000 ,, 120 Fey 1 Gy days Werk done by all of them together in «13, 120 ‘Also, work done by Sunil and Sanjay in 4 ie B 4 days = 75 Remaining work a 2 Which is to be done by Sanjay. ‘Now, Ajay Sunil and Sanjay, can complete the work in Peas and Ajay and Sunil ‘can complete the work in 10 days. Sanjay alone can complete the work in 120 x10 ASS = 120 days. 10-12 120 $ Cf the workis done by Sanjay in >> = 10 days. 8. Ans. (b) Bansal, Guptal and Singhal together can finish the work in 4 days. Bansal and Gupta together can do it in Bays, Gupta and Singhal together can do it in 8 days. Therefore, Bansal alone can ‘complete the work in om (8) ro x (Here, x= 4 and y = 8 days Also, Singhal alone can complete the work in Bx 1 |days = -( F—|das Here, x= 4 and y 2 = 24 days. Bansal and Singhal can complete the work in (2 2 -(2) days. = (es sa) [Here x = 24 and y = 8] = 6 days. Gupta alone can complete the work 6x4 6-4 9. Ans. (a) Here, x= 10 andk = 2 The time taken by Bindal and Jindal. ‘Working together, to complete the work. x 10 (, 10 rata ® Ronat days 10. Ans. (a) Here, * Reasoning & Aptitude | 127 128 |» Time & Work MADE EASY ‘Therefore, A and B, working together, can ‘complete the work in 4x45 days = 4848 days, We F654 OS ie, 12 days 11. Ans. (a) We have M, = 24D, = 27, W, = 1, t, M, = 14, Dp =2,Wp= 1b =9 M,D,t,W, = M,DjtW, = 24x27 x7 x1 = 14xD, x9 xt = D, = 36 days. 12. Ans. (a) We have, M, = 45, D, = 90, t,=12,W,=1, M, = 60, Dp =? =10, W, 9.45 x30 x12 xi = 60 xD, x 10 x1 => D, = 27 days 13. Ans. (b) Here, a and d= 11 Required number of days -( ab Jae *\Beraa)** b= 6,n = 20,0 =-6, ays = 6 days. ( 20x 4x6 6x64 4x11, 14, Ans. (b) 1 (AsB)'8 Sday’s work = (54) z Remaining work = 1 ~ 1 Cs 2 days’ work = = Now A's 5 days’ work : B's § days’ work " Y work = © C82 days! work = = C's share =As_[450 - (225 + 150)] = Rs. 75 16. Ans. (d) |B does x part of work in 1 hour then B does Sean g hour Now, A does work int hour 34 Itis given that + 33 = 75 14 80 = og Bompletewerkin 22 daysand Ain68 dys, 16. Ans. (c) ‘st man's 3 days’ work = 2nd man's 3 days’ work = 2 1,3).4 “(2"8) 8 They should get money in the ratio Boy's 3 days’ work = 1 ie4:34 ole 1 8 Boys share = Rs, 2600 As. 76 MADE EASY * Reasoning & Aptitude | 128 17. Ans. (b) Obviously, (5M +28) = 4(1M + 18) 2 M=28 Work done by a man and boy are in the ration 2: 4 48, Ans. (b) (A+ Bys2 days’ work 14427 Evident ¥ 977" 36 the work done by A and B during 5 paris of 7 35 days =5x 2-38 jays =x Remaining work = Now, : work is done by Ain 1 day 1 = i 3G Work, will be done by A in So, total time taken = 104 days 19. Ans. (c) 23 B's 23 days’ work = 5 2 40 40 Now, (A+B) 1 day's work Remaining work = 1- 7 30 Works done by A and Bin t day a Gp Works done by A and B in 960%17 _ 17x40 80, A left after 9 days. 9 days 20. Ans. (a) 1_8 1 30° 80" 160 (A+0y's tday’s work (A+B)'s 1 day's work = tal 7 Att 30 40 120 Work done in first 2 days 8,7 er 750" 720” 800 Work done in 82 = 16 cays 67x8 _ 67 600 75 a % On 17 th day(A + 8) will work and they will Work left = 1 8 finish 2 nish 755 7 160 (On 18th day (A + C) will work and they 32 120 willirish tin “-x == 5 days Whole work will be done in 1722 days 35 nooo '* Reasoning & Aptitude | 129 130 | © Time & Work MADE EASY Pipes & Cisterns 1, One tap can fill a eastern in 2 hours and ‘another can empty the cistern in hours. How long will they take to fil the cistern if both the taps are opened? (@) 6hours. (©) 6.30hours (©) Thours (@) None of these 2. Atap can fila tank in 25 minutes and another ccan empty it in 50 minutes. Find whether the tank will be filled up or emptied and in how many minutes? (@) Tankis filed up in 60 minutes (©) Tank is emptied in 25 minutes (©) Tankis filed up in 25 minutes (0) None of these 3. Two taps A and B can fill a tank in 10 hours and 15 hours, respectively. If both the taps are opened together the tank wil be full in (@) Shours (b) 6hours: (©) Shours (d) None of these 4, Two pipes can fill a tank in 10.hours and 12 hours, respectively. While a third pipe ‘emptied the full tank in 20 hours. If all the three pipes operate simultaneously, in how much time the tank willbe filed? (@) 7hours 30 minutes (b) Ghours 40 minutes (@) 8hours 30 minutes (d) None of these 5. Acistern is normally filed in 8 hours but takes 10. If two pipes function simultaneously, the reservoir will be filed in 12 hours. One pipe fill the reservoir 10 hours faster than the other. How many hours does the faster pipe teke to fil the reservoir? (@) 35 hours (©) 40hours (b) 30hours (@) None of these One fil pipe A is 3 times faster than second fil pipe B and takes 32 minutes less than the fill pipe B. When wil the cistern be full both pipes are opened together? (2) 28minutes —_(b) 24 minutes (©) 30minutes (d) datainadequate ‘Twopipes Aand B can filla cistern in 4minutes and 6 minutes, respectively. If these pipes are tured on alternately for 1 minute each how long wil it take for the cistern to fil? (@) 4min 40sec —(b) 3min 20sec (©) 4min 50sec (d) 3min30 sec A cistern is provided by two taps A and 8. A can fill it in 20 minutes and B in 25 taps minutes. Both the taps are kept open for ‘S minutes and then the second is turned off ‘The cistern willbe completely filed in another @ 11 minutes (>) 10 minutes (©) 1Sminutes (4) 12 minutes Acistern has two taps which filitin 12 minutes and 15 minutes respectively. There is also a waste pipe in the cistern. When all the pipes are opened, the empty cistern is full 20 minutes. How long will the waste pipe take to emply a full cistern? (@) 8minutes —_(b) 10 minutes (©) 12minutes —(d) 16 minutes 2 hours longer to fil because of a leak in its Solutions bottom. Ifthe cisterns full, the leak will empty itin 1. Ans. (a) (@) 35hours (b) 45hours Here, x= 2andy=3 (©) 40hours (6) None of these Part ofthe cistern filed in 1 hour MADE EASY * Reasoning & Aptitude | 130 Total time taken to fil the cistern = 6 hrs. 2. Ans. (a) Here, x = 25 and y = 60 :. Part of the tank filled or emptied in (minutes ‘Which is positive, therefore the tank will be filed. ++ Total time taken to fil the tank = 50 minutes 3. Ans. (b) Here, x= 10 and y = 16. The tank willbe full in -(Z ros 7) . (1 18 “(10+15, 4, Ans. (a) Here, x= 10, y= 12 and z The tank willbe full in xyx. -(2 2 rors rows or 6 hours. 20 aya ye -( 10x12x-20 era) -(3) hours or, 7 hours 30 minutes 5. Ans. (0) Here, x= Bandy =8 +2= 10, The leak will empty the cistern in = (2) hours -F 8x10 = (23) hours or, 40 hours, |. Ans. (b) Let one pipe take x hours to fil the reservoirs. Then, another pipe takes (x~ 10) hours. = x(x~10) = 12x+x-10) = x?-34r+120=0 OF (x-30)(x-4)=0 x=Q0orx=4 ‘The faster pipe takes 30 hours to fill the reservoir Ans. (b) Here, k= 3 and.x = 32 ke Cistern willbe fullin = —“- minutes (e-P = SSE ites = 24 minut age Mites = 24 minutes Ans. (a) ‘As the pipes are operating alternately, thus their 2 minutes job is 141-5 46712 Inthe next 2 minutes the pipes can fill another 3 part of cistern, Therefore, in 4 minutes the two pipes which are operating alternately att. ‘The part of the cistern left unfille MADE EASY * Reasoning & Aptitude | 131 132 | * Time & Work 1 Pipe A can fil {of the cistern in 1 minutes. Pipe A can fill got the cistern in axe 8 minutes $5 mina Total time taken to fill the cistern 2_42 ase eae +5245 min (Or, 4 minutes 40 seconds. 9. Ans. (a) 1,19 Part illed in t minute 35+ 5 = 755 9 Part filed in 5 minutes = x 400 Unfilled part = 155 This is to be filled by A alone and hence will it be filed in 20x55 = 11 minutes. 10. Ans. (b) Work done by waste pipe in 1 minutes (b+4)-4 = 2" 15)" 20 Waste pipe can empty the cistern in 40 minutes. oooo MADE EASY Pipes & Cisterns Practice Exercise: II Two taps A and B can fill a tank in 10 hours and 15 hours respectively. If both the teps are opened together, the tank will be ful i: (a) Shrs. (0) 6hrs. © 12tms — @ 7h 2 2 Two pipes can fill a tank in 10 hours and 12 hours respectively while a third pipe empties the full tank in 20 hours. If all the three pipes ‘operate simultaneously, in how much time the tank will be filed? (@ Ths (©) 8hrs (©) 7hrs 30min. (d) 8hrs 30 min, ‘An electric pump can fill a tank in 3 hours. Because of aleakin the tank, ittook 3¥ehours to fil the tank. The leak can drain out allthe water of the tank in: (@) 10%hvs (0) 12his (c) 2thrs. ~ (d) 24 hrs: Taps A and B can fill a bucket in 12 minutes ‘and 15 minutes respectively. If both are opened and Ais closed after 3 minutes, how much further time would it take for B to fillthe bucket? (@) 7 min. 45sec (b) 7 min 18sec (©) 8min. Ssec (4) 8min 18 sec If two pipes function simultaneously, the reservoir will be filed in 12 hours. One pipe fills the reservoir 10 hours faster than the other How many hours ittakes the second pipes to fil the reservoir? (@) 25hrs (0) 28hrs (©) 30hrs (0) 35s MADE EASY * Reasoning & Aptitude | 132 6. 12 buckets .of water fill a tank when the capacity of each bucket is 13.5 liters. How many buckets will be needed to fill the same. tank, ifthe capacity of each buckets 9 liters? @s8 (o) 16 (©) 16 (@) 1 7. Bucket P has thrice the capacity as bucket Q. It takes 60 tuins for bucket P to fil the ‘empty drum. How many turns it will take for both the buckets P and Q, having each turn together to fill the empty drum? (@) 20 (b) 40 ©) 45 (@) 9 8, Two pipes A and B can fill a cistern in 12 minutes and 15 minutes respectively but a thitd pipe C can empty the full tank in 6 minutes, A and B are kept open for § minutes in the begining and then C is also opened. in What time is the cistern emptied? (@) 30min. (0) 33 min. © 37min (@) 45min 9. Three pipes A, B and C can fill a tank in hours. After working at ittogether for 2hours, Cis closed and A and 8 can fil the remaining part in 7 hours. The number of hours taken by C alone to fill the cistern, is: @ 10 (b) 12 © 14 (@) 16 10. A leak in the bottom of tank can empty the full tankin 8 hours. An inlet pipe fills water at the rate of 6 liters @ minute. When the tank is full, the inlet is opened and due to the leak, the tank is empty in 12 hours. How many ters does the cistern hold? (@) 7580 (©) 7960 (©) 8290 (@) 8640 11. A cistern has two taps which fil it in 12 min. and 15 min, respectively, Theres alsoa waste pipe in the cistern, When all the three are ‘opened, the empty cister is fullin 20 minutes. How long will the waste pipe take to empty the full cistern? (@) min (©) 12min (0) 10min (A) 16min 12. Two pipes A and 8 can fill a cistern in 12 minutes and 16 minutes respectively. if booth the pipes are opened together, then after ‘how much time 8 should be closed so that the tank is full in 9 minutes? @ 33 min (©) 4min 1 3 (@) 45 min (@) 43 min Solutions 1. Ans. (b) 1 AS 1 hour's work= ; LBs hours work = T= (A+ B)s 1 hour's work (S+4 sot 10°16)" 306 Both the taps can fill the tank in 6 hours. 2. Ans. (c) Net part filed in 1 hour -(S+4 4) § 22 “(40 "12" 20)" 60° 18 The tank willbe ulin hs = T hrs. 30 min. 3. Ans. (c) Work done by the leak in 1 hour w4 2h Leak will empty the tank in 21 hours, MADE EASY * Reasoning & Aptitude | 133 134 | © Time & Work 4. Ans. (d) Patt filled in 3 min, = B min, 15sec Remaining partis filed by 8 in 8 min, 15 sec. 5. Ans. (c) Let the reservoir be filled by first pipe in x hours, The second pipe will flit in (x +10) hours. xH10+ 1 727 Geri) 12 > (x-20)(x+6)=0 x =20 Second pipe takes 30 hrs to fill the reservoir 6. Ans. (d) Capacity of the tank = (12x 13.6) litres = 162 litre. Capacity of each bucket = 9 tres Number of buckets needed = (2 7. Ans. (c) Let capacity of P be x litres. Then, capacity of Q = 3 litres. Capazity of the drum = 60x litres. Required number of turns MADE EASY 60 x (3) 8. Ans, (d) Par filled in § min waft. 1) fe. 9)-3 = 5-35) (5«5) a Pert emptied in 1 min., when ll the pipes are opened a) e 3) 4 A+t)-(t-S)-2 16) (6 20)" 6 Now, 4 part is emptied in 4 min Sart will be emptied on (cox) = 45 min, 9. Ans. (c) Patt filled in 2hours = Remaining part = (* (A+B) 7 hour's work < (A+B)'s 1 hour's work = 2/21 C's t hour's work = [(A+B+C)' 1 hour's work] - (A+B)'s 1 C.alone can fill the tank in 14 hours. 10. Ans. (d) ‘Work done by the main 1 hour “G3 1 a” 70)" 24 Work done by the inlet in 1 min. MADE EASY ‘* Reasoning & Aptitude | 134 1 Volume of =775 part = 6 litres Volume of whole = (1440 x6) litres = 8640 litres, 11. Ans. (b) Work done by waste pipe in 1 min, at. (3 2)--2 20°12" 15)" 10 [-Ve sign means emptying} Waste pipe will empty the full cistern in 10min, oooo 12. Ans. (b) Let 8 be closed after x minutes. Then, Part filled by (A +B) in x min. + Part filled by A in (9 -x) min, 8 OF 7x + 36 ~ 4x = 48 orx = 4. So, B must be closed after 4 minutes MADE EASY * Reasoning & Aptitude | 135 Time, Speed & Distance Time term ‘Time’ and ‘Distance’ are related to the speed or velocity of a moving object SPEED: We define the speed of an abject as the distance travelled by it in per unit time interval. It is obtained by dividing the distance covered by the ‘object, by the time it takes to cover that distance. Thus, Distance Travelled Time taken’ SpeedWvelocity = Notes: 41. Ifthe time taken is constant, the distance travelled is proportional to the speed, that is more the speed: more the distance travelled in the same time. 2. Ifthe speedis constant, the distance travelled is proportional to the time taken that is more the distance travelled; more the time taken at the same speed, 3. Ifthe distance travelled is constant, the speed isinversely proportional tothe time taken, that is, more the speed; less the time taken for the same distance travelled. Distance Time 1. Speer 2. Distance = Speed xTime Distance 3. Time = Distance me= “Speed Units of Measurement Generally, if the distance is measured in kilometers, we measure time in hours and speed inkilometer per hour and is written as km/hr; and if distance is measured in metres then time is taken in second and speed in metre per second and is written as m/sec. Conversion of Units 1000 meter 5 = misec. kemjhour = WOO meter rmlhour 60x 60 seconds 18 tm/s0o= “2 kmvtr Thus, x kmihr = (= x 3) misec = (+5) mises, and ,xm/sec = (« x mn Example 1: Raman can gover adistance of 30km in 8 hours in his bioycle, calculate the speed of Raman MADE EASY * Reasoning & Aptitude | 137 138 | * Time, Speed & Distance Solution: Disance Covered _ 30 Time taken 37 OY Example 2: Sanjeev can cover a distance of 250 km by travelling at the speed of 50 kmh. Find the time taken by him to cover the distance. Solution Speed = _ Distance _ 250 Time= “Speed ~ 50 = S hours Example 3: Atrain running at a speed of 90 km/hr ‘passes a pole on the platform in 20 seconds. Find the length of the train in meters Solution: ‘Speed of the train = 90 km/hr 5 =90x e 25 m/s Length of the train = speed of the train x time taken crossing the pole. = 25 x20 = 500m. Relative Speed CASE |: Same Direction When objects are moving in same direction relative speed is equal to difference between speed of two objects. Relative speed: s, SL, =8,-8, 5 i Case -I: Opposite Direction (face to face)/ (back to back) When objects are moving in opposite direction facing each other then Relative speed = (S, + S,) Ls <2 (fave to face) plateform, Railway tunnel etc) t Lithe St = Case Ill: Train crossing a moving object without length. > In opposite direction: br > In same direction: br ts (Sr So) Case IV: Train crossing a moving object with length: In opposite direction. Lytle 5,+8, > Insame direction pe ttl $+-S, Example 1: A train crosses a pole in 18 seconds If the length of the train is 300 metres, find the speed of the train, a Sr Solution: so t= 2 _ som/sec t 15, Example 2: Two trains of length 120 metres and ‘80 metres are running on parallel ines in opposite directions with the speed of 95 krvhr and 55 km/hr respectively. In what time will they pass each other. Ly tly Solutions: t= here S, + §, = 95 + 85 = 90 km/hr = 0x5. = 90x 55 = 25 m/sec = 25m/s 120+ 80 _ 200 - & seconds So, t= 25 (OS MADE EASY © Reasoning & Aptitude | 139 140 | © Time, Speed & Distance MADEEASY Boats and Streams The problems of boats and streams are also dependent on the basic equation of ime, speed and distance. Speed x Time = Distance. The speed of Boat islet S, The speed of stream is S The speed of movement of boatis dependent ‘on whether the boat bood is moving in which directions (©) While moving upstream (or against the flowof the water), the speed of moverient is given by Sy = S,- Ss (©) While moving down stream (or with the flowof the water) the speed of morement is given by Sp = Sp +S nooo | Solved Exampls 1. Walking at 3/4 of his normal speed, Ankur is 16 minutes late in reaching his office. The usual time taken by him to cover the distance between his home and his office is (@) 48minutes (6) 60 minutes (©) 42minutes _(@) 62 minutes Now St =T+16 4 1 & AT=16 T= 48 Sanjay and Naveen travel the same distance at the rate of 6 km per hour and 10km perf hour respectively. If Sanjay takes 30 minutes | longer than Naveen, the distance travelled by eachis (@) 6km (b) 10m () 75km (2) 20m Ans. (c) 18 2 =7.5km Two trains, Calcutta Mail and Bombay Mail, start at the same time from stations Calcutta ‘and Bombay respectively towards each other. Alter passing each other, they take 12 hours and 3 hours to reach Bombay and Calcutta respectively Ifthe Calcutta Mail is moving at the speed of 48 km/h. the speed of the Bombay Mail is (@) 24 km/m (©) 22kmin (o) 21 kmih (@) 96 km/n Ans. (d) Ans. (2) Calcutta Bombay Speed xTime = Distance ce SxT=D sn 12 meeting point here S is changed to Sesot must be i changed to =T to keep D constant ie eM 34 og cme 3sx4t=p 8, Vi2 2 a vh ars | MADE EASY * Reasoning & Aptitude | 140 4, Walking at 3/4 of his normal speed, a man takes 2(1/2)hours more than the normal ime, Find the normal time. (@) 75h (0) 6h ©) 8h (@) 12h ‘Ans. (a) S xT =D (costant) 35,4 Gsxgt=0 5. What is the time taken by Rohan to cover a distance of 360km by a motorcycle moving at a speed of 10 m/s @) 10h (b) 5h ©) 8h (@) 6h, Ans. (a) toms = 10% 2 = 96 kmh Speed 36 6. Rajdhani Express travels 650 km in § h and. another 940 km in 10 h. Whats the average speed of train? (@) 1880kmin —_(b) 68 kmh (©) 106kmm = (¢) 126k Ans. (c) ‘Average speed = T2taDistance Travelled s9e Speed = Total Time taken 94 E288 _ ook 7. Acar travels from A to B at V, km/h, travels ‘back from B to A at V, km/h and again goes back from A to B at V, km/h. The average speed of the car is, VN 2VNp ©) Yay ©) Bray 3VVp _3WNp ©) Wray @) Wray Ans. (c) Total Distance Average speed = “SE oraree 30 3 ay 0, 5,0 7,2 a+ Vy Ye Ve MMe Narayan Murthy walking at a speed of 20 km/h reaches his college 10 minutes late. Next time he increases his speed by § km/h. but finds that he is stil late by 4 minutes. What is the distance of his college from his house. @) 20km (o) 6m (©) 124m (6) None ct these Ans. (d) Let distance be D km D 7410 20° '* 0 o @ Jayshree goes to office at a speed of 6 km/h and returns to her home at a speed of 4 km/ hi. If she takes 10 hours in all, what is the distance between her office ang her home? (2) 24km. (0) 12km (©) 10km (@) None of these Ans. (a) MADE EASY * Reasoning & Aptitude | 141 142 | Time, Speed & Distance Let the distance be D krn then 1,1 ol gtg|=10 10 & 72 10. A motor car does a journey in 17.5 hours, covering the first half at 30 km/h and the second half at 40 km/h. Find the distance of 24 km the journey, (@) 684 km (b) 600 km (©) 120km. (@ 540km. Ans. (b) Here Total time = 17.5 hours let total Distance be 2D km ° D then 242 L475 "30° 40 14 o[S+a]- 175 7 Dapp = 175 175x120 7 D=300km Total Distance = 20'= 600 km D 11, Manish travels a certain distance by car « the rate of 12kmyh and walks back at the ret” of 3 kmh. The whole journey took § hours. What is the distance he covered on the car? (@) 12km (©) 30km () 15km. () 6km Ana. (a) Let Distance be D km o,0 12. A railway passenger counts the telegraph poles on the rail road as he passes them, The telegraph poles are at a distance of 50 meters. What will be his count in 4 houts, if the speed of the train is 45 km per hour (@) 600 (bo) 2500 (c) 3600 (d) 5000 Ans. (c) Total Distance covered = 45 x4 = 180km 180000 Number of Poles 3600 13, Two tains A and B start simultaneousy in the opposite direction from two points A and B and arrive at their destinations 9 and 4 hours respectively after their meeting each other. ‘Atwhat rate does the second train 8 tevel i the first train travels at 80 km per hour (@) 60kmn (©) 100km/h (©) 120km*h ——_() None of these Ans. (0) ——__ + A Meeting point 8 > Se S. . a 8 vo s 3 S,= 120 km/hour 14, A journay of 192 km takes 2 hours less by a fast train than by a slow train If the average) speed of the slow train be 16 kmph less than that of fast train, what is the average speed of the faster train, (@) 32kmph (b) 16kmph (©) 12kmph (6) 48kmph Ans. (d) * Reasoning & Aptitude | 142 ; i ; ; : let speed of fast train be S kmyhour then 192.192 This ype of questions can be directly solved bby going through option heare using equation (1) and puting options we get $ = 48 km/hour 16. Apassenger train takes 2h less fora journey ‘of 300 kilometres if its speed is increased by 5 kmph over its usval speed. Find the usual speed, ° (@) tokmph ——(b)12kmph * (©) 20kmph (@) 25kmph ‘ Ans. (d) going directly through options and using equation (1) we get $= 25 km/hour P16. A plane left half an hour later than the © scheduled time and in order to reach its » destination 1500 kilometre away in time, ithad to increase its speed by $3.93 per cent over 'ts usual speed. Find its increased speed. (@) 250kmph (b) 500 kmph (©) 750kmph (d) None of these Ans. (c) 93.39% = of normal speed i () ; 0 {going through option and (1) we'get 750 km/hour 17.A train moves at a constant speed of 120 kmjh for one kilometre and at 40 kmph for the next one kilometre. What is the average speed of the train. (@) 48kmph (©) 50kmph (©) 80kmph (6) None of these Ans. (d) istance covered A Distance covered erage Speed Time taken 141 =F 2x120x40 Worm | 120+40 = 60km/hour 18. Acar travels 1/Sof the distance on a straight road with a velocity of 10 kmih, the next 1/3 with a velocity of 20 km/h and the last 1/3 with a velocity of 60 km/h, Whats the average velocity of the car for the whole journey? (@) 18 km/h (0) 10min (©) 20kmm (@) 15kmih ‘Ans. (a) Average speed = Distance Covered Time taken Dj 0,0 3733 3 DTotTotr itt 3%20°3*i03"60 20°70* 60 = 18 km/hour 19, Walking at 3/4 of his usual speed, a man is 16 minutes late for his office. The usual time. taken by him to cover that distance is (2) 48 minutes (b) 60 minutes (c) 42 minutes (d) 62 minutes Ans. (a) S xT = Distance (constant) 4 Time has become 3 T © Reasoning & Aptitude | 143

You might also like